Anesthesia for Thoracic Surgery

¡Supera tus tareas y exámenes ahora con Quizwiz!

Which of the following is CONCURRENT PROBLEMS THAT SHOULD BE TREATED PRIOR TO ANESTHESIA IN COPD PATIENTS A. Laryngospasm B. Atelectasis C. infection D. Pulmonary edema

A.

Which of the following is the best device for absolute lung isolation A. DLT B. Bronchial blocker C. Endobronchial tube D. Univent tubes

A.

After the tracheal resection is completed, most patients are kept in a position of neck ______ to reduce tension on the suture line. A. Flexion B, Extension

A. Anesthesia for Tracheal Resection - Tracheal resection and reconstruction is indicated in patients who have a tracheal obstruction as a result of a tracheal tumor, previous tracheal trauma (most 1988commonly because of postintubation stenosis), congenital anomalies, vascular lesions, and tracheomalacia. For patients who have operable tumors, approximately 80% undergo segmental resection with primary anastomosis, 10% undergo segmental resection with prosthetic reconstruction, and the remaining 10% undergo placement of a T-tube stent. - Diagnostic studies are reviewed as part of the preoperative evaluation. The CT scan is a useful diagnostic tool to evaluate the degree, level, and length of the lesion. Bronchoscopy is one of the definitive diagnostic tests for tracheal obstruction. Bronchoscopy for a patient with tracheal stenosis should be carried out in the operating room where the surgical and anesthesia teams are present and ready to intervene should loss of airway occur. An advantage of rigid bronchoscopy over flexible bronchoscopy is that it can bypass the obstruction and provide a ventilation pathway if complete obstruction occurs. During surgery, all patients should have an invasive arterial catheter placed to facilitate measurement of arterial blood gases, as well as to measure arterial blood pressure. CVP catheters are only used if the patient requires cardiopulmonary bypass (CPB). - A variety of methods for providing adequate oxygenation and elimination of CO2 have been used during tracheal resection. The different alternatives include (1) standard orotracheal intubation, (2) insertion of a sterile SLT into the opened trachea or bronchus distal to the area of resection, (3) high-frequency jet ventilation (HFJV) through the stenotic area, (4) high-frequency positive-pressure ventilation (HFPPV), and (5) the use of CPB. Induction of anesthesia in patients with a compromised airway requires good communication between the surgical team and the anesthesiologist. The surgeon should always be in the operating room during induction and available to manage a surgical airway if this becomes necessary.213 A rigid bronchoscope must be immediately available. The patient should be thoroughly preoxygenated with 100% oxygen before induction. The airways of patients with congenital or acquired tracheal stenosis are unlikely to collapse during induction of anesthesia. However, intratracheal masses may lead to airway obstruction with induction of anesthesia and should be managed similarly to anterior mediastinal masses (discussed later). One airway management technique is to begin the case with rigid bronchoscopy and tracheal dilation and then to pass an SLT through the stenosis. This tube is withdrawn into the proximal trachea once the distal trachea is opened and a second sterile SLT is placed into the distal trachea by the surgeon. Ventilation is done via a sterile anesthetic circuit passed across the drapes into the surgical field. With a low tracheal lesion, a right thoracotomy provides the optimal surgical exposure. A sterile SLT is used to provide ventilation to the lung distal to the resection. After the posterior anastomosis is completed, the endobronchial tube is removed and the original SLT is advanced past the site of resection (Fig. 66-45). This technique can also be used for carinal resection. - A third technique for airway management during tracheal resection includes HFJV through a small-bore endotracheal tube or catheter.214 With this technique, a small-bore uncuffed catheter is placed through the stenotic area, and ventilation is accomplished by intermittently exposing the lung to a high flow of fresh gas through the catheter. Other techniques that have been used for oxygenation during distal airway resections include HFPPV, helium-oxygen mixtures, and CPB. After the tracheal resection is completed, most patients are kept in a position of neck flexion to reduce tension on the suture line. Replacement of the SLT by an LMA for emergence facilitates bronchoscopy if required. A thick chin-sternum suture may be placed for several days to maintain neck flexion, or a cervical splint may be used.215 A T-tube with the upper limb 0.5 to 1 cm above the vocal cords may be inserted at the end of surgery in cases where glottic edema is a concern, or for patients requiring 1989ventilatory support. If a tracheostomy is performed, it will be done distal to the anastomosis. Early extubation is highly desirable. If a patient requires reintubation, it should be performed with a flexible fiberoptic bronchoscope by advancing an SLT under direct vision over the bronchoscope and then placing it in the patient's trachea. The patient is kept in a head-up position to diminish swelling. Steroids may be useful in these cases to decrease airway edema. - One of the complications in the postoperative period is tetraplegia, with hyperflexion of the neck having been implicated as a potential cause. In these cases, it is necessary to cut the chin stitch. An infusion of propofol and remifentanil, with fiberoptic bronchoscopy guidance and full patient cooperation, can aid extubation.216

Lung volume reduction surgery is most useful in patients with A. Homogenous emphysema B. Heterogenous emphysema

A. Ventilation-Perfusion Assessment - When the preoperative lung function tests indicate that the patient is at an increased risk for perioperative complications, split lung function tests of ventilation and perfusion are valuable for predicting postresection lung function.53,58 Removal of a diseased portion of lung may not decrease overall lung function; in fact, it may improve it. The extent of pulmonary surgery has been found to correlate inversely with the intraoperative partial pressure of arterial oxygen (PaO2), where patients undergoing pneumonectomy had higher PaO2 than those undergoing lobectomy, which in turn were higher than those undergoing segmentectomy.3 This paradox is related to the corresponding amount of perfusion of the diseased lung. With larger, central tumors (such as would require a pneumonectomy), perfusion, and thus shunting under OLV, is diminished in comparison to a more peripheral lesion requiring a limited resection. Likewise, the results of perfusion scanning can predict the degree of hypoxemia during OLV, as the degree of perfusion to the operative lung is proportional to the degree of potential shunt produced when ventilation to that lung ceases.3 - Ventilation can be measured by having the patient inhale one vital capacity breath of a radioisotope and measuring isotope counts with multiple scanners placed over the chest wall. Intravenously injected radioisotope can be imaged to show the distribution of perfusion to all areas of the lung. After determining function in various areas of the lung, calculations can then estimate postresection function by multiplying the current function by the fraction of functioning lung that will remain postoperatively. Calculations based on segmental lung regions may help predict outcomes for patients undergoing lung volume reduction surgery. This procedure of removing emphysematous portions of lung to improve overall lung function has proved efficacious and particularly beneficial in allowing resection of cancerous lung tissue from patients in whom overall lung function studies would have contraindicated surgery.40 Lung volume reduction surgery is most useful in patients with heterogeneous emphysema (particularly when the emphysematous lobe is also the one containing the tumor), where removal of a lung segment or lobe will result in better pulmonary function overall. Incidentally, this effect is appreciated more often with upper rather than lower lobectomy, in which patients with a low preoperative FEV1 tend to demonstrate improvement in the FEV1 following upper lobectomy.59 - Dynamic MRI and quantitative CT are newer modalities used to determine postresection pulmonary function.60,61 Dynamic MRI traces the movement of oxygen or sulfur hexafluoride to reflect diffusing capacity.62 Quantitative CT is intended to provide more specific data than global measurements such as FEV1. This scan can be used to quantify low-attenuation (emphysematous) areas of lung to determine both overall proportion and regional distribution of disease. Results are comparable to FEV1 in predicting obstruction.63

Periods of deep hypothermic circulatory arrest DHCA are usually limited to ___ minutes for Pulmonary Thromboendarterectomy A. 10 B. 20 C. 30 D. 40

B. Pulmonary Thromboendarterectomy - Pulmonary thromboendarterectomy is a potentially curative procedure for chronic thromboembolic pulmonary hypertension (CTEPH). CTEPH is a progressive disorder that responds poorly to conservative therapy, and pulmonary thromboendarterectomy is its most appropriate treatment, with a perioperative mortality of approximately 4%, which is less than lung transplantation. The majority of patients with CTEPH present for medical evaluation late in the disease because many have not had an obvious episode of deep venous thrombosis or pulmonary embolus, and the progression of the disease is insidious. Patients present with severe dyspnea on exertion and signs of right-sided heart failure. Surgical candidates have hemodynamically significant pulmonary vascular obstruction (pulmonary vascular resistance >300 dynes/sec/cm-5), with potentially accessible proximal areas of thromboemboli. - An inferior vena cava filter is often placed prophylactically before surgery. Surgery is performed via a midline sternotomy with CPB with or without periods of deep hypothermic circulatory arrest (DHCA). Anesthetic management is essentially the same as for a patient with primary pulmonary hypertension for lung transplantation, with the exception that there is no need for lung isolation, so airway management is done with a standard ETT. Monitoring includes femoral and pulmonary artery catheters, TEE, processed electroencephalography, and rectal/bladder thermometers.240 - These patients are at risk of hemodynamic collapse because of right ventricular failure from hypotension during induction of general anesthesia. Induction may be performed with etomidate or ketamine to avoid hypotension. Support of systemic vascular resistance with noradrenaline or phenylephrine is usually required. If DHCA is used, it is preceded by mannitol and methylprednisolone to try to decrease cerebral cellular edema and to improve free-radical scavenging. The speed of warming and cooling is controlled on CPB to maintain a temperature gradient less than 10° C between the blood and bladder/rectal temperatures. Periods of DHCA are usually limited to 20 minutes. Massive pulmonary hemorrhage occurs rarely during CPB in these cases. Management of pulmonary hemorrhage is done according to the algorithm outlined in Figure 66-50. The instillation of phenlyephrine 10 mg and vasopressin 20 U diluted in 10 mL saline via the ETT may be beneficial. Postoperatively, the patients are kept sedated, intubated, and ventilated for at least 24 hours to decrease the risk of reperfusion pulmonary edema. Noradrenalin or vasopressin infusions may be used to elevate the systemic vascular resistance and decrease cardiac output to decrease pulmonary blood flow. Management of Tracheoinnominate Artery Fistula Hemorrhage • Overinflate the tracheostomy cuff to tamponade the hemorrhage. • If this fails: • Replace the tracheostomy tube with an oral endotracheal tube. Position the cuff with fiberoptic bronchoscopic guidance just above the carina. • Apply digital compression of the innominate artery against the posterior sternum using a finger passed through the tracheostomy stoma. • If this fails: • Slowly withdraw the endotracheal tube and overinflate the cuff to tamponade. • Proceed with definitive therapy: sternotomy and ligation of the innominate artery.

True or False Most adults with bronchial anastomoses have no problem with endotracheal intubation.

True Subsequent Anesthesia for the Pulmonary Transplant Recipient - Many lung transplant recipients subsequently require anesthesia for related or unrelated surgical problems.231 The frequency with which they present for surgery may be caused by complications of immunosuppression (e.g., infection, tumor, renal failure) or complications of the transplantation (e.g., bronchial stenosis, bronchiolitis obliterans). Most transplant recipients can be managed according to routine anesthetic practice including optimal perioperative respiratory care, antibiotic prophylaxis, and maintenance of immune suppression. Results of recent blood gas analysis, chest radiographs, and CT scans are vital in managing these patients. Most adults with bronchial anastomoses have no problem with endotracheal intubation. If endobronchial intubation or a DLT is used, a bronchoscopy should be performed first to assess the bronchial anastomoses, and the intubation should be done with fiberoptic guidance. Single-lung transplant recipients with native lung emphysema are a specific concern. They have a marked imbalance in pulmonary compliance, with the native lung highly compliant and the donor lung of normal or reduced (if there is rejection) compliance. However, the major proportion of the pulmonary blood flow is usually to the allograft. With standard methods of positive-pressure ventilation, dynamic hyperinflation of the emphysematous lung with hemodynamic instability and problems with gas exchange may develop in these patients. They may require the use of a DLT and independent lung ventilation techniques, reducing the airway pressure and minute ventilation in the native lung, if positive-pressure ventilation is required. Lung Volume Reduction - Although resection of bullae is a well-established thoracic surgical procedure, the use of multiple wedge resections to reduce lung volume and improve symptoms in severe emphysema (lung volume reduction) is a procedure in evolution. Depending on the patient and center, this procedure may be unilateral or bilateral and performed by thoracotomy, sternotomy, or VATS. The surgery is more effective in patients with heterogeneous lung disease, where the most severely affected areas (often apical) can be resected, than in patients with homogenous types of emphysema (e.g., α1-antitrypsin deficiency). Patients with extremely severe disease (FEV1 or DLco <20% predicted) have poor survival after surgery.232 This procedure is now most commonly seen as an option for patients with severe emphysema who have contraindications to transplantation. Immediate postoperative improvements in symptoms and pulmonary function are common, and many patients are able to discontinue or reduce home oxygen therapy. These changes are the result of a decompression of the airways and a reduction of airflow resistance and work of breathing,233 which results in a dramatic fall in auto-PEEP (intrinsic PEEP) with a corresponding increase in dynamic compliance. Despite the encouraging changes in early postoperative pulmonary function, the improvement in respiratory function as a result of this surgery is transient.234 However, this must be viewed in the context of the short life expectancy of patients with this degree of emphysema and the potential for improvement in their quality of life from the operation. Anesthetic management is similar to that for other patients with severe COPD having thoracic surgery (see COPD in Preoperative Evaluation, discussed earlier) with the risk of hypotension on induction caused by auto-PEEP and the need for excellent analgesia to avoid postoperative mechanical ventilation.235 This procedure is now performed in some centers using one-way valves placed bronchoscopically in the most involved segments to cause collapse of severely emphysematous distal lung regions,236 thus avoiding surgery.

The most important diagnostic tool for the patient with a mediastinal mass is A. XRAY B. MRI C. CT SCAN D. Ultrasound

C. Mediastinal Masses - Patients with mediastinal masses, particularly masses in the anterior or superior mediastinum, or both, present unique problems for the anesthesiologist. Patients may require anesthesia for biopsy of these masses by mediastinoscopy or VATS, or they may require definitive resection via sternotomy or thoracotomy. Tumors of the mediastinum include thymoma, teratoma, lymphoma, cystic hygroma, bronchogenic cyst, and thyroid tumors. Mediastinal masses may cause obstruction of major airways, main pulmonary arteries, atria, and the superior vena cava. During induction of general anesthesia in patients with an anterior or superior mediastinal mass, airway obstruction is the most common and feared complication. It is important to note that the point of tracheobronchial compression usually occurs distal to an endotracheal tube (Fig. 66-51), and it is not possible to forcibly pass an endotracheal tube through the airway once it has collapsed. A history of supine dyspnea or cough should alert the clinician to the possibility of airway obstruction on induction of anesthesia. Life-threatening complications may occur in the absence of symptoms in children. The other major complication is cardiovascular collapse secondary to compression of the heart or major vessels. Symptoms of supine presyncope suggest vascular compression. - Anesthetic deaths have mainly been reported in children. These deaths may be the result of the more compressible cartilaginous structure of the airway in children or because of the difficulty in obtaining a history of positional symptoms in children. The most important diagnostic tool for the patient with a mediastinal mass is the CT scan of the trachea and chest. Children with tracheobronchial compression noted to be greater than 50% on CT scan cannot be safely given general anesthesia.246 Flow-volume loops, specifically the exacerbation of a variable intrathoracic obstructive pattern (expiratory plateau) when supine, are unreliable247 for predicting which patients will have intraoperative airway collapse.248 Transthoracic echocardiography is indicated for patients with vascular compression symptoms. Management - General anesthesia will exacerbate extrinsic intrathoracic airway compression in three ways. First, reduced lung volume occurs during general anesthesia and tracheobronchial diameters decrease according to lung volume. Second, bronchial smooth muscle relaxes during general anesthesia, allowing greater compressibility of large airways. Finally, third, paralysis eliminates the caudal movement of the diaphragm seen during spontaneous ventilation. This eliminates the normal transpleural pressure gradient that dilates the airways during inspiration and minimizes the effects of extrinsic intrathoracic airway compression. - Management of these patients is guided by their symptoms and the CT scan (Boxes 66-15 and 66-16). Patients with "uncertain" airways should have diagnostic procedures performed under local or regional anesthesia whenever possible. Patients with "uncertain" airways requiring general anesthesia need a step-by-step induction of anesthesia with continuous monitoring of gas exchange and hemodynamics. This "NPIC" (noli pontes ignii consumere; i.e., "don't burn your bridges") anesthetic induction can be an inhalation induction with a volatile anesthetic such as sevoflurane or IV titration of propofol with or without ketamine, maintaining spontaneous ventilation until either the airway is definitively secured or the procedure is completed.249 Awake intubation of the trachea before induction is a possibility in some adult patients if the CT scan shows an area of noncompressed distal trachea to which the endotracheal tube can be advanced before induction. If muscle relaxants are required, ventilation should first be gradually taken over manually to ensure that positive-pressure ventilation is possible, and only then can a short-acting muscle relaxant be administered (Box 66-17). - Development of airway or vascular compression requires that the patient be awakened as rapidly as possible and then other options for the procedure be explored. Intraoperative life-threatening airway compression usually has responded to one of two therapies: either repositioning of the patient (it must be determined before induction if there is a position that causes less compression and fewer symptoms) or rigid bronchoscopy and ventilation distal to the obstruction (this means that an experienced bronchoscopist and equipment must always be immediately available in the operating room for these cases). The rigid bronchoscope, even if passed into only one mainstem bronchus, can be used for oxygenation during resuscitation (see Rigid Bronchoscopy, discussed earlier).250 Once adequate oxygenation has been restored, the rigid bronchoscope can be used to position an airway exchange catheter, over which an ETT is passed after the bronchoscope is withdrawn. An alternative technique to secure the airway with rigid bronchoscopy is to first mount an ETT over a small rigid bronchoscope (e.g., 6 mm) and then perform rigid bronchoscopy using the bronchoscope to deliver the ETT distal to the obstruction.251 Institution of femorofemoral CPB before induction of anesthesia is a possibility for some adult patients who are "unsafe" for NPIC general anesthesia. The concept of CPB "standby" during attempted induction of anesthesia is fraught with danger252 because there is not enough time after a sudden airway collapse to establish CPB before hypoxic cerebral injury occurs.253 Other options for "unsafe" patients include local anesthetic biopsy of the mediastinal mass or biopsy of another node (e.g., supraclavicular), preoperative radiotherapy with a nonradiated "window" for subsequent biopsy, preoperative chemotherapy or short-course steroids, and CT-guided biopsy of a mass or drainage of a cyst. The salient points in managing a patient with an anterior or superior mediastinal mass include254: 1. In virtually all children and adults with a mediastinal mass, diagnostic procedures and imaging can be performed, if necessary, without subjecting the patient to the risks of general anesthesia.255 2. An extrathoracic source of tissue for diagnostic biopsy (pleural effusion or extrathoracic lymph node) should be sought as an initial measure in every patient. 3. Regardless of the proposed diagnostic or therapeutic procedure, the flat, supine position is never mandatory. 4. In the high-risk child (Box 66-18) without extrathoracic lymphadenopathy or a pleural effusion, prebiopsy steroid therapy is justifiable.256 In this case, coordination with oncology, surgery, and anesthesiology is essential to organize timing of the biopsy. An alternative to preoperative steroids in the cooperative high-risk patient includes irradiating the tumor while leaving a small area covered with lead for subsequent biopsy. With improved awareness of the risk of acute intraoperative airway obstruction in these patients, life-threatening events are now less likely to occur in the operating room. In children, these events now tend to occur preoperatively if the patient is forced to assume a supine position for imaging. In adults, acute airway obstruction is now more likely to occur postoperatively in the recovery room.257 Vigilance must be maintained throughout the entire perioperative period.

This is the procedure of choice for the diagnosis and management of diseases of the pleura, nondiagnosed peripheral pulmonary nodules, and interstitial lung disease. A. EBUS B. Rigid bronchoscopy C. VATS D. Open thoractomy

C. Pulmonary Surgery - Any given pulmonary resection can be accomplished by a variety of different surgical approaches. The approach used in an individual case will depend on the interaction of several factors that include the site and pathology of the lesion(s) and the training and experience of the surgical team. Common thoracic surgical approaches and their generally accepted advantages and disadvantages are listed in Table 66-10. Minimally Invasive Thoracoscopic Surgery - VATS is the procedure of choice for the diagnosis and management of diseases of the pleura, nondiagnosed peripheral pulmonary nodules, and interstitial lung disease. Since the start of the modern era of thoracoscopic surgery in the early 1990s, VATS has been proposed as a less invasive approach than open procedures. Today it is a well-accepted and established procedure and has become the first-choice technique for lung biopsies, pleurectomies, sympathectomies, and other various pulmonary disorders.172 In addition, VATS may be used in a variety of other surgical procedures. Some centers routinely perform the majority of lobectomies under VATS. The outcomes for VATS lobectomy in patients with limited respiratory reserve seem superior to that for open thoracotomy. The spirometry threshold for increased risk for VATS seems to be a ppoFEV1 of 30% versus 40% for open thoracotomy (Fig. 66-40); however, it was not possible to define a threshold for ppoDLco.173 Other surgeries, such as spinal fusion and scoliosis, have been performed with VATS. The advantage of VATS, when compared with open thoracotomy, include: (1) reduced hospital length of stay, (2) less blood loss if no mishaps occur, (3) less pain, (4) improvement in pulmonary function when compared with open thoracotomy,174 (5) early patient mobilization with early recovery and rapid return to work and daily activities, and (6) less inflammatory reaction, as measured by cytokine response in patients undergoing VATS lobectomy compared with open thoracotomy.175 - VATS lobectomy has been demonstrated to be a safe and effective procedure to treat early-stage NSCLC.176 Thoracoscopic lobectomy is performed with a limited number of ports (1-3) and an access incision of approximately 5 cm in length.177 The advantage of the VATS 1981technique is that the ribs are not spread open. VATS procedures are commonly performed in the lateral decubitus position; however, bilateral VATS procedures, such as bilateral wedge resections or lung volume reduction, can be performed in the supine position. - Robotic thoracic surgery has been suggested as the logical advancement of VATS because of the perceived better three-dimensional vision and increased range of motion in the chest for the surgeon with robotic techniques (Fig. 66-41).178 Important points in anesthetic management are outlined in Box 66-11. Anesthetic Considerations for Robotic Thoracic Surgery • A protocol for rapid emergency undocking (<60 s) of the robot must be developed and practiced in advance. • Limit access to the patient. The position of lung isolation device needs to be confirmed before docking the robot. • Extensions to monitoring lines and anesthesia circuit may be required. • There is an increased need for intrathoracic CO2 insufflation with possible venous return and hemodynamic compromise. • Take precautions so that the operating room table cannot be moved while the robot is docked. • Potentially prolonged procedures therefore increase risk of positional neuropathies; fluid restriction advisable.

Recipients of lung transplantation include the following except A. COPD B. Cystic fibrosis C. Pulmonary fibrosis D. Lung cancer

D. Lung Transplantation - End-stage pulmonary disease is one of the most common causes of death. Lung transplantation is a definitive treatment for these patients. Indications and contraindications to lung transplantation are summarized in Box 66-12. Approximately 1500 lung transplants are performed annually worldwide; the number is limited by the supply of donor organs. Recipients fall into four major categories (by frequency of indication): 1. COPD 2. Cystic fibrosis (CF) (Fig. 66-49) and other congenital forms of bronchiectasis 3. Pulmonary fibrosis: idiopathic, associated with connective tissues disorders, other 4. Primary pulmonary hypertension There are also several other, rarer indications such as primary bronchoalveloar lung cancer, lymphangioleiomyomatosis, and so on.228 Depending on the patient's pathophysiology, there are several surgical options: single-lung transplantation, bilateral sequential (double) lung transplantation, heart-lung transplantation, and living-related lobar transplantation. An overall 5-year survival rate of 50% is the benchmark but depends on recipient age and diagnosis. Survival is generally better 1993with double-lung than with single-lung transplants, with the exception of older pulmonary fibrosis patients, for whom there is no outcome difference between the two procedures. - Anesthetic airway management is most commonly done with a DLT. The advantage of using a DLT in transplantation is that it allows direct continuous access to both lungs for suctioning, oxygenation, and examination of the bronchial anastomoses. Several advances have allowed the use of DLTs for transplantation. The use of segmental bronchial lavage at induction via an SLT, before placement of the DLT facilitates suctioning via the DLT in patients with copious secretions. Monitoring includes invasive arterial and pulmonary catheters and transesophageal echocardiography in most centers. Anesthetic maintenance is based mainly on IV infusions because of the frequent need for airway access (suctioning, bronchoscopy), which causes problems in maintaining a stable level of inspired anesthetic vapor. Despite advances in lung preservation techniques, it is optimal to limit the donor lung ischemic period to 4 hours. - There are wide variations in the frequency of use of CPB during lung transplantation; some centers always use CPB and some centers almost never use it. CPB temporarily decreases postoperative gas exchange, increases intraoperative blood loss, and increases the duration of postoperative mechanical ventilation. There is no net decrease in short- or long-term mortality attributable to CPB. Extracorporeal membrane oxygenation (ECMO) is used in some centers in place of CPB intraoperatively.229 ECMO is also used increasingly for postoperative respiratory support. - The intraoperative anesthetic complications depend, in large part, on the underlying lung disease. Emphysema patients are prone to hypotension on induction from positive-pressure ventilation (see COPD in "Preoperative Evaluation," discussed earlier). Problems in patients with CF include the inability to both deal with thick bronchial secretions and adequately ventilate these patients. CF patients, because they have both increased inspiratory and expiratory flow resistances, may benefit from slow inspiratory phase ventilation with a high airway pressure.230 Because of the severely 1994decreased lung compliance, this method of ventilation causes little hemodynamic compromise in this subgroup of patients, if air trapping is avoided. Other recipient disease-specific problems include: hemodynamic collapse on induction because of right-heart dysfunction in primary pulmonary hypertensives, the poor tolerance of pulmonary fibrosis patients for OLV, and the risk of pneumothoraces in patients with lymphangiomyomatosis.

Which of the following is indicated in bullectomy A. N2O use B. High respiratory rates C. High FO2 D. High PEEP

A & D. Bullectomy - Patients with bullous COPD are often treated with VATS to prevent pneumothorax or tension pneumothorax, which may result from ruptured bullae. To reduce the risk of bulla rupture, spontaneous ventilation is desirable under anesthesia until the chest is opened. Patients with severe cardiopulmonary disease may not be able to ventilate adequately under general anesthesia, however, and positive-pressure ventilation may be required. Small VTs, high respiratory rates, and high FiO2 can be delivered by gentle manual ventilation to keep airway pressures below 10-20 cm H2O.178 An alternative to positive-pressure ventilation is high-frequency jet ventilation, which is used to decrease the chance of barotrauma.179 - The use of N2O should be avoided in bullous disease because it rapidly enlarges the air-filled spaces. The anesthetic plan can involve general or epidural anesthesia, and is based on the patient's cardiopulmonary status and the anesthetist's desire to maintain spontaneous ventilation. The risk of bulla rupture persists even after surgery, so the same measures taken to avoid high airway pressure must be observed.

This is the standard operation for the management of lung cancer because local recurrence of the tumor is reduced compared with lesser resections. A. Lobectomy B. Sleeve lobectomy C. Pneumonectomy D. Wedge resection

A. Lobectomy - Lobectomy is the standard operation for the management of lung cancer because local recurrence of the tumor is reduced compared with lesser resections. Lobectomy is commonly performed via open thoracotomy or VATS. Sometimes, if the clinical staging of the lung cancer is advanced, an elective lobectomy is converted to a bi-lobectomy (right lung) or pneumonectomy during the operation. Although a posterolateral thoracotomy is the classic incision for lobectomies, anterolateral and muscle-sparing lateral incisions have also been used. - Postoperative analgesia is commonly performed with TEA or paravertebral analgesia (see Postoperative Analgesia later in the chapter). An arterial line for management of arterial blood gases and measurement of systemic blood pressure is used in all patients requiring an open thoracotomy or major VATS surgery. In addition, a large-bore IV catheter should be placed to facilitate rapid transfusion if necessary. Patients undergoing a lobectomy must be kept normothermic and normotensive and have an acceptable PaO2 and oxygen saturation, particularly during OLV. Patients should have a thermal blanket covering the lower extremities to prevent hypothermia and its deleterious effects during HPV. After the lobe and blood vessels have been dissected, a test maneuver is performed with the surgeon clamping the surgical bronchus to confirm that the specific lobe is extirpated. This maneuver is accomplished by unclamping the limb of the DLT connector of the respective side, or in the case of a bronchial blocker, by deflating the blocker balloon, and reexpanding the lung with manual ventilation. During 1982VATS lobectomy, because of the interference to the surgical field caused by reinflating the residual lobe, the anesthesiologist may be asked to fiberoptically inspect the bronchial tree to confirm patency of the bronchus of the noninvolved lobe(s). Once the lobectomy has been performed, the bronchial stump is usually tested with 30 cm H2O positive pressure in the anesthetic circuit to detect the presence of air leaks. A patient undergoing an uncomplicated lobectomy can usually be extubated in the operating room provided preoperative respiratory function is adequate (see Preoperative Assessment earlier in the chapter) and the patient is alert, warm, and comfortable ("AWaC"). - Pancoast tumors are carcinomas of the superior sulcus of the lung and can invade and compress local structures including the lower brachial plexus, subclavian blood vessels, stellate ganglion (causing Horner syndrome), and vertebrae. Lobectomy may require a two-stage procedure with an initial operation for posterior instrumentation/stabilization of the spine. During lobectomy, extensive chest wall resection may be required and massive transfusion is a possibility. Peripheral lines and monitoring should be in the contralateral arm to accommodate the frequent compression of the ipsilateral vessels during surgery.

Which is the preferred method for lung isolation for Pulmonary resection right side A. Right DLT B. Left DLT

B.

This is currently the most common NSCLC in both sexes. These tumors tend to be peripheral and often metastasize early in their course, particularly to brain, bones, liver, and adrenals. A. Large-cell undifferentiated carcinoma B. Adenocarcinoma C. Small-cell lung cancer D. Carcinoid tumor E. Pleural tumors

B. Adenocarcinoma - Adenocarinoma is currently the most common NSCLC in both sexes. These tumors tend to be peripheral and often metastasize early in their course, particularly to brain, bones, liver, and adrenals. They often invade extrapulmonary structures, including the chest wall, diaphragm, and pericardium. Almost all Pancoast tumors are adenocarcinomas. A variety of paraneoplastic metabolic factors can be secreted by adenocarcinomas, such as growth hormone and corticotropin. Hypertrophic pulmonary osteoarthropathy is particularly associated with adenocarcinoma. - Bronchioloalveolar carcinoma is a subtype of adenocarcinoma that is not related to cigarette smoking. In its early stages, it lines the alveolar membrane with a thin layer of tumor cells without destroying the alveolar architecture. Because of its low potential to spread outside of the lungs, multifocal bronchioloalveolar carcinoma can be treated by lung transplantation.51 Large-Cell Undifferentiated Carcinoma - This is the least common of the NSCLCs. It tends to present as a large, often cavitating peripheral tumor. The rapid growth rate may lead to widespread metastases, similar to adenocarcinoma. Small-Cell Lung Cancer - This tumor of neuroendocrine origin is considered metastatic on presentation and is usually regarded as a medical, not a surgical, disease. Surgery is only very rarely indicated. The staging system differs from NSCLC and is divided simply into a limited stage and an extensive stage. Treatment of limited-stage SCLC with combination chemotherapy (etoposide/cisplatin or cyclophosphamide/doxorubicin/vincristine) gives objective response rates in more than 80% of patients. In addition, these patients typically receive aggressive radiotherapy to the primary lung tumor and prophylactic cranial irradiation. Despite this initial response, the tumor invariably recurs and is quite resistant to further treatment. The overall survival rate is no more than 10%. Extensive-stage disease is treated with chemotherapy and palliative radiation as needed. - SCLC causes a variety of paraneoplastic syndromes owing to the production of peptide hormones and antibodies. The most common of these is hyponatremia, usually as a result of an inappropriate production of antidiuretic hormone. Cushing syndrome and hypercortisolism through ectopic production of adrenocorticotropic hormone are also commonly seen. - A rare neurologic paraneoplastic syndrome associated with small-cell lung tumors is the Lambert-Eaton (also called Eaton-Lambert) myasthenic syndrome caused by impaired release of acetylcholine from nerve terminals. This typically presents as proximal lower limb weakness and fatigability that may temporarily improve with exercise. 1951The diagnosis is confirmed by electromyography showing increasing amplitude of unusual action potentials with high-frequency stimulation. Similar to those with true myasthenia gravis, paitents with myasthenic syndrome are extremely sensitive to nondepolarizing muscle relaxants (see Chapter 18). However, they respond poorly to acetylcholinesterase inhibitors such as neostigmine.52 Of clinical importance, subclinical involvement of the diaphragm and muscles of respiration may exist. Thoracic epidural analgesia has been used after thoracotomy in these patients without complication. Neuromuscular function may improve after resection of the lung cancer. Carcinoid Tumors - These are part of a continuum of neuroendocrine tumors that form a spectrum of diseases from SCLC as the most malignant to typical carcinoid as the most benign. Five-year survival after resection for typical carcinoid exceeds 90%. Systemic metastasis is rare, as is the carcinoid syndrome, which is caused by the ectopic synthesis of vasoactive mediators, and is usually seen with carcinoid tumors of gut origin that have metastasized to the liver. Atypical carcinoid tumors are more aggressive and may metastasize. Carcinoid tumors can precipitate an intraoperative hemodynamic crisis or coronary artery spasm, even during bronchoscopic resection.53 The anesthesiologist should be prepared to deal with severe hypotension that may not respond to the usual vasoconstrictors and will require the use of the specific antagonists octreotide or somatostatin.54 Pleural Tumors - Localized fibrous tumors of the pleura are usually large, space-occupying masses that are attached to visceral or parietal pleura. They can be either benign or malignant. - Malignant pleural mesotheliomas are strongly associated with exposure to asbestos fibers. Their incidence in Canada has almost doubled in the past 15 years. With the phasing out of asbestos-containing products and the long latent period between exposure and diagnosis, the peak incidence is not predicted for another 10 years. The tumor initially proliferates within the visceral and parietal pleura, typically forming a bloody effusion. Most patients present with shortness of breath or dyspnea on exertion from this pleural effusion. Thoracentesis often relieves the symptoms but rarely provides a diagnosis. Pleural biopsy by VATS is the most efficient method to secure a diagnosis, and talc pleurodesis is performed during the same anesthetic period to treat the effusion. - Malignant pleural mesotheliomas respond poorly to therapy, and the median survival is less than 1 year. In patients with early disease, extrapleural pneumonectomy may be considered but it is difficult to know whether survival is improved. Recently, several groups have reported improved results with combinations of radiation, chemotherapy, and surgery. Extrapleural pneumonectomy is an extensive procedure that is rife with potential complications, both intraoperative and postoperative.55 Blood loss from the denuded chest wall or major vascular structures is always a risk. Complications related to resection of the diaphragm and pericardium are additional risks to those of pneumonectomy.

The most common concurrent illness in the thoracic surgical population is A. Lung cancer B. COPD C. Emphysema D. ARDS

B. Chronic Obstructive Pulmonary Disease - The most common concurrent illness in the thoracic surgical population is COPD, which incorporates three disorders: emphysema, peripheral airways disease, and chronic bronchitis (see Chapter 39). Any individual patient may have one or all of these conditions, but the dominant clinical feature is impairment of expiratory airflow.28 Assessment of the severity of COPD is made on the basis of the FEV1% of predicted values. The American Thoracic 1947Society categorizes stage I as greater than 50% predicted, stage II as 35% to 50%, and stage III as less than 35%. Stage I patients should not have significant dyspnea, hypoxemia, or hypercarbia, and other causes should be considered if these are present. Respiratory Drive - Many patients with stage II or III COPD have an elevated PaCO2 at rest. It is not possible to differentiate these "CO2 retainers" from nonretainers on the basis of history, physical examination, or spirometric pulmonary function testing.29 CO2 retention is related more to an inability to maintain the increased work of respiration required to keep the PaCO2 normal in patients with mechanically inefficient pulmonary function and not primarily resulting from an alteration of respiratory control mechanisms. It was previously thought that chronically hypoxemic/hypercapnic patients relied on a hypoxic stimulus for ventilatory drive and became insensitive to PaCO2. This explained the clinical observation that COPD patients in incipient respiratory failure could be put into a hypercapnic coma by the administration of a high concentration of oxygen (FiO2). In actuality, only a minor fraction of the increase in PaCO2 in such patients is caused by a diminished respiratory drive, because minute ventilation is basically unchanged.30 The PaCO2 rises because a high FiO2 causes a relative decrease in alveolar ventilation and an increase in alveolar dead space and shunt by the redistribution of perfusion away from lung areas of relatively normal matching to areas of very low ratio because regional hypoxic pulmonary vasoconstriction (HPV) is decreased31 and also as a result of the Haldane effect.32 However, supplemental oxygen must be administered to these patients postoperatively to prevent hypoxemia associated with the unavoidable fall in functional residual capacity (FRC). The attendant rise in PaCO2 should be anticipated and monitored. To identify these patients preoperatively, all stage II and III COPD patients need an arterial blood gas analysis. Nocturnal Hypoxemia - COPD patients have decreases in blood oxygen saturation more frequently and severely than normal patients during sleep.33 This is because of the rapid/shallow breathing pattern that occurs in all patients during rapid-eye-movement sleep. In COPD patients breathing air, this causes a significant increase in the respiratory dead space/tidal volume (VD/VT) ratio and a fall in alveolar oxygen tension (PAO2) and PaO2. This is not the sleep apnea-hypoventilation syndrome (SAHS). There is no increased incidence of SAHS in COPD.

This bronchial blocker has a steerable tip to facilitate advancement into the appropriate bronchus. A. Cohen blocker B. Univent tube C. EZ blocker D. Ardnt

Bronchial Blockers A bronchial blocker consists of a catheter with an inflatable balloon that blocks the bronchus of the operative lung. Blockers can be incorporated into a side channel of an ETT (e.g., Univent tube), or they can be separate devices that are inserted either through the regular ETT lumen or outside of it (more common in pediatrics) (Fig. 30.13). Common options for stand-alone bronchial blockers include the 8F Fogarty embolectomy catheter, the Cohen Flextip blocker, the Coopdech blocker, the EZ blocker, and the Arndt Bronchial Blocker (formerly known as the Wire-Guided Endobronchial blocker). Features - Bronchial blockers are guided into the appropriate bronchus with the aid of a bronchoscope (Fig. 30.14). Wire-guided blockers have a loop on the end through which the fiberscope is passed, facilitating guidance of the blocker into the bronchus after the fiberscope. The Cohen blocker has a steerable tip to facilitate advancement into the appropriate bronchus. The Univent tube consists of an integrated ETT with a second lumen for a deployable bronchial blocker. The EZ blocker has a forked distal end that functions like the Carlens hook to rest on the carina, while one arm of the fork has the inflatable balloon to occlude the bronchus it occupies.

This is the most effective treatment modality for symptomatic pulmonary alveolar proteinosis. A. Pulmonary Thromboendarterectomy B. VATS C. Bronchopulmonary lavage D. EBUS

Bronchopulmonary Lavage - Bronchopulmonary lavage (BPL) is a treatment during which up to 10 to 20 L of normal saline is instilled and then drained in 500- to 1000-mL aliquots through one side of a DLT into one lung under general anesthesia until the effluent is clear.241 It may then be performed on the contralateral lung during the same anesthetic or during a subsequent anesthetic after a period of several days for recovery. It is the most effective treatment modality for symptomatic pulmonary alveolar proteinosis. This lung disease results from accumulation in the alveoli of a lipoprotein material similar to surfactant.242 This disease seems to have an immune component, and some patients respond to conservative medical therapy with granulocute-macrophage colony-stimulating factor.243 Other pathologic states that have been treated by bronchopulmonary lavage include cystic fibrosis, asthma, radioactive dust inhalation, lipoid pneumonitis, and silicosis, all without convincing success. - General anesthesia is induced and maintained with intravenous infusions as for lung transplantation. Airway management is done with a left-sided DLT.244 The patient is kept in the supine position during the procedure. Because of transmitted hydrostatic pressure from the lavage lung to the pulmonary circulation, oxygenation increases during the filling phase and decreases during the emptying phase in synchrony with changes in the pulmonary blood flow distribution. These changes are usually transient and well tolerated. In severely hypoxemic patients presenting for BPL a variety of maneuvers have been used (such as inflation of a PA catheter balloon in the pulmonary artery (PA) of the lavage lung while adding NO to the ventilated lung245 or the use of ECMO) to maintain oxygenation. - Usually 10 to 15 L is instilled and more than 90% is recovered, leaving a deficit of less than 10%. At the end of the procedure, the lavaged lung is thoroughly suctioned. A dose of furosemide (10 mg) is administered to increase diuresis of absorbed saline. If the plan is to proceed to a lavage of the contralateral lung, there is a period of at least 1 hour of TLV to allow recovery of the lavaged lung, during which arterial blood gases are monitored. If there is a persistent large alveolar-arterial oxygen gradient, the procedure is terminated at this stage and the patient will undergo lavage of the other lung at a later date. The postprocedure, after reintubation with an SLT, is a fiberoptic bronchoscopic inspection performed for suctioning. Conventional ventilation with PEEP is continued, usually for less than 2 hours. Observation in the intensive care 1997unit for 24 hours is part of the routine procedure. Some patients require lavage every few months, whereas others remain in remission for years.

Males with height less than 170 cm have a DLT size of (Fr) A. 35 B. 37 C. 39 D. 41

C.

Which of the following is not a complication associated after thoracotomy A. Increased incidence of dysrhythmias B. Increased inflammation C. Increased cardiac output D. Increased loss of pulmonary vasculature

C. Complications After Thoracotomy - A number of complications may occur following thoracic surgery. Various factors have been correlated with an increased risk of complications. Admission to the intensive care unit or heightened surveillance should be considered for postoperative patients with the following characteristics: pulmonary fibrosis, age greater than 80 years, PPO FEV1 or DLCO less than 40%, ASA status over 3, surgical time longer than 80 minutes, intraoperative hemorrhage, and others.180-183 The following frequent predictors of pulmonary complications are quantified in the FLAM score (named after two of its designers Francesco Leo and Marylene Anziani), which shows good predictive capacity for impending respiratory complications: dyspnea, chest X-ray changes, required oxygen administration, auscultated changes, cough, and bronchial secretions.184 Among the most common postoperative complications are respiratory failure, cardiac dysrhythmia or failure, and acute lung injury (ALI). - Significant factors associated with ALI after pulmonary resections include right pneumonectomy, intraoperative overhydration with high vascular volume, high intraoperative airway pressure during OLV, and preoperative alcohol abuse. Other factors that have been suggested are female gender, poor postoperative predicted lung function, trauma, infection, chemotherapy, mediastinal lymphatic damage, transfusion and administration of fresh frozen plasma, oxygen toxicity, prolonged OLV (greater than 100 minutes), and an increased postoperative urine output.143,185 OLV causes inflammatory changes in the ventilated and nonventilated lung, and some of the damage appears to occur upon reexpansion of the deflated lung.186-190 - Protective ventilation strategies should always be employed to reduce the generation of inflammatory processes.136,191 There appears to be a lasting effect of OLV that is not reversed when reverting to two-lung ventilation. A small animal study demonstrated that after OLV has been discontinued, very low V/Q ratios in the range of 0.3-0.5 persist in the dependent lung.192 This, coupled with diffuse alveolar damage in comparison to controls, suggests that anesthetists should expect hypoxemia from the resultant V/Q mismatch that may persist for an unknown period after OLV. Even in the absence of an inflammatory response, there is evidence of vascular injury to lung tissue that is collapsed during OLV, which suggests that CPAP may have both short- and long-term benefits.193 - Minimizing pulmonary intravascular pressures by intraoperative fluid restriction is advocated to decrease postoperative complications.194 Surgical requirements for proper hydration and tissue perfusion must be balanced with the desire to prevent high postoperative intravascular pressures that can cause acute pulmonary edema. Decreased cardiac output in the early postoperative period can be caused by several factors, including blood loss, herniation of the heart through a pericardial defect, right-sided heart failure, and dysrhythmias. Generally, blood entering the pleural space drains into chest tubes at a rate of less than 500 mL per day. Chest tube drainage greater than 200 mL per hour necessitates surgical exploration. An obstructed chest tube can conceal bleeding in a hemothorax. Hypotension, unexplained tachycardia, and decreasing hematocrit are other signs associated with hemorrhage. - Loss of pulmonary vasculature with lung resection can result in increased PVR and right-sided heart failure. The reduction in cardiac ejection fraction is greater following pneumonectomy than that following lobectomy. Conditions that increase the likelihood of right-sided heart failure include postoperative pneumonia, hypercarbia, and acidosis. Vasodilators are useful for decreasing the PVR. Amrinone or dobutamine can be administered for inotropic support, as needed. - Supraventricular dysrhythmias are common after thoracotomy and may herald other serious complications.195 Morbidity and mortality rates in patients with supraventricular tachydysrhythmias are high, with 25% associated with death within 30 days postoperatively, despite institution of aggressive treatment. Administration of a β-blocking agent can help prevent atrial dysrhythmias. Metoprolol or esmolol provide rate control, and their cardioselectivity limits adverse effects on bronchial tone. Digitalis, adenosine, calcium channel blockers, and β-blockers are useful for treating supraventricular tachydysrhythmias. Respiratory complications in the early postoperative period include atelectasis, pneumonia, respiratory failure, bronchopleural or bronchocutaneous fistula, pneumothorax, and pulmonary edema. Aggressive respiratory care to prevent deterioration and allow weaning from ventilation is vital. - Nerve injuries that may follow thoracic surgery include damage to the phrenic nerve as it passes through the mediastinum, and damage to the left recurrent laryngeal nerve, which is vulnerable during dissection of aortopulmonary lymph nodes and mediastinal procedures.196 Spinal cord injury is a possibility if an intercostal artery supplying a major radicular artery is injured, or if an epidural hematoma is created by surgical dissection between the pleura and the epidural space. Nerve injuries related to surgical positioning are also possible complications.

This are products of abnormalities in the tracheobronchial budding process of lung organogenesis. They may occur peripherally within the lung parenchyma (70%) or centrally attached to the mediastinum or hilum. A. Bleb B. Bullae C. Cysts D. Pneumatocele

C. Cysts - Congenital bronchogenic cysts are products of abnormalities in the tracheobronchial budding process of lung organogenesis. They may occur peripherally within the lung parenchyma (70%) or centrally attached to the mediastinum or hilum. Bronchogenic cysts become problematic if they become enlarged, exerting a mass effect on functional lung or mediastinal structures; if they rupture and create a pneumothorax; or if they become infected. Small cysts without communication to a bronchus are asymptomatic and may be incidentally noted as round, clearly demarcated lesions on chest radiographs. Communicating cysts often produce air-fluid levels, are prone to recurrent infection, and may trap air by a ball-valve mechanism, risking rapid expansion or rupture. Infected cysts may be obscured by surrounding pneumonia, or they may be difficult to differentiate from an empyema. CT scans help differentiate cystic from solid lesions. Conservative surgical excision of bronchogenic cysts is generally recommended, regardless of whether a bronchial communication is evident. - Pulmonary hydatid cysts are watery, parasitic cysts containing larvae of the dog tapeworm Echinococcus granulosus.226 E. granulosus is a relatively common cause of pulmonary cysts in endemic areas (e.g., Australia, New Zealand, South America, and some third-world regions). Hydatid cysts may grow in diameter by as much as 5 cm per year and become medically problematic in several ways. By mass effect, they may exert pressure on adjacent structures (e.g., bronchus, great vessels, esophagus). Spontaneous or traumatic rupture may occur, sending fluid, parasites, or laminated debris into adjacent tissue, bronchus, pleura, or the circulation (i.e., systemic emboli). Hypersensitivity reactions, bronchospasm, and anaphylaxis can result. Drainage into the bronchi may cause dramatic expulsion of fluid with respiratory distress or asphyxiation, depending on the amount of fluid involved. Rupture into the pleural space may result in a large hydropneumothorax, severe dyspnea, shock, suffocation, or anaphylaxis. Rupture becomes more dangerous and more likely as cysts become larger. It is recommended that any cyst larger than 7 cm should be removed. - Small, intact peripheral cysts are often easily enucleated without loss of lung parenchyma. Segmentectomy or lobectomy is indicated when single or multiple cysts occupy most of the segment or lobe. Patients with suppurative cysts should be prepared for surgery with postural drainage and antibiotics. Lung isolation and/or reduced airway pressure during dissection may be helpful in preventing herniation of the cyst. Increased airway pressure at the time of delivery may aid in removal of the cyst. The multiple bronchial openings in the residual cavity must then be identified and closed. Multiple "leak tests" with saline poured into the residual opening may be required to locate all bronchial openings. An alternative surgical strategy is to inject hypertonic saline into the cyst to sterilize it, followed by aspiration of the contents and removal of the evacuated cyst.

For laryngoscopy, the lubricated DLT is advanced with the distal curve concave anteriorly until the vocal cords are passed. The stylet is usually removed at this point to reduce the potential of the rigid tube causing mucosal damage. The tube is then rotated ___ degrees toward the bronchus that is to be intubated. A. 30 B. 60 C. 90 D. 180

C. Insertion of Double-Lumen Endotracheal Tubes - The DLT has two curves along its length to aid in its placement. A stylet aids placement through the larynx. Some practitioners prefer the Macintosh blade for intubation because it offers greater clearance for the tube and may decrease the chance of balloon rupture from the teeth.100 For laryngoscopy, the lubricated DLT is advanced with the distal curve concave anteriorly until the vocal cords are passed. The stylet is usually removed at this point to reduce the potential of the rigid tube causing mucosal damage. The tube is then rotated 90 degrees toward the bronchus that is to be intubated. The DLT is advanced to approximately a 27-cm depth in females or a 29-cm depth in males, or until resistance is met.101 - The tracheal cuff requires 5 mL to 10 mL of air, and the bronchial cuff requires 1 mL to 2 mL of air. Overinflation of the bronchial cuff can cause its lumen to become narrowed or occluded, and increases the risk of bronchial damage. Unlike most tracheal high-volume, low-pressure cuffs, the bronchial cuff holds a small volume and can exert high pressure on the endobronchial mucosa. For that reason, unless unilateral lung contamination exists, the bronchial cuff should be deflated during the procedure once OLV is no longer needed. After the tube is situated in the bronchus, adapters are attached to the two lumens for interface with the anesthesia circuit. Auscultation of breath sounds is a simple, though not highly reliable, method of determining the position of a DLT (Box 30.4). When properly positioned, breath sounds should be auscultated in all fields of the lung corresponding to the bronchial lumen (depending on the use of a left- or right-sided tube) when that lumen alone is ventilated. Breath sounds should be heard only in the opposite lung, when the tracheal lumen is ventilated. Fig. 30.11 outlines some auscultation findings expected with various tube positions. - Flexible fiberoptic bronchoscopy is essential to verify placement of the DLT (see Fig. 30.11 and Box 30.5). Fiberoptic bronchoscopy has revealed a 38% to 83% incidence of malpositioning of DLTs that were judged by auscultation to be properly placed.91,102 Some particular advantages of fiberoptic inspection of the DLT over auscultation are guidance during initial placement, the ability to visualize the correct depth of the bronchial cuff, and visualization of proper positioning of the right upper lobe port (if present). Placement of the tube should again be verified by bronchoscopy after the patient is positioned laterally because the DLT will commonly withdraw from the bronchus by 1 cm.103 When separation of the lungs is required to prevent cross-contamination, the integrity of the bronchial seal can be tested by connecting a tube from the bronchial port to a water seal and then providing ventilation through the tracheal lumen. Incompetence of the bronchial cuff will be evidenced by egress of bubbles in the water seal (Fig. 30.12). Auscultation of Breath Sounds After Placement of a Double Lumen Tube 1. Inflate the tracheal cuff. 2. Verify bilaterally equal breath sounds. If breath sounds are present on only one side, both lumens are in the same bronchus. Deflate the cuff and withdraw the tube 1-2 cm at a time until breath sounds are equal bilaterally. 3. Inflate the endobronchial cuff. 4. Clamp Y-piece to the endobronchial lumen and open the lumen to atmosphere. 5. Verify breath sounds in the correct lung (tracheal side) and the absence of breath sounds in the opposite lung (bronchial side). 6. Verify the absence of air leakage through the bronchial lumen.* 7. Unclamp and reconnect the endobronchial lumen and verify bilateral breath sounds. 8. Clamp Y-piece to the tracheal lumen and open the lumen to atmosphere. 9. Verify breath sounds in the correct lung (bronchial side) and the absence of breath sounds in the opposite lung (tracheal side). 10. Verify that breath sounds are equal at the apex of the lung and at the base. If the apex is diminished, withdraw the tube until upper lung sounds return. 11. Verify the absence of air leakage through the tracheal lumen.*

A rare cause of hypoxemia associated with thoracic surgery is reversal of shunt flow through an undiagnosed A. Ductus arteriosus B. PDA C. Foramen Ovale D. Cronoary sinus

C. Invasive Hemodynamic Monitoring (See Chapter 45) Arterial Line - Transient severe hypotension from surgical compression of the heart or great vessels can occur during intrathoracic procedures. For this reason, in addition to the utility of intermittent arterial blood gas sampling, it is useful to have beat-to-beat assessment of systemic blood pressure during the majority of thoracic surgery cases. Naturally, exceptions occur during limited procedures, such as thoracoscopic resections in younger and healthier patients. For most thoracotomies, placement of a radial artery catheter can be in either the dependent or nondependent arm. Central Venous Pressures - It is a common impression that central venous pressure (CVP) readings in the lateral position with the chest open are not reliable as a monitor of volume status. The CVP may be a useful monitor postoperatively, particularly for cases in which fluid management is critical (e.g., pneumonectomies). A CVP may be required in some cases for vascular access or for vasopressor/inotrope infusions. It is our practice to routinely place CVP lines in pneumonectomy cases, complex procedures, or redo thoracotomies, but not for lesser resections unless there is significant other concurrent illness. Our choice is to use the right internal jugular vein to minimize the risk of pneumothorax for CVP access unless there is a contraindication. Internal jugular CVP data are not reliable in patients with superior vena cava obstruction. Pulmonary Artery Catheters - Similar to CVP data, intraoperative pulmonary artery pressure may be a less accurate indicator of true left-heart preload in the lateral position with the chest open than in other clinical situations. This is partly because it is often initially not known whether the catheter tip lies in the dependent or nondependent lung. In addition, it is possible that thermodilution cardiac output data may be unreliable if there are significant transient unilateral differences in perfusion between the lungs, as can occur during OLV. There is no consensus on the reliability of thermodilution cardiac output data during OLV.66 Fiberoptic Bronchoscopy - Placement of DLTs and bronchial blockers is discussed later in the section "Lung Isolation." Significant malpositions of DLTs and blockers that can lead to desaturation during OLV are often not detected by auscultation or other traditional methods of confirming placement.67 Placement of DLTs or blockers should be performed with fiberoptic bronchoscopic guidance and should be reconfirmed after placing the patient in the surgical position, because a large number of these tubes/blockers migrate during repositioning of the patient.68 Continuous Spirometry - The development of side-stream spirometry has made it possible to continuously monitor inspiratory and expiratory volumes, pressures, and flow interactions during one-lung anesthesia. This monitoring is particularly useful during pulmonary resection surgery. The breath-by-breath monitoring of inspired and expired tidal volumes gives early warning of accidental changes in the intraoperative position of a DLT, with loss of lung isolation if the expired volume suddenly decreases (there is normally a 20- to 30-mL/breath difference caused in part by the uptake of inspired oxygen). The development of a persistent end-expiratory flow during OLV, which correlates with the development of auto-PEEP, can be seen on the flow-volume loop.69 Also the ability to accurately measure differences in inspiratory and expiratory tidal volumes is extremely useful in assessing and managing air leaks during and after pulmonary resections. Transesophageal Echocardiography - Transesophageal echocardiography (TEE) allows the anesthesiologist to view a continuous real-time monitor of myocardial function and cardiac preload (see Chapter 46). This information is difficult to estimate intraoperatively in the lateral position from other hemodynamic monitors.70 Potential indications for intraoperative TEE that apply to thoracic surgery include hemodynamic instability (Fig. 66-8), pericardial effusions, cardiac involvement by tumor, air emboli, pulmonary thromboendarterectomy, thoracic trauma, lung transplantation, and pleuropulmonary disease. A rare cause of hypoxemia associated with thoracic surgery is reversal of shunt flow through an undiagnosed patient foramen ovale. When PEEP (to 15 cm H2O) was applied during controlled ventilation for nonthoracic surgery, 9% of the patients developed a right-to-left intracardiac shunt.71 TEE should be capable of detecting situations in patients who might intraoperatively develop a right-to-left interatrial shunt during or after thoracic surgery. Other New Monitoring Technology - Cerebral oximetry (SctO2) has been reported for intraoperative monitoring during OLV.72 Older, debilitated patients are more likely to have falls in SctO2 during OLV, and these are associated with decreases in SpO2 and postoperative cognitive dysfunction. However, it has not been shown whether any treatment for decreases in SctO2 affects outcomes. Goal-directed fluid therapy using indirect monitors of cardiac output or venous oxygen saturation seems to improve fluid management in abdominal surgery.73 This is currently a much needed area of research in thoracic anesthesia.

Carbon dioxide (CO2) retention with an arterial partial pressure (PaCO2) greater than __ mm Hg is an indicator of poor ventilatory function A. 25 B. 35 C. 45 D. 55

C. Laboratory Assessment - Measurement of preoperative room air arterial blood gases should be considered for patients with COPD, and it is useful in guiding postoperative ventilation. Carbon dioxide (CO2) retention with an arterial partial pressure (PaCO2) greater than 45 mm Hg is an indicator of poor ventilatory function. However, hypercapnia is not a reliable predictor of increased risk of perioperative pulmonary complications.40 Preoperative hypoxemia (blood oxygen saturation [SpO2] < 90%) and, particularly, desaturation during exercise, may be predictive of increased complications following thoracic surgery.41,42 However, in general, blood gas analysis is not a reliable tool for predicting postoperative pulmonary complications,43 and the correlation between desaturation during exercise and postoperative complications is not a consistent finding.44 - Hypoalbuminemia is the most common laboratory finding, and serves as an important predictor of pulmonary complications. Numerous studies have demonstrated increases in postoperative pulmonary complications among patients with low serum albumin levels (generally < 3.6 g/dL).45-48 This factor increases risk as much as 2.5 times,47 and albumin level maintenance is a measured factor in the American College of Surgeons' National Surgical Quality Improvement Program (NSQIP).49 The blood urea nitrogen level is also identified by NSQIP data as a predictive factor for pulmonary complications when it is greater than 22 mg/dL.47,48 - Other laboratory analyses should include renal function indicators (particularly for patients treated with nephrotoxic drugs such as methotrexate, gemcitabine, and cisplatin),50,51 sodium (related to syndrome of inappropriate antidiuretic hormone secretion),23 and calcium (due to parathyroid hormone-like protein).21,52 Echocardiogram - Echocardiographic findings that are consistent with pulmonary disease are increased thickness of the right ventricular free wall, chamber enlargement, septal shift, tricuspic regurgitation, and a decreased right ventricular ejection fraction. In response to chronic hypoxia, hypoxic pulmonary vasoconstriction causes elevated pulmonary artery pressures. This results in increased right ventricular afterload, which can lead to right ventricular dysfunction. Increased PVR and right ventricular strain cause concern in patients undergoing pneumonectomy or extensive partial resection because of the added resistance produced by clamping the vasculature of one lung in the surgical procedure. Echocardiography is the best initial tool for assessing pulmonary hypertension, but additional studies with pulmonary angiography, ventilation-perfusion scintigraphy, computed tomography (CT), and magnetic resonance imaging (MRI) may also be used for more in-depth evaluation.39

This is a leading cause of postoperative morbidity and mortality in patients undergoing major lung resection. A. Hemorrhage B. Infection C. Respiratory failure D. Cadiac herniation

C. Postoperative Management Early Major Complications - There are multiple potential major complications that can occur in the immediate postoperative period after thoracic surgery, such as torsion of a remaining lobe after lobectomy, dehiscence of a bronchial stump, or hemorrhage from a major vessel. Fortunately, these are infrequent and when they do occur, the principles of management are as outlined earlier for common and specific procedures. Among these possible complications, two will be discussed in more detail: (1) respiratory failure, because it is the most common cause of major morbidity after thoracic surgery; and (2) cardiac herniation, because, although it is rare, it is usually fatal if it is not quickly diagnosed and appropriately treated. Respiratory Failure - Respiratory failure is a leading cause of postoperative morbidity and mortality in patients undergoing major lung resection. Acute respiratory failure after lung resection is defined as the acute onset of hypoxemia (PaO2 <60 mm Hg) or hypercapnia (PaCO2 >45 mm Hg), or the use of postoperative mechanical ventilation for more than 24 hours or reintubation for controlled ventilation after extubation. The incidence of respiratory failure after lung resection is between 2% and 18%. Patients with decreased respiratory function preoperatively are at increased risk of postoperative respiratory complications. In addition, age, the presence of coronary artery disease, and the extent of lung resection play major roles in predicting postoperative morbidity and mortality. Crossover contamination, because of the failure of lung isolation intraoperatively during lung resection in patients with contaminated secretions, can result in contralateral pneumonia and postoperative respiratory failure.263 Mechanical ventilation during the postoperative period after lung resection is associated with the risk of acquired nosocomial pneumonia and bronchopleural fistula. - Decreased pulmonary complications in high-risk patients are associated with the use of thoracic epidural analgesia during the perioperative period.2 The prevention of atelectasis and secondary infections have been attributed to better preservation of the functional residual volume, efficient mucociliary clearance, and alleviation of the inhibiting reflexes acting on the diaphragm in patients receiving epidural analgesia.264 Chest physiotherapy, incentive spirometry, and early ambulation are crucial to minimize pulmonary complications after lung resection. For an uncomplicated lung resection, early extubation is desirable to avoid potential complications that can arise as a result of prolonged intubation and mechanical ventilation. Current therapy to treat acute respiratory failure is supportive therapy attempting to provide better oxygenation, treat infection, and provide vital organ support without further damaging the lungs. Cardiac Herniation - Acute cardiac herniation is an infrequent but well described complication of pneumonectomy, where the pericardium is incompletely closed or the closure breaks down.265 It usually occurs immediately or within 24 hours after chest surgery and is associated with a more than 50% mortality rate. Cardiac herniation may also occur after a lobar resection with pericardial opening, or in other chest tumor resections involving the pericardium or in trauma.266 When cardiac herniation occurs after a right pneumonectomy, the clinical presentation is caused by impairment of the venous return to the heart with a concomitant increase in CVP, tachycardia, profound hypotension, and shock. An acute superior vena cava syndrome ensues because of the torsion of the heart.267 In contrast, when the cardiac herniation occurs after a left-sided pneumonectomy, there is less cardiac rotation, but the edge of the pericardium compresses the myocardium. 2000This may lead to myocardial ischemia, the development of arrhythmias, and ventricular outflow tract obstruction. Cardiac herniation occurs after chest closure because of the pressure difference between the two hemithoraces. This pressure difference may result in the heart being extruded through a pericardial defect. Management for a patient with a cardiac herniation should be considered as dire emergent surgery. The differential diagnosis should include massive intrathoracic hemorrhage, pulmonary embolism, or mediastinal shift from improper chest drain management. Early diagnosis and immediate surgical treatment by relocation of the heart to its anatomic position with repair of the pericardial defect or by the use of analogous or prosthetic patch material is key to patient survival. Because these patients have undergone a previous thoracotomy, all precautions should be taken for a "redo" exploration. This includes the use of large-bore intravenous catheters and an arterial line. Maneuvers to minimize the cardiovascular effects include positioning the patient in the full lateral position with the operated side up. Because time is crucial, an SLT is used. Vasopressors or inotropes, or both, are required to support the circulation while exploration takes place. The use of TEE intraoperatively and after resuscitation and relocation of the heart can be considered during pericardial patch repair to prevent excessive compression of the heart by the repair.268 In general, patients undergoing an emergency thoracic reexploration remain intubated and are transferred to the intensive care unit postoperatively.

Which of the following is not one of the 4 M's of anesthetic considerations in lung cancer patients A. Mass effect B. metabolic effects C. Metamorphosis D. Medications

C. Preoperative Evaluation of Patients for Pulmonary Surgery 1. Evaluate comorbidities: • Smoking-related complications • Cor pulmonale • Effects of paraneoplasms • Cardiovascular disease (unstable angina, MI within 6 weeks, or significant dysrhythmias = high risk of cardiac complications) • Treatment side effects (particularly from cytotoxic drugs and radiation) 2. ECG and chest x-ray for signs of cardiovascular dysfunction and effects of lung pathology 3. Laboratory assessment of electrolytes, blood count, albumin, and renal function indicators 4. Lung function testing: 80-40-15 rule: • FEV1 and DLCO > 80% predicted = no additional testing needed. If < 80 or dyspnea present, diffusing capacity and postoperative function should be predicted • PPO FEV1 and DLCO < 40% predicted = increased risk; exercise testing should be evaluated • V̇O2 max < 15 mL/kg per min = increased risk. 5. For COPD, consider blood gas and response to bronchodilators

PPO FEV1 of this suggest high risk patients A. 10 B. 30 C. 50 D. 70

C. Pulmonary Function Tests - Patients presenting for lung resection should undergo pulmonary function testing to assess for airflow limitation, diffusion defect, and cardiopulmonary reserve.53 Assessment should include the response to bronchodilators for patients who demonstrate obstructive disease (see Fig. 29.18). The American Thoracic Society considers a 12% improvement in forced expiratory volume 1 (FEV1) post-bronchodilator therapy to be significant. Whereas this parameter has a low sensitivity to diagnose asthma,54 the relative responsiveness of the patient to bronchodilators can suggest the utility of using bronchodilators for intraoperative management. Pulmonary assessment by spirometry should be based upon values obtained post-bronchodilator therapy, as these would represent the patient's potential lung function once optimized on medications. No single pulmonary function measurement provides an overall risk assessment. For example, although the FEV1 is the most prevalent spirometric measurement, one case series of 100 thoracic surgery patients with very low FEV1 values (less than 35%) demonstrated a low rate of mortality and ventilator dependence (although the patients did show a prolonged duration of hospitalization and air leak).55 In another series of 109 elderly patients, stair-climbing ability was better correlated (inverse relationship) with postoperative cardiopulmonary complications than was forced vital capacity (FVC) or predicted postoperative (PPO) FEV1.56 Therefore, a multimodal approach must be taken, considering airflow (PPO FEV1), parenchymal function (diffusing capacity of the lungs for carbon monoxide, DLCO), and cardiopulmonary reserve (maximum volume of oxygen [V̇O2 max]). The general cutoff points indicating increased risk among these parameters is below 40% for PPO FEV1 and DLCO, and 15 mL/kg per min for V̇O2 max. However, the surgical approach must also be considered as part of the risk evaluation. A large study involving more than 13,000 subjects validated that PPO FEV1 and PPO DLCO estimates were inversely proportional to the rate of complications and mortality. However, the absolute rate of complications and mortality was significantly lower in patients undergoing thoracoscopic, rather than open, lobectomy, even when the PPO FEV1 and PPO DLCO were less than 40%.57 - It should be noted that guidelines used to assess surgical candidacy are not intended to dictate candidacy for anesthesia. If the patient is a candidate for surgery, it is less likely that there will be anesthetic-specific concerns about pulmonary function that would override the surgical decision, particularly as many of these surgeries are performed to treat cancer. Notwithstanding this, it is helpful for the anesthetist to understand the patient's risk stratification to plan for the level of ventilatory support required during and after surgery. Assessment must consider multiple functional variables. Similar to standards set by the American Thoracic Society and the European Respiratory Society, the ACCP proposes the following assessment of risk factors for patients with lung cancer undergoing lung surgery: preoperative FEV1 greater than 80% of the predicted value (or > 2 L for pneumonectomy or > 1.5 L for lobectomy) indicates average risk, and no further assessment of lung function is required. The DLCO should be assessed if diffuse parenchymal disease or dyspnea on exertion is noted. If the FEV1 or DLCO is less than 80% of the predicted value, then the predicted postoperative FEV1 and DLCO are assessed.40 This is accomplished either through radionucleotide scanning, or mathematically (based on the proportion of total lung that will remain after the planned resection). The PPO FEV1 can be calculated by multiplying the current FEV1 by the fraction of functioning lung or the fraction of lung segments that will remain after surgery.15 For high-risk patients, more detailed assessment via radionuclide scanning, CT scanning, or MRI is advisable. Postoperative PPO FEV1 values greater than 40% of the predicted value for the patient indicate average risk. Values less than 30% of the predicted value indicate increased risk, and intermediate values warrant exercise testing to assess oxygen consumption (V̇O2 max) A V̇O2 max of less than 15 mL/kg per min indicates high risk, whereas a value greater than 15 mL/kg per min indicates average risk.40 The European Respiratory Society places V̇O2 max more prominently in the assessment, and considers high-risk cutoffs as being less than 30% for PPO FEV1 and DLCO, and less than 10 mL/kg per min for V̇O2 max.53 Regarding anesthetic planning, average-risk patients (e.g., PPO FEV1 > 40%) are likely to be extubated immediately following surgery. High-risk patients (e.g., PPO FEV1 < 30%) have a higher likelihood of requiring some degree of postoperative ventilation. Planning for intermediate-risk patients (e.g., PPO FEV1 30% to 40%) is further individualized based upon other assessment parameters.

Which of the following is not a Recommendations for Avoiding Hypoxemia and Acute Lung Injury During One-Lung Ventilation A. FiO2 < 1.0 B. Low TV C. Routine use of PEEP D. Routine use of CPAP to dependent lung

D.

Which of the following is not an INTRAOPERATIVE COMPLICATIONS THAT OCCUR WITH INCREASED FREQUENCY DURING THORACOTOMY A. Hypoxemia B. Arrhytmias C. Bronchospasm D. Hyperthermia

D.

This can be done as a last resort to stop all significant flow through the lung contributing to the shunt. A. CPAP B. PEEP C. Intermittent reinflation D. Ligation of the pulmonary artery

D. - If CPAP and PEEP fail, early ligation of the pulmonary artery in pneumonectomy patients may be used to improve oxygenation. If the pulmonary artery is planned to be ligated during the procedure, clamping it will immediately stop all significant flow through the lung contributing to the shunt. For the same reason, manual compression of the lung will also improve the PaO2, but the expense of cardiac output and tissue trauma advises against this as a regular strategy.160 If it becomes impossible to maintain adequate oxygenation with OLV in spite of CPAP and PEEP, manual two-lung ventilation can be used, with pauses in ventilation coordinated with the surgeon's activities to facilitate exposure, suturing of the lung, or other needs. Communication with the surgical team is vital throughout the procedure, especially during the evaluation and correction of hypoxia. - At the conclusion of the resection, the surgeon will commonly ask that the operative lung be reinflated using large VTs so that air leaks may be detected. At this time, the lung separator (DLT clamp or bronchial blocker) should be discontinued, and the lung inflated with slow breaths, achieving a peak inspiratory pressure of 30-40 cm H2O.89 Reexpansion of the lung can be observed while performing this maneuver, which also helps to reverse atelectasis in the lungs. Following lung reexpansion, the bronchial cuff should be deflated on the DLT to both reduce pressure on the bronchial mucosa and obviate any detrimental effects of slight tube malpositioning. Deflated, the cuff does not pose the threat of herniating over the carina or obstructing the lobar bronchi. Box 30.8 outlines the management of one-lung anesthesia. - Emerging techniques of OLV focus attention away from manipulating ventilation of the operative lung to manipulation of perfusion of the nonoperative lung. V/Q matching can be supported by encouraging more perfusion to the ventilated lung or selectively diminishing perfusion to the nonventilated lung. Inhaled epoprostenol (prostacyclin) and N2O are beneficial for increasing the perfusion of the dependent lung. The combination of inhaled epoprostenol and IV phenylephrine (for vasoconstriction of the operative lung) is an effective strategy.161 Another experimental approach is the use of almitrine, which enhances HPV of the nonventilated lung. Studies have shown a more than 100% increase in Pao2 when almitrine and N2O are used on their respective lungs162,163; however, toxicity, cost, and challenges related to setting up and administering N2O are deterrents to using these interventions as part of a routine plan, except for the most critical cases.

Which of the following is not an Anesthetic Management of Mediastinoscopy Hemorrhage A. Stop surgery and pack the wound B. Begin the resuscitation and call for help C. Place a double-lumen tube or bronchial blocker D. Reexplore the cervical incision immediately

D. Anesthetic Management of Mediastinoscopy Hemorrhage 1. Stop surgery and pack the wound. There is a serious risk that the patient will approach the point of hemodynamic collapse if the surgery-anesthesia team does not realize soon enough that there is a problem. 2. Begin the resuscitation and call for help, both anesthetic and surgical. 3. Obtain large-bore vascular access in the lower limbs. 4. Place an arterial line (if not placed at induction). 5. Prepare for massive hemorrhage with blood warmers and rapid infusers. 6. Obtain cross-matched blood in the operating room. 7. Place a double-lumen tube or bronchial blocker if the surgeon believes that thoracotomy is a possibility. 8. Once the patient is stabilized and all preparations are made, the surgeon can reexplore the cervical incision. 9. Convert to sternotomy or thoracotomy if indicated.

This uses a radial probe through a working channel of the flexible fiberoptic bronchoscope can be used to identify mediastinal and hilar lymph nodes A. Fiberoptic bronchoscopy B. Rigid bronchoscopy C. Mediastinoscopy D. EBUS

D. Endobronchial Ultrasound-Guided Biopsy - Various alternative techniques are available for obtaining pathology specimens from the mediastinal lymph nodes. These include CT-guided percutaneous needle aspiration, conventional bronchoscopy with transbronchial needle aspiration, and endobronchial ultrasound-guided biopsy. Endobronchial ultrasonography (EBUS) using a radial probe through a working channel of the flexible fiberoptic bronchoscope can be used to identify mediastinal and hilar lymph nodes.171 Under direct EBUS 1980guidance with fine-needle aspiration for mediastinal staging, an ultrasound puncture bronchoscope can be used to assist with the safe and accurate diagnostic interventional bronchoscopy of the mediastinal and hilar lymph nodes. Management of these patients is often done at a satellite location, either in the bronchoscopy facility or a CT suite. In general, these patients are managed with topical anesthesia (aerosolized lidocaine) and conscious sedation (e.g., fentanyl and/or midazolam). EBUS may replace mediastinoscopy as the standard method of staging lung cancer before resection.

These can be an effective adjunct to methods of postthoracotomy analgesia. These can be done percutaneously or under direct vision when the chest is open. A. Intrapleural B. Epidural C. Paravertebral D. Intercostal

D. Local anesthetics/Nerve Blocks Intercostal Nerve Blocks - Regional blocks of the intercostal nerves supplying the dermatomes of the surgical incision can be an effective adjunct to methods of postthoracotomy analgesia. These can be done percutaneously or under direct vision when the chest is open. The duration of analgesia is limited to the duration of action of the local anesthetic used, and the blocks will need to be repeated to have any useful effect on postoperative lung function. Indwelling intercostal catheters are an option but they can be difficult to position reliably percutaneously. Nerve blocks are useful supplements for the pain associated with the multiple small-port incisions and chest drains after VATS. It is important to avoid injection into the intercostal vessels, which are adjacent to the nerve in the intercostal groove. It is also important to place the block near the posterior axillary line to be certain to block the lateral cutaneous branch of the intercostal nerve. Total bupivacaine dose for a single session of blocks should not exceed 1 mg/kg (e.g., for a 75-kg patient: 3 mL bupivacaine 0.5% with epinephrine 1:200,000 at each of 5 levels). Intrapleural Analgesia - Intrapleural local anesthetics produce a multilevel intercostal block. The analgesia is extremely dependent on patient position, infusion volume, chest drains, and the type of surgery. Despite occasional successes, most clinicians have not found the reliability of intrapleural techniques adequate to justify their use on a routine basis.275 Epidural Analgesia - The routine use of neuraxial analgesia for postthoracotomy patients is a relatively recent concept. Spinal injection of opioids can have a duration of analgesia that approaches 24 hours after thoracotomy. Because of the concerns of possible infection with subarachnoid catheters and the need to repeat spinal injections, investigation and therapy have focused on epidural techniques. A meta-analysis of respiratory complications after various types of surgery has shown that epidural techniques reduce the incidence of respiratory complications.276 Lumbar epidural analgesia has been replaced by thoracic epidural analgesia for thoracic surgery, with infusions of local anesthetic and opioid. These combinations provide better analgesia at lower doses than either drug will alone.277 The use of epidural infusions has an excellent record for patient safety when used on routine postoperative surgical wards.278 The use of a paramedian approach to the epidural space in the midthoracic levels (Fig. 66-53) has improved the success rate for many clinicians. Ultrasonographic guidance has not yet proved to be as useful for thoracic epidural catheter placement as it has for other types of regional blockade.279 - Research has shed light on the pharmacology that underlies the synergy between local anesthetics and opioids to produce segmental epidural analgesia. In a double-blind randomized study, Hansdottir and associates280 compared epidural infusions of lumbar sufentanil, thoracic sufentanil, and thoracic sufentanil plus bupivacaine for postthoracotomy analgesia. Infusions were titrated for equianalgesia at rest. Thoracic sufentanil plus bupivacaine provided significantly better analgesia with movement and less sedation than the other infusions. Although sufentanil dosages and serum levels were significantly lower 2002in the combined sufentanil plus bupivacaine group than in the other two groups, lumbar cerebrospinal fluid levels of sufentanil at 24 and 48 hours were higher in the combined group than in the thoracic sufentanil group. This suggests that the local anesthetic facilitates entry of the opioid from the epidural space into the cerebrospinal fluid. - In patients with severe emphysema, thoracic epidural analgesic doses of bupivacaine do not cause any significant reduction in lung mechanics or increase in airway resistance.281 In volunteers, a thoracic level of epidural blockade increases FRC.282 This increase is largely caused by an increase in thoracic gas volume caused by a fall in the resting level of the diaphragm without a fall in tidal volume. Differences in lipid solubility that create relatively minor clinical differences in the effects of opioids when used systemically cause major differences in the effects of these same opioids when used neuraxially. The highly lipid-soluble agents (e.g., fentanyl, sufentanil) are associated with narrow dermatomal spread, rapid onset, and low incidence of pruritis/nausea, and can be potentiated by epinephrine. However, these lipid-soluble agents have significant absorption and systemic effects when used as epidural infusions.283 For incisions that cover many dermatomes (e.g., sternotomy) or for procedures that have combined abdominal and thoracic incisions (e.g., esophagectomy), the hydrophilic opioids (e.g., morphine, hydromorphone) are preferable.

These are thin-walled, air-filled spaces generated by pulmonary infections or trauma. A. Bleb B. Bullae C. Cysts D. Pneumatocele

D. Pneumatocele - Pneumatoceles are thin-walled, air-filled spaces generated by pulmonary infections or trauma. They usually appear in the first week of pneumonia and resolve spontaneously within 6 weeks. As with other lung cysts, potential complications of pneumatoceles include secondary infection and enlargement as a result of air entrapment, with possible rupture or displacement and compression of normal lung. Adverse hemodynamic consequences may result either from a tension pneumothorax or a tension pneumatocele. The latter is unusual and is presumed to result from a one-way valve mechanism, usually in the setting of positive-pressure mechanical ventilation.227 Sometimes surgical decompression is required and is performed by percutaneous needle aspiration, catheter drainage, or chest tube drainage under CT or fluoroscopic guidance. Rarely is thoracoscopic or open surgical drainage or excision required.

Which of the following is not an indication for Video-Assisted Thoracoscopic Surgery A. Biopsy and staging of cancer B. Identification of pericardial disease C. Assessment of injury from trauma D. Resection of mediastinal tumor(s)

None of the above

In lung cancer, 75% to 80% of these tumors diagnosed A. Small-cell lung cancer (SCLC) B. Non-small-cell lung cancer (NSCLC) C. Carcinoid tumor D. Adenoid cystic carcinoma

B. Primary Thoracic Tumors - The majority of patients presenting for major pulmonary surgery will have some type of malignancy. Because the different types of thoracic malignancies have varying implications for both surgery and anesthesia, it is important for the anesthesiologist to have some knowledge of the presentation and biology of these cancers. By far the most common tumor is lung cancer. There are more than 200,000 new cases of lung cancer diagnosed per year in North America and more than 1.2 million diagnosed in the world. Lung cancer is currently the leading cause of cancer deaths in both genders in North America subsequent to the peak incidence of smoking in the period 1940 to 1970.48 Worldwide, lung cancer incidence is more than double in men than in women (rate ratio 2.5:1.0).49 - Lung cancer is broadly divided into small-cell lung cancer (SCLC) and non-small-cell lung cancer (NSCLC), with approximately 75% to 80% of these tumors diagnosed as NSCLC. Other less common and less aggressive tumors of the lung include the carcinoid tumors (typical and atypical) and adenoid cystic carcinoma. Compared with lung cancer, primary pleural tumors are rare. They include the localized fibrous tumors of pleura (previously referred to as benign mesotheliomas) and malignant pleural mesothelioma. Asbestos exposure is implicated in as many as 80% of malignant pleural mesotheliomas. A dose-response relationship is not always apparent, and even brief exposures can lead to the disease. An exposure history is often difficult to obtain because the latent period before clinical manifestation of the tumor may be as long as 40 to 50 years. 1950 - Tobacco smoke is responsible for approximately 90% of all lung cancers, and the epidemiology of lung cancer follows the epidemiology of cigarette smoking with approximately a 3-decade lag time.50 Other environmental causes include asbestos and radon gas (a decay product of naturally occurring uranium), which act as co-carcinogens with tobacco smoke. For a pack-a-day cigarette smoker, the lifetime risk of lung cancer is approximately 1 in 14. Assuming that current mortality patterns continue, cancer will pass heart disease as the leading cause of death in North America in this decade. Non-Small-Cell Lung Cancer - This pathologically heterogeneous group of tumors includes squamous cell, adenocarcinoma, and large-cell carcinoma. Overall 5-year survival with surgery approaches 40%. This seemingly low figure must be viewed in the light of an estimated 5-year survival without surgery of less than 10%. Although it is not always possible to be certain of the pathology of a given lung tumor preoperatively, many patients will have a known tissue diagnosis at the time of preanesthetic assessment based on prior cytology, bronchoscopy, mediastinoscopy, or transthoracic needle aspiration. This is useful information for the anesthesiologist to obtain preoperatively. Specific anesthetic implications of the different types of lung cancer are listed in Table 66-2. Squamous Cell Carcinoma - This subgroup of NSCLC is strongly linked to cigarette smoking. The tumors tend to grow to a large size and metastasize later than others. They tend to cause symptoms related to local effects of a large tumor mass with a dominant endobronchial component, such as cavitation, hemoptysis, obstructive pneumonia, superior vena cava syndrome, and involvement of mainstem bronchus, trachea, carina, and main pulmonary arteries. Hypocalcemia may be associated with this cell type because of elaboration of a parathyroid-like factor and not as a result of bone metastases.

Which of the following lung cancer causes hypercalcemia A. Squamous cell B. Adenocarcinoma C. Large cell D. Small cell E. Carcinoid

A.

Which pulmonary resection approach provides excellent exposure to entire operative hemithorax A. Posterolateal thoractomy B. lateral muscle sparing thoractotomy C. Axillary thoracotoym D. Sternotomy

A.

In an Anesthetized Lateral Position, Chest Closed, With Spontaneous Ventilation Ventilation is preferentially distributed in the A. Independent B. Dependent

A. - Anesthetized Lateral Position, Chest Closed, With Spontaneous Ventilation - A change in the distribution of ventilation is seen with the induction of anesthesia, even when spontaneous respiration is maintained. Functional residual capacity (FRC) decreases almost immediately upon the induction of anesthesia. The weight of the mediastinum and the cephalad displacement of the diaphragm by abdominal contents further decrease FRC in the dependent lung and reduce the proportion of the favorable zone 3 area. Lower volumes in each lung shift their place on the compliance curve. The lungs are less compliant when they are either at a very high volume (distended alveoli) or a very low volume (atelectasis). In the anesthetized patient, the nondependent lung moves from a flat, noncompliant portion of the compliance curve to a more compliant position. Although anesthesia results in a net loss of FRC, the relative proportion of FRC in the nondependent lung increases in contrast to the dependent lung.89 As the dependent lung loses FRC, its volume becomes so low as to decrease its compliance. It shifts to a less compliant, flatter portion of the curve (Fig. 30.6). Ventilation is therefore preferentially distributed to the nondependent lung, whereas gravity-dependent blood flow preferentially goes to the dependent lung, resulting in a mismatch of ventilation and perfusion.

Superior vena cava (SVC) syndrome can manifest with the following except A. dilation of ipsilateral veins of the upper part of the thorax and neck B. edema and rubor of the face, neck, upper torso, and airway C. edema of the conjunctiva with or without proptosis D. shortness of breath

A. - Biopsy of masses should be performed with the patient under local anesthesia whenever possible. Bilevel positive airway pressure (BiPAP) has been used in this situation to support the airway and maintain spontaneous ventilation while still providing sedation.170,171 Radiation therapy may be helpful to decrease the mass of the tumor before major surgery is attempted. For airway management, awake fiberoptic bronchoscopy and intubation enables the anesthetist to evaluate the large airways for obstruction and place the ETT beyond the obstruction while maintaining spontaneous ventilation. The effect of positional changes can be assessed with the bronchoscope. Spontaneous ventilation should be maintained as long as possible or throughout the procedure if feasible. The ability to effectively provide positive-pressure ventilation should be guaranteed prior to administering muscle relaxants. The use of a helium-oxygen mixture can improve airflow during partial obstruction by decreasing turbulence past the stenotic area.172 Mediastinal masses can cause compression of great vessels or cardiac chambers. Patients with any cardiac or great vessel involvement should receive only local anesthesia whenever possible, remain in the sitting position, and maintain spontaneous respirations. Cardiopulmonary bypass must be able to be implemented within minutes in the case of sudden cardiopulmonary collapse. - Patients with mediastinal masses may develop superior vena cava (SVC) syndrome, which is venous engorgement of the upper body caused by compression of the SVC. The following signs and symptoms are associated with SVC syndrome: dilation of collateral veins of the upper part of the thorax and neck; edema and rubor of the face, neck, upper torso, and airway; edema of the conjunctiva with or without proptosis; shortness of breath, headache, visual distortion, or altered mentation.173 Placement of IV lines in the lower extremities is preferred; insertion in sites above the SVC could delay the drug's effect as a result of slow distribution. Fluids should be administered with caution, because large volumes can worsen symptoms.

In the lateral position, the blood flow to the dependent lung is generally thought to be increased by __% compared with the same lung in the supine position. A. 10 B. 20 C. 30 D. 40

A. Perfusion - Gravity has some effect on distribution of pulmonary blood flow. In the lateral position, the blood flow to the dependent lung is generally thought to be increased by 10% compared with the same lung in the supine position.100 However, the distribution of pulmonary blood flow in various positions may be related more to inherent pulmonary vascular anatomy than to gravity (Fig. 66-23).101 The matching of ventilation and perfusion will usually be decreased in the lateral position compared with the supine position during anesthesia. Pulmonary arteriovenous shunt during general anesthesia will usually increase from approximately 5% in the supine position to 10% to 15% in the lateral position.102

The general incidence of hypoxemia is __% to __% during one-lung ventilation (OLV) A. 5-10 B. 10-15 C. 15-20 D. 20-25

A. - Thoracic surgery is greatly facilitated by the contribution of anesthesia care, which can isolate the movement of one lung during ventilation and create a quiet surgical field. Although this procedure requires advanced techniques related to airway management, it has been in existence almost as long as tracheal intubation itself. In 1928, Guedel, Magill, Waters, and other pioneers first achieved closed endotracheal anesthesia. The treatment of tuberculosis and empyema, however, required isolation of the infected lung, and in1931 Joseph Gale and Ralph Waters first described "closed endobronchial anesthesia." Bronchial blockade for selective ventilation and lung isolation was reported by Magill in 1936. In 1950, Björk and Carlens were credited with the first use of a double lumen endotracheal tube (ETT) for thoracic surgery, the bronchospirometric double lumen tube (DLT), which Carlens described the year before.1,2 DLTs evolved in design, and the use of the Fogarty embolectomy catheter for bronchial blockade gave way to the Univent tube invented by Inoue and colleagues. - Development of airway devices is continuously evolving, and anesthetists caring for patients undergoing thoracic surgery must be skilled at insertion, maintenance, and monitoring of these devices for proper function. However, the process of lung isolation facilitates the surgical procedure, but compounds a central concern in thoracic anesthesia: maintaining effective gas exchange in the face of ventilation and perfusion mismatch. General anesthesia creates atelectasis, which is compounded by muscle relaxants and lateral positioning, but ventilation and perfusion are further mismatched when the thorax is opened, and ceasing ventilation of one lung is the final insult. Fortunately, physiologic processes such as hypoxic pulmonary vasoconstriction combat the inherent shunt, and anesthetic management is geared toward supporting those processes while fostering oxygenation through various ventilation modalities. The general incidence of hypoxemia is 5% to 10% during one-lung ventilation (OLV).3 - Besides managing the complexities of a double lumen ETT, the anesthetist must also be cognizant of the effects of underlying disease as it relates to management of ventilation (bullous disease contraindicating nitrous oxide [N2O]), interactions with anesthetic drugs (small cell carcinoma being associated with myasthenic syndrome), and even concerns about oxygen toxicity (in patients treated with bleomycin and other chemotherapeutics).

This is considered one of the most effective methods for treating postoperative pain for thoracic surgery A. Thoracic epidural analgesia B. Paravetebral nerve block C. IV Ketamine D. IV Toradol

A. Analgesia for Thoracic Surgery - A thoracotomy is known as one of the most painful operations, and postoperative pain can be very protracted (as in postthoracotomy pain syndrome). Most of the pain is caused by resection of thoracic tissue and bone in order for the surgeon to enter the chest cavity. This can lead to complications such as pneumonia and atelectasis.87 Pain immediately after thoracic surgery causes splinting, decreased respiratory effort, hypoxemia, and respiratory acidosis. Aggressive management of pain is aimed at seeking a balance between comfort and respiratory depression in patients with decreased lung function. Residual pain exists in half of thoracotomy patients after 1 year, and in one-third of patients after 4 years.88 - Several options can be considered in the management of postoperative pain. Patients can titrate IV patient-controlled analgesia to obtain a more constant level of analgesia than that provided by intermittent intramuscular injections, but the benefits of avoiding systemic opioids have made regional anesthesia emerge as a superior method of pain control. - Thoracic epidural analgesia is considered one of the most effective methods for treating postoperative pain.129 An epidural catheter is placed at the level of T6 to T8 and infused with epidural opioids or dilute solutions of local anesthetics to provide analgesia. The efficacy of epidural analgesia may be improved with adjunctive interventions, such as IV administration of ketamine and nonsteroidal analgesics.130 - As an alternative regional anesthesia technique, paravertebral nerve blocks can be placed at the level of the incision plus one or two intercostal interspaces above and below. This technique provides good short-term pain relief and reduces opioid requirements. Paravertebral block provides quality pain control to rival epidural analgesia.131,132

This is performed to reconfirm the diagnosis (if a tumor compresses the airway) or to determine the invasion and obstruction of the distal airway (in relation to the extension of the bronchial resection). A. Fiberoptic bronchoscopy B. Rigid bronchoscopy C. Mediastinoscopy D. EBUS

A. Anesthetic Management for Common Surgical Procedures Flexible Fiberoptic Bronchoscopy - Flexible fiberoptic bronchoscopy is a diagnostic and therapeutic procedure of great value in the clinical practice of thoracic surgery and anesthesia. In many centers, it is common practice to perform flexible fiberoptic bronchoscopy before lung resections to reconfirm the diagnosis (if a tumor compresses the airway) or to determine the invasion and obstruction of the distal airway (in relation to the extension of the bronchial resection). Anesthetic Management - There are multiple techniques for flexible fiberoptic bronchoscopy. Options include awake versus general anesthesia and oral versus nasal approaches. Options for local anesthesia include: topical anesthesia via a nebulizer, handheld aerosol, or soaked pledgets; nerve blocks (laryngeal and/or glossopharyngeal nerves); and direct administration of local anesthetic through the bronchoscope (spray-as-you-go technique)162 with/without sedation/opioid or antisialogogues. Options during general anesthesia include spontaneous versus positive-pressure ventilation with/without muscle relaxation. Airway management during general anesthesia can be done with an endotracheal tube or a laryngeal mask airway (LMA). A Portex swivel connector (Smiths Medical, Ashford, Kent, UK) with a self-sealing valve is used to facilitate the ventilation and manipulation of the bronchoscope; at the same time inhalation and/or intravenous agents can be used for anesthesia. Patients who have copious secretions in the preoperative period should receive anticholinergic medication to ensure a dry field, which provides optimal visualization with the use of a flexible bronchoscope. - The advantages of an LMA technique include visualization of the vocal cords and subglottic structures as well as a lower airway resistance versus an ETT when the bronchoscope is inserted (Fig. 66-36). This is particularly useful in a patient with a difficult airway, when maintaining spontaneous respiration may be the safest method of anesthetic management.163 Self-expanding flexometallic tracheal and bronchial stents can be placed with fiberoptic or rigid bronchoscopy (Fig. 66-37). However, silastic airway stents require rigid bronchoscopy for placement.A.

In an Anesthetized, Paralyzed, Mechanically Ventilated patient, ventilation favors A. Independent B. Dependent

A. Anesthetized, Paralyzed, Mechanically Ventilated - Under mechanical ventilation, the diaphragm no longer contributes to ventilation of the lower lung, and FRC further declines as the compression from abdominal viscera is no longer counteracted by the force of the contracting diaphragm (Fig. 30.7). With the initiation of mechanical ventilation and the absence of diaphragmatic contraction, ventilation further shifts to follow the path of least resistance, favoring the nondependent lung. The ventilation-perfusion relationship further deteriorates. The addition of positive end-expiratory pressure (PEEP) to mechanical ventilation may help restore FRC and improve the ventilation-perfusion ratio.

This is a subpleural collection of air under the visceral pleura caused by a ruptured alveolus. A. Bleb B. Bullae C. Cysts D. Pneumatocele

A. Blebs, Bullae, Cysts, and Pneumatoceles Blebs - A bleb is a subpleural collection of air under the visceral pleura caused by a ruptured alveolus. The air dissects through the pulmonary parenchyma and enlarges to form a bubble on the surface of the lung. Blebs most commonly occur at the apices of the lung and can rupture into the interpleural space, causing a pneumothorax. A single episode of spontaneous pneumothorax is usually treated conservatively with chest tube drainage until the air-leak has stopped. Resection of blebs is commonly indicated for recurrent pneumothoraces, bilateral pneumothoraces, or prolonged chest tube drainage. Resection of blebs after a single pneumothorax may be indicated if the patient's occupation exposes them to significant rapid fluctuations in atmospheric pressure (e.g., flight crews or scuba divers). Resection is most commonly combined with a procedure to obliterate the pleural space by partial pleurectomy or pleural abrasion. Resection of blebs is most often performed by VATS. Although VATS procedures per se are generally associated with a limited need for postoperative analgesia, pleurectomy or abrasion is very painful.

The overall documented incidence of postthoracotomy ischemia is 5% and peaks on days 2 to 3 postoperatively. A. 5 B. 10 C. 15 D. 20

A. Concomitant Medical Conditions Cardiac Disease (See Chapter 39) Cardiac complications are the second most common cause of perioperative morbidity and mortality in the thoracic surgical population. Ischemia - Because the majority of pulmonary resection patients have a smoking history, they already have one risk factor for coronary artery disease. Elective pulmonary resection surgery is regarded as an intermediate-risk procedure in terms of perioperative cardiac ischemia.15 The overall documented incidence of postthoracotomy ischemia is 5% and peaks on days 2 to 3 postoperatively. Beyond the standard history, physical, and electrocardiogram, routine screening testing for cardiac disease does not appear to be cost-effective for all prethoracotomy patients. Noninvasive testing is indicated in patients with major (i.e., unstable ischemia, recent infarction, severe valvular disease, significant arrhythmia) or intermediate (i.e., stable angina, remote infarction, previous congestive failure, diabetes) clinical predictors of myocardial risk. Therapeutic options to be considered in patients with significant coronary artery disease are optimization of medical therapy, coronary angioplasty, or coronary artery bypass, either before or at the time of lung resection. Timing of lung resection surgery after a myocardial infarction is always a difficult decision to make. Limiting the delay to 4 to 6 weeks in a medically stable and fully investigated and optimized patient seems acceptable after myocardial infarction. The appropriate delay after coronary stenting is conventionally 4 to 6 weeks after bare metal stents and 12 months after drug-eluting stents.16 Surgery should be delayed until it is safe to temporarily discontinue major antiplatelet drugs (except aspirin). A recent study upholds the 6-week delay after bare metal stents but suggests that the risks after drug-eluting stents are minimal after 6 months

Chronic reflux of acidic gastric contents can lead to ulceration, inflammation, and eventually A. Benign esophageal stricture B. Esophageal perforation and rupture C.Achalasia D. Esophagorespirtory tract fistula

A. Esophageal Perforation and Rupture - There are multiple causes of esophageal perforation, including foreign bodies, endoscopy, bougienage, traumatic tracheal intubation, gastric tubes, and oropharyngeal suctioning. Iatrogenic causes are the most common, with upper gastrointestinal endoscopy being the most frequent cause. A rupture is a burst injury often caused by uncoordinated vomiting, straining associated with weight-lifting, childbirth, defecation, and crush injuries to the chest and abdomen. The rupture is usually located within 2 cm of the gastroesophageal junction on the left side. Rupture is the result of a sudden increase in abdominal pressure with a relaxed lower esophageal sphincter and an obstructed esophageal inlet. In contrast to a perforation, in the presence of a rupture, the stomach contents enter the mediastinum under high pressure and the patient becomes symptomatic much more abruptly. - In addition to chest and/or back pain, patients with intrathoracic esophageal perforation or rupture may develop hypotension, diaphoresis, tachypnea, cyanosis, emphysema, and hydrothorax or hydropneumothorax.207 Radiologic studies may reveal subcutaneous emphysema, pneumomediastinum, widening of the mediastinum, pleural effusion, and pneumoperitoneium. Minor perforations can in some cases be managed conservatively. Major injuries will rapidly develop mediastinitis and sepsis if not treated surgically, so repair and drainage is an emergency procedure usually performed via a left or right thoracotomy. Achalasia - Achalasia is a disorder in which there is a lack of peristalsis of the esophagus and a failure of the lower esophageal sphincter to relax in response to swallowing. Clinically, the patients have esophageal distention that may lead to chronic regurgitation and aspiration. The goal of treatment is to alleviate the distal obstruction. This can be done by either esophageal dilatation or by surgery. Dilatation, which carries with it the risk of perforation, can be achieved by mechanical, hydrostatic, or pneumatic means. The surgical repair consists of a Heller myotomy, which is an incision through the circular muscle of the esophagogastric junction. The myotomy is often combined with a hiatal hernia repair to prevent subsequent reflux. This can be performed via thoracotomy, laparotomy, or laparoscopy.208 The Dor operation is a modification of the Heller procedure in which a stent is inserted into the muscular defect created by the myotomy to prevent muscular reapposition and thus recurrent dysphagia.209 Esophagorespiratory Tract Fistula - Esophagorespiratory tract fistula in an adult is most often a result of malignancy. Sometimes the fistula is benign and may be caused by injury from a tracheal tube, trauma, or inflammation. Of the malignant fistulas, approximately 85% are secondary to esophageal cancer. In contrast to the pediatric patient with esophagorespiratory tract fistula, which usually connects the distal esophagus to the posterior tracheal wall, these fistulas may connect to any part of the respiratory tract.210 In most cases, the fistula can be seen on esophagoscopy or bronchoscopy. In malignant cases, the goal of surgery is usually palliation. The technique of lung isolation will depend on the location of the fistula. One option in adults with a distal tracheal fistula is the use of bilateral small (5-6 mm ID) endobronchial tubes.211 Zenker Diverticulum - Zenker diverticulum is actually a diverticulum of the lower pharynx that arises from a weakness at the junction of the thyropharyngeus and cricopharyngeus muscles just proximal to the esophagus. It is commonly considered an esophageal lesion because of its proximity to the upper esophagus and because the underlying cause may be a failure of relaxation of the upper esophageal sphincter during swallowing. Early symptoms may be nonspecific such as dysphagia or complaints of food being stuck in the throat. As the diverticulum enlarges, patients describe noisy swallowing, regurgitation of undigested food, and coughing spells while supine. Recurrent aspiration and pneumonia may develop. The major concern for anesthesia is the possibility of aspiration on induction of general anesthesia for excision of the diverticulum.212 Even prolonged fasting does not ensure that the diverticulum will be empty. The best method to empty the diverticulum is to have the patient express and regurgitate the contents immediately before induction. Many of these patients will be used to doing this on a regular basis at home. Because the diverticulum orifice is almost always above the level of the cricoid cartilage, cricoid pressure during a rapid-sequence induction does not prevent aspiration and may contribute to aspiration by causing the sac to empty into the pharynx. Surgical excision is usually done through a lower left neck incision. The safest method of managing the airway for these patients may be awake fiberoptic intubation. However, intubation has been managed without incident using a modified rapid-sequence induction without cricoid pressure and with the patient supine and in a head-up position of 20 to 30 degrees. Other considerations in these patients include the possibility of perforation of the diverticulum when passing an orogastric or nasogastic tube or an esophageal bougie.

This is an anatomic pulmonary resection of the pulmonary artery, vein, bronchus, and parenchyma of a particular segment of the lung. It is usually performed as surgical therapy for patients with primary lung cancer and limited cardiorespiratory reserves. A. Segmentectomy B. Wedge resection

A. Limited Pulmonary Resections: Segmentectomy and Wedge Resection - A limited pulmonary resection is one in which less than a complete lobe is removed. The two procedures fitting this definition are segmentectomy and wedge resection. Segmentectomy is an anatomic pulmonary resection of the pulmonary artery, vein, bronchus, and parenchyma of a particular segment of the lung. Segmentectomy is usually performed as surgical therapy for patients with primary lung cancer and limited cardiorespiratory reserves. In contrast, a wedge resection is a nonanatomic removal of a portion of the lung parenchyma with a 1.5- to 2.0-cm margin and can be accomplished by open thoracotomy or VATS. Wedge resections are most commonly performed for diagnosis of lung lesions with unknown histology or for palliation in patients with metastatic lesions in the lungs from distant primary tumors. Lung cancers that are considered for limited resection are usually smaller than 3 cm and are located in the periphery of the lung with regional lymph nodes free of metastatic cancer. - A group of patients considered for limited pulmonary resection are those who develop a new primary lesion after a previous lobectomy or pneumonectomy. The patient with compromised lung function presents a greater risk in the intraoperative period (hypoxemia during OLV or prolonged intubation after surgery). Cerfolio and associates195 reported that lung cancer patients with compromised pulmonary function can safely undergo limited pulmonary resection if selected appropriately. Segmentectomies and wedge resections can be performed with any of the standard thoracotomy or VATS incisions. Segments that are most commonly resected are in the upper lobes or the superior segments of the lower lobes. - Anesthetic technique and monitoring are essentially the same as for larger pulmonary resections. To facilitate surgical exposure and achieve OLV, it is necessary to use either a DLT or a bronchial blocker. If the patient had a previous contralateral lobectomy or a pneumonectomy, selective lobar collapse with the use of a bronchial blocker will facilitate surgical exposure while maintaining oxygenation. In selected cases, the combined use of a DLT and a bronchial blocker will allow selective lobar collapse/ventilation in the ipsilateral lung.196 It is very important to use low tidal volumes (i.e., 3-5 mL/kg) during selective lobar ventilation, particularly in patients with previous pneumonectomy to prevent overinflation in the remaining lobes. - Segmentectomy plays a significant role in the management of patients with a second primary lung cancer. Many of these patients have previously undergone thoracic surgery, including previous lobectomy or pneumonectomy; therefore the potential for increased intraoperative bleeding is always a risk. In addition, because many of these patients have compromised lung function, early extubation may not be feasible. A common complication after surgery is an air leak. Chest tubes are placed to maximize postoperative expansion and minimize space complications. Suction and underwater-seal chest drainage is used in the postoperative period.

The most common lung isolation technique is A. DLT B. Bronchial blocker C. Single-lumen tubes

A. Lung Isolation - Lung isolation techniques are primarily designed to facilitate OLV in patients undergoing cardiac, thoracic, mediastinal, vascular, esophageal, or orthopedic procedures involving the chest cavity.74 Lung isolation is also used to protect the lung from soiling by the contralateral lung in such cases as bronchopleural fistula, pulmonary hemorrhage, and whole-lung lavage. In addition, lung isolation can be used to provide differential patterns of ventilation in cases of unilateral reperfusion injury (after lung transplantation or pulmonary thromboendarterectomy) or in unilateral lung trauma. - Lung isolation can be achieved by three different methods: DLTs, bronchial blockers, or single-lumen tubes (SLTs) (Table 66-4). The most common technique is with a DLT. The DLT is a bifurcated tube with both an endotracheal and an endobronchial lumen and can be used to achieve isolation of either the right or the left lung. The second method involves blockade of a mainstem bronchus to allow lung collapse distal to the occlusion. These bronchial blockers can be used with a standard endotracheal tube or contained within a separate channel inside a modified SLT such as the Univent tube (LMA North America, San Diego, Calif). The final option for lung isolation is to use either an SLT or an endobronchial tube that is advanced into the contralateral mainstem bronchus, protecting this lung while allowing collapse of the lung on the side of surgery (Fig. 66-9). This technique is rarely used today in adult practice (except in some cases of difficult airways, carinal resection, or after a pneumonectomy), owing to the limited access to the nonventilated lung and the difficulty in positioning a standard SLT in the bronchus. This technique is still used when needed in infants and small children: an uncuffed uncut pediatric-size endotracheal tube is advanced into the mainstem bronchus under direct guidance with an infant bronchoscope.

For bilateral procedures it is advisable to operate first on the lung that has better gas exchange and less propensity to desaturate during OLV. For the majority of patients, this means operating on the ______ side first. A, Right B. Left

A. Mechanical Restriction of Pulmonary Blood Flow - It is possible for the surgeon to directly compress or clamp the blood flow to the nonventilated lung.159 This can be done temporarily in emergency desaturation situations or definitively in cases of pneumonectomy or lung transplantation. Another technique of mechanical limitation of blood flow to the nonventilated lung is the inflation of a pulmonary artery catheter balloon in the main pulmonary artery of the operative lung. The pulmonary artery catheter can be positioned at induction with fluoroscopic guidance and inflated as needed intraoperatively. This has been shown to be a useful technique for resection of large pulmonary arteriovenous fistulas.160 Hypoxemia Prophylaxis - Most treatments outlined as therapies for hypoxemia can be used prophylactically to prevent hypoxemia in patients who are at high risk of desaturation during OLV. The advantage of prophylactic therapy of hypoxemia, in addition to the obvious patient safety benefit, is that maneuvers involving CPAP or alternative ventilation patterns of the operative lung can be instituted at the onset of OLV in a controlled fashion and will not require interruption of surgery and emergent reinflation of the nonventilated lung at a time that may be extremely disadvantageous. Bilateral Pulmonary Surgery - Because of mechanical trauma to the operative lung, the gas exchange in this lung will always be temporarily impaired after OLV. Also HPV offset may be delayed after reinflation of the first lung collapsed. Desaturation during bilateral lung procedures is particularly a problem during the second period of OLV (i.e., during OLV of the lung that has already had surgery).161 Thus, for bilateral procedures it is advisable to operate first on the lung that has better gas exchange and less propensity to desaturate during OLV. For the majority of patients, this means operating on the right side first.

For thoracotomies, the arterial cannula is generally placed in the ____________ arm, where it is more easily stabilized. A. Dependent B. Independent

A. Monitoring Plan - The purpose of monitoring during thoracic surgery is the quick recognition of sudden and severe changes in ventilation and hemodynamics that can accompany positioning, OLV, and surgical manipulation of the airway and thoracic structures. According to American Association of Nurse Anesthetists (AANA) guidelines, standard monitors should be used.78 Airway pressure monitoring helps detect changes in airway compliance and assists in identifying the proper placement of DLTs. Capnography is useful for determining the adequacy of when one lung is deflated, and also for detecting abrupt changes in cardiac output, which may accompany positioning or surgical manipulation. Considering pre-existing ventilation derangements and changes with lateral position, the gradient of end-tidal to arterial CO2 may be wider than the typical 5 mm Hg. There is evidence that, even if the gradient is determined early, it may not remain the same throughout the intraoperative course. Electrocardiogram - All patients require continuous monitoring of the ECG. Typically, anesthetists monitor a limb lead (II) for easy rhythm recognition and a precordial lead to add sensitivity for detection of ischemia. A landmark article by London identified V5 as the precordial lead which would add the most sensitivity to ischemia detection.79 That article noted that monitoring a combination of leads II and V5 will help detect more than 85% of myocardial ischemic episodes. A more recent study (2014), which accounted for both the changing pattern of ischemia over time (onset vs. peak), as well as comparison to preoperative values, found lead V4 to be the most sensitive for detecting ischemia, followed by V5.38 In practice, concerns for positioning, surgical site prep, and access to the skin electrode may result in the precordial lead not being placed in the precisely correct location. Various studies have determined that myocardial ischemia frequently occurs in leads V3 through V6.38,80,81 A consistent finding is that monitoring a combination of leads is significantly more effective than monitoring a single lead.38,79,82 A second intraoperative lead monitored in the V4-5 position or in whatever anterolateral position provides the most isoelectric ST segment is desirable.38 Arterial Pressure Monitoring - Arterial blood pressure monitoring allows the anesthetist to identify acute hypotension during surgical manipulation. It also facilitates sampling of arterial blood for blood gas analysis. For thoracotomies, the arterial cannula is generally placed in the dependent arm, where it is more easily stabilized. For mediastinoscopy, the arterial monitoring site is selected to provide indication of innominate artery occlusion (see section on mediastinoscopy later).

The major cause of perioperative morbidity and mortality in the thoracic surgical population is A. Respiratory complications B. Cardiac complications C. Vascular complications D. Renal complications

A. Perioperative Complications - The major cause of perioperative morbidity and mortality in the thoracic surgical population is respiratory complications. Major respiratory complications—atelectasis, pneumonia, and respiratory failure—occur in 15% to 20% of patients and account for the majority of the expected 3% to 4% mortality rate.2 For other types of surgery, cardiac and vascular complications are the leading cause of early perioperative morbidity and mortality. Cardiac complications such as arrhythmia and ischemia occur in 10% to 15% of the thoracic population.

In a healthy, conscious, spontaneously breathing patient, the ventilation of the dependent lung increases approximately __% when the patient is turned to the lateral position. A. 10 B. 20 C. 30 D. 40

A. Physiologic Changes in the Lateral Position (See Chapter 41) Ventilation - Significant changes in ventilation develop between the lungs when the patient is placed in the lateral position.95 The compliance curves of the two lungs are different because of their difference in sizes. The lateral position, anesthesia, paralysis, and opening of the thorax all combine to magnify these differences between the lungs (Fig. 66-22). The compliance curve (change in volume versus change in pressure) of a lung depends on the balance of two "springs": the chest wall (normally distending the lung) and the elastic recoil of the lung itself. Any factor that changes the mechanics of either of these springs places the lung on a different compliance curve.96 - In a healthy, conscious, spontaneously breathing patient, the ventilation of the dependent lung increases approximately 10% when the patient is turned to the lateral position. Once the patient is anesthetized and paralyzed, the ventilation of the dependent lung then decreases by 15%. Although the ventilation does not change significantly once the nondependent hemithorax is open, the FRC of this nondependent lung tends to increase by approximately 10%. These changes depend on the method used for ventilation in the individual patient. When the chest is open, because of disruption of the chest wall, both lungs tend to collapse to a minimal lung volume if expiration is prolonged. Thus the end-expiratory volume of each lung is directly a function of the time allowed for expiration. The compliance of the entire respiratory system increases significantly once the nondependent hemithorax is open. - Because of the fall in FRC and compliance of the nondependent lung in the lateral position, application of PEEP selectively to this lung only (using a DLT and two anesthetic circuits) will improve gas exchange.97 This is different from the effect of nonselectively applying PEEP to both lungs in the lateral position, where PEEP tends to preferentially distribute to the most compliant lung regions and will tend to hyperinflate the nondependent lung without causing any improvement in gas exchange.98 - Atelectasis will develop in an average of 6% of the lung parenchyma after induction of anesthesia in the supine position. This atelectasis will be evenly distributed in the dependent portions of both lungs.99 By turning the patient to the lateral position, there will be a slight decrease of total atelectasis to 5% of lung volume, but this will now be concentrated totally in the dependent lung.

Negative pleural pressure is greatest at the A. Apex B. Base C. Mid lung D. Lateral

A. Physiology of the Lateral Decubitus Position - Positional changes and alterations in chest wall integrity produce significant alterations in ventilation and perfusion of the lungs during thoracic surgery. Due to the critical nature of the procedure and the degree of pulmonary pathology that is frequently associated with these patients, a thorough knowledge of positioning, ventilation, perfusion, and strategies to minimize the potential for hypoxia are discussed. A thorough discussion of pulmonary anatomy and physiology is present in Chapter 29. Upright Position - The distribution of perfusion in the lungs depends on gravity in relation to the level of the heart and on pressures transmitted through alveoli. In a spontaneously breathing, upright patient, perfusion increases from the apex to the base of the lung (Fig. 30.1). Ventilation also increases from apex to base, based on the relative compliance of alveoli. Owing to downward traction from gravity, negative pleural pressure is greatest at the apex of the lung, and this factor keeps alveoli distended (Fig. 30.2). Dependent alveoli are less distended and therefore more compliant (can expand by a greater volume for a given pressure change because they are starting at a lower resting volume). Therefore, most of a tidal volume (VT) breath is distributed to the dependent alveoli (Fig. 30.3). The increase in both ventilation and perfusion from apex to base is not parallel, and is certainly more complexly arranged than in neatly-divided zones. However, the general increase in ventilation and perfusion from top to bottom results in efficient gas exchange.

If a left-sided DLT or bronchial blocker is used for a left pneumonectomy, it must be withdrawn ________ stapling the bronchus to avoid accidental inclusion into the suture line. A. Before B. After

A. Pneumonectomy - Complete removal of the lung is required when a lobectomy or its modifications is not adequate to remove the local disease and/or ipsilateral lymph node metastases. Atelectasis and pneumonia occur after pneumonectomy as they do after lobectomy, but they may be less of a problem because of the absence of residual parenchymal dysfunction on the operative side. However, the mortality rate after pneumonectomy exceeds that for lobectomy because of postoperative cardiac complications and acute lung injury. The overall operative mortality for the first 30 days after pneumonectomy ranges from 5% to 13% and correlates inversely with the surgical case volume.183 The risk of complications increases fivefold in patients aged 65 and older.184 - Thoracotomy for pneumonectomy is usually performed through a standard posterolateral incision. After all vessels are stapled, stapling of the bronchus occurs and the entire lung is taken from the chest. A test for air leaks is generally performed at this point and reconstruction of the bronchial stump is completed. The bronchial stump should be as short as possible to prevent a pocket for the collection of secretions. There is no consensus among thoracic surgeons on the best method of management of the postpneumonectomy space. If suction is applied to an empty hemithorax or a chest drain is connected to a standard underwater-seal system, it may cause a mediastinal shift with hemodynamic collapse. Some thoracic surgeons do not place a chest drain after a pneumonectomy; some thoracic surgeons prefer to use a temporary drainage catheter to add or remove air. The removal of air, ranging from 0.75 to 19831.5 L, is necessary to empty the chest and to keep the mediastinum and the trachea in the midline ("balanced"). Some surgeons place a specifically designed postpneumonectomy chest drainage system with both high- and low-pressure underwater relief valves to balance the mediastinum.185 A chest radiograph is mandatory after the patient arrives in the postanesthesia care unit or in the surgical intensive care unit to assess the mediastinal shift. The patient scheduled to undergo a pneumonectomy is considered at high risk for perioperative morbidity and mortality. The placement of large-bore intravenous lines is necessary in case blood products need to be administered. An invasive arterial line is placed for measurement of beat-to-beat blood pressure and to monitor arterial blood gases. A CVP catheter is recommended to help guide intravascular fluid management and to administer vasopressors if needed, specifically in the postoperative period. A major lung resection, such as pneumonectomy, decreases ventilatory function and has significant effects on right ventricular function.186 - Immediately after pneumonectomy, the right ventricle may dilate and the right ventricular function decreases. Increased right ventricular afterload is caused by an increase in pulmonary artery pressure and pulmonary vascular resistance. This is considered to be one of the main causes of right ventricular dysfunction after a major lung resection. - Management of lung isolation in a pneumonectomy patient can be achieved with a DLT, bronchial blocker, or SLT. When using a DLT for a pneumonectomy patient, it is optimal to use a device that does not interfere with the ipsilateral airway (i.e., for a right pneumonectomy use a left-sided DLT). If a left-sided DLT or bronchial blocker is used for a left pneumonectomy, it must be withdrawn before stapling the bronchus to avoid accidental inclusion into the suture line. - Specific areas of concern in the management of the patient undergoing pneumonectomy include: (1) fluid management, (2) intraoperative tidal volume, and (3) acute lung injury post surgery. Fluid administration after major lung resection continues to be an issue. In a retrospective report by Zeldin and associates,187 the risk factors that were identified for the development of acute lung injury ("postpneumonectomy pulmonary edema") were a right-sided pneumonectomy, increased perioperative IV fluid administration, and increased urine output in the postoperative period. A more recent study by Licker and colleagues has shown that the excessive administration of intravenous fluids in thoracic surgical patients (more than 3 L in the first 24 hours) is an independent risk related to an acute lung injury.188 There is reasonable clinical evidence that excessive fluid administration is associated with the development of an acute lung injury, which has a high mortality rate after pneumonectomy. Consequently, pneumonectomy patients should have restricted intraoperative fluid administration while preserving renal function. Some cases may require the use of inotropes/vasopressors to maintain hemodynamic stability while restricting fluids (see Box 66-8). - Respiratory failure is a leading cause of postoperative morbidity and mortality in patients undergoing pneumonectomy. A retrospective report189 involving 170 pneumonectomy patients showed that patients who received median tidal volumes greater than 8 mL/kg had a greater risk of respiratory failure in the postoperative period after pneumonectomy. In contrast, patients who received tidal volumes less than 6 mL/kg were at lower risk for respiratory failure. Schilling and colleagues190 have shown that a tidal volume of 5 mL/kg during OLV significantly reduces the inflammatory response to alveolar cytokines. Considering these factors, it is prudent to use lower tidal volumes (i.e., 5-6 mL/kg ideal body weight) in the pneumonectomy patient and limit peak and plateau inspiratory pressures (i.e., <35 and 25 cm H2O, respectively) during OLV. - The incidence of acute lung injury (postpneumonectomy pulmonary edema) after pneumonectomy is only 4%. However, the mortality rate is 30% to 50%. The etiology seems multifactorial. One study190 has identified four independent risk factors for acute lung injury after pulmonary resection: (1) pneumonectomy; (2) excessive administration of fluids in the intraoperative period; (3) high intraoperative ventilatory pressure index (combined airway pressure and time); and (4) preoperative alcohol abuse. The incidence of an acute lung injury is greater for a right- versus a left-sided pneumonectomy. This may be related to the higher postoperative pulmonary artery pressures after a right versus a left pneumonectomy (Fig. 66-43).191 Currently, only symptomatic management is appropriate for this lung injury. This includes fluid restrictions, diuretic administration, low ventilatory pressures and tidal volumes (if mechanical ventilation is used), and measures to decrease the pulmonary artery pressure. Extracorporeal techniques of ventilatory assist may be useful in managing this complication.192q

Fiberoptic bronchoscopy is performed first through the A. Tracheal lumen B. Bronchial lumen

A. Positioning of Double-Lumen Tubes - Auscultation alone is unreliable for confirmation of proper DLT placement. Auscultation (Fig. 66-14) and bronchoscopy should both be used each time a DLT is placed and again when the patient is repositioned. Fiberoptic bronchoscopy is performed first through the tracheal lumen to ensure that the endobronchial portion of the DLT is in the left bronchus and that there is no bronchial cuff herniation over the carina after inflation. Through the tracheal view, the blue endobronchial cuff ideally should be seen approximately 5 mm below the tracheal carina in the left bronchus. It is crucial to identify the takeoff of the right upper lobe bronchus through the tracheal view. Going inside this right upper lobe with the bronchoscope should reveal three orifices (apical, anterior, posterior). This is the only structure in the tracheobronchial tree that has three orifices. In the supine patient, the takeoff of the right upper lobe is normally on the lateral wall of the right mainstem bronchus at the 3- to 4-o'clock position in relation to the main carina. Broncho-Cath tubes from Mallinckrodt (St. Louis, Mo) have a radiopaque line encircling the tube. This line is proximal to the bronchial cuff and can be useful while positioning a left-sided DLT. The radiopaque marker is 4 cm from the distal tip of the endobronchial lumen. This marker reflects white during fiberoptic visualization and, when positioned slightly above the tracheal carina, should provide the necessary margin of safety for positioning into the left mainstem bronchus.82 The next observation with the fiberoptic bronchoscope is made through the endobronchial lumen to check for patency of the tube and determination of the margin of safety. The orifices of both the left upper and lower lobes must be identified to avoid distal impaction in the left lower lobe and occlusion of the left upper lobe (Fig. 66-15). Figure 66-16 displays the tracheobronchial anatomy along with fiberoptic bronchoscopy findings from the endotracheal or endobronchial lumen for a left-sided DLT.

Which of the following is not a Predictor of Airway Compromise in Children with a Mediastinal Mass A. Posterior location B. Histologic diagnosis of lymphoma C. Superior vena cava syndrome D. Radiologic evidence of major vessel compression or displacement

A. Predictors of Airway Compromise in Children with a Mediastinal Mass • Anterior location • Histologic diagnosis of lymphoma • Superior vena cava syndrome • Radiologic evidence of major vessel compression or displacement • Pericardial or pleural effusion

This is is the leading cause of cancer deaths in the United States A. Bronchogenic carcinoma B. Adenocarcinoma C. Oat Cell carcinoma D. Squamous cell carcinoma

A. Preoperative Preparation - Bronchogenic carcinoma is the leading cause of cancer deaths in the United States.4 Lung cancer is most often discovered only once the patients exhibit symptoms, and even with aggressive multimodal treatment, prognosis is poor. A review of patients with non-small cell carcinoma found a 5-year survival rate of less than 7%.5 However, resection of the affected lung tissue offers a better prognosis as compared to radiation and chemotherapy, and so cancer is a common indication for lung surgery. There is a strong association between lung cancer and chronic obstructive pulmonary disease (COPD), such that the incidence of lung cancer is four times higher among COPD sufferers than it is in the general population, and underlying COPD almost doubles the 3-year mortality from lung cancer.6-8 Many patients presenting for lung surgery will have complex underlying pathology. Therefore, evaluating respiratory function and predicting postresection function are crucial to anticipating the patient's intraoperative and postoperative care needs. - The surgical risk assessment for patients in need of pulmonary resection surgery focuses on the risk of potential postoperative complications and whether postoperative pulmonary function will be sufficient to allow for an adequate quality of life. Between 20% and 30% of patients with lung cancer are found to be surgical candidates,9 and almost 40% of these candidates are disqualified based on poor lung function alone.10 The anesthetic risk assessment specific to pulmonary surgery focuses on how the underlying pathology will challenge the maintenance of adequate gas exchange and general homeostasis under OLV, and the potential for postoperative respiratory failure to make weaning and extubation difficult. However, mortality from unresected carcinomas is sufficiently high that the risks of postoperative complications would need to be extraordinarily significant before they would preclude surgery. COPD is the progressive destruction of alveoli and lung architecture that occurs over the course of years. Considering the ageing nature of our population and the increase in the incidence of obesity, it is not surprising that lung resection surgeries are now being performed on more patients who have end-stage COPD, morbid obesity, or who are of advanced age.11,12 Evidence shows that these patients can be treated safely. The American College of Chest Physicians (ACCP) does not set a maximal age cutoff for pulmonary surgical candidacy.13 Changes in surgical techniques, especially the use of video-assisted thoracoscopic surgery, have markedly decreased the incidence of postoperative pulmonary complications, and will necessitate a reevaluation of the testing required for preoperative assessment.14 Given the large physiologic changes that occur after pneumonectomy, complete pulmonary function testing, as well as cardiac testing, is indicated. - Fear of creating pulmonary insufficiency by lung resection is an important concern, and numerous studies have attempted to determine the lowest limit of pulmonary function that will allow surgery to be safely performed. However, research findings are limited in their ability to predict particular complications. Studies performed to predict postoperative pulmonary complications after lung resection demonstrate that patients develop both pulmonary and cardiac complications such as dysrhythmias, myocardial infarction, pulmonary embolism, pneumonia, and empyema. These complications influence the duration of mechanical ventilation and outcome; however, none of these complications can be predicted by preoperative studies of pulmonary function. Box 30.1 presents some commonly-used preoperative assessment criteria for lung resection.

Which of the following is an inappropriate intervention for Induced Pulmonary Hemorrhage A. Initially position the patient with the bleeding lung independent. B. Withdraw the pulmonary artery catheter several centimeters, leaving it in the main pulmonary artery C. Isolate the lung by endobronchial double- or single-lumen tube or bronchial blocker. D. Position the patient with the isolated bleeding lung nondependent.

A. Pulmonary artery Catheter-Induced Hemorrhage - Hemoptysis in a patient with a pulmonary artery catheter must be assumed to be caused by perforation of a pulmonary vessel by the catheter until proven otherwise. The mortality rate may exceed 50%. This complication seems to be occurring less than previously, possibly related to stricter indications for the use of pulmonary artery catheters and more appropriate management of the catheters with less reliance on wedge measurements. Therapy for pulmonary artery catheter-induced hemorrhage should follow an organized protocol with some variation depending on the severity of the hemorrhage (Box 66-13). During Weaning from Cardiopulmonary Bypass - Weaning from CPB is one of the situations in which pulmonary artery catheter-induced hemorrhage is most likely to occur. Management of the pulmonary artery catheter during CPB by withdrawal from a potential wedge depth and observing the pulmonary artery pressure waveform to avoid wedging during CPB may decrease the risk of this complication. When hemoptysis does occur in this situation, there are several management options available (Fig. 66-50). The anesthesiologist should resist the temptation to rapidly reverse the anticoagulation to quickly discontinue CPB, because this can lead to fatal asphyxiation from hemorrhage. Resumption of full CPB ensures oxygenation while the tracheobronchial tree is suctioned and then visualized with fiberoptic bronchoscopy. The use of a pulmonary artery vent may be required to decrease the pulmonary blood flow sufficiently to define the bleeding site (usually the right lower lobe). The pleural cavity should be opened to assess the lung parenchymal damage. Conservative management with lung isolation, to avoid lung resection, is optimal therapy if possible. In patients with persistent hemorrhage who are not candidates for lung resection, temporary lobar pulmonary artery occlusion with a vascular loop during and after weaning from CPB may be an option. Posttracheostomy Hemorrhage - Hemorrhage in the immediate postoperative period after a tracheostomy is usually from local vessels in the incision such as the anterior jugular or inferior thyroid veins. Massive hemorrhage 1 to 6 weeks postoperatively is most commonly caused by tracheoinnominate artery fistula.239 A small sentinel bleed occurs in most patients before a massive bleed. The management protocol for tracheoinnominate artery fistula is outlined in Box 66-14. Management of the Patient with a Pulmonary Artery Catheter Induced Pulmonary Hemorrhage • Initially position the patient with the bleeding lung dependent. • Perform endotracheal intubation, oxygenation, airway toilet. • Isolate the lung by endobronchial double- or single-lumen tube or bronchial blocker. • Withdraw the pulmonary artery catheter several centimeters, leaving it in the main pulmonary artery. Do not inflate the balloon (except with fluoroscopic guidance). • Position the patient with the isolated bleeding lung nondependent. Administer positive end-expiratory pressure to the bleeding lung if possible. • Transport the patient to medical imaging for diagnosis and embolization if feasible.

Sleeve pneumonectomy are most commonly performed for _____-sided tumors and can usually be performed without cardiopulmonary bypass via a _____ thoracotomy. A. Right B. Left

A. Sleeve Pneumonectomy - Tumors involving the most proximal portions of the mainstem bronchus and the carina may require a sleeve pneumonectomy. These are most commonly performed for right-sided tumors and can usually be performed without cardiopulmonary bypass via a right thoracotomy. A long SLT can be advanced across into the left mainstem bronchus during the period of tracheo-bronchial anastomosis. High-frequency positive-pressure ventilation (HFPPV) has also been used for this procedure and the combined use of HFPPV and a DLT has been described.194 Because the carina is surgically more accessible from the right side, left sleeve pneumonectomies are commonly performed as a two-stage operation. First is a left thoracotomy and pneumonectomy, and a right thoracotomy for the carinal excision follows. The complication and mortality rates are higher, and the 5-year survival (20%) is significantly lower than for other pulmonary resections. Postpneumonectomy pulmonary edema is particularly a problem after right sleeve pneumonectomy.

Turning the patient to the lateral position will __________ respiratory dead space and the arterial to end-tidal CO2 tension gradient (Pa-etCO2). A. Increase B. Decrease

A. Tidal Volume - There will be an optimal combination of tidal volume, respiratory rate, I:E ratio, or pressure- or volume-control ventilation for each individual patient during OLV. However, to try to assess each of these parameters while still providing anesthesia with the available anesthetic ventilators is not practical, and the clinician must initially rely on a simplified strategy (Table 66-9). The results of alterations in tidal volume are unpredictable. This may be caused partly by the interaction of tidal volume with auto-PEEP. The use of 5 to 6 mL/kg ideal body weight tidal volumes plus 5 cm H2O PEEP initially for most patients (except those with COPD) seems a logical starting point during OLV. Tidal volume should be managed so that peak airway pressures do not exceed 35 cm H2O. This will correspond to a plateau airway pressure of approximately 25 cm H2O.136 Peak airway pressures exceeding 40 cm 1972H2O may contribute to hyperinflation injury of the ventilated lung during OLV.137 - Turning the patient to the lateral position will increase respiratory dead space and the arterial to end-tidal CO2 tension gradient (Pa-etCO2). This will usually require a 20% increase in minute ventilation to maintain the same PaCO2. Individual variations in Pa-ETCO2 gradient can become much larger, and PetCO2 is less reliable as a monitor of PaCO2 during OLV. This effect is possibly because there are individual differences in the excretion of CO2 between the dependent and nondependent lungs.

This BB has a retractable loop that is placed over the fiberoptic bronchoscope, which is then used to guide the blocker into place. They usually advance easily into the right mainstem bronchus without the loop A. Arndt blocker B. Cohen blocker C. Fuji uniblocker D. EZ blocker

A. Wire-Guided Endobronchial Blocker (Arndt Blocker) - Figure 66-18, A, displays the Arndt blockers. The Arndt blocker has a retractable loop that is placed over the fiberoptic bronchoscope, which is then used to guide the blocker into place. The Arndt blockers usually advance easily into the right mainstem bronchus without the loop. The original Arndt design was an elliptical blocker, but a spherical blocker is now available that may function better in the short, right mainstem bronchus. Cohen Endobronchial Blocker - The Cohen blocker (see Fig. 66-18, B, left) uses a wheel located in the most proximal part of the unit that deflects the tip of the distal part of the blocker into the desired bronchus. This blocker has been preangled at the distal tip to facilitate insertion into a target bronchus. On the distal shaft above the balloon, there is an arrow that, when seen with the fiberoptic bronchoscope, indicates in which direction the tip deflects. To position the Cohen blocker, the arrow is aligned with the bronchus to be intubated, the proximal wheel is turned to deflect the tip toward the desired side, and then the blocker is advanced with fiberoptic guidance. Fuji Uniblocker - The Fuji Uniblocker (see Fig. 66-18, B, right) is an independent blocker that is made of silicone material and has a simple, fixed distal hockey-stick (ice hockey) angulation to facilitate insertion. The blocker is simply rotated to the left or right as needed under fiberoptic bronchoscope guidance for placement in the required bronchus. EZ Blocker - The EZ blocker is a recently introduced 7-Fr, 4-lumen catheter with a Y-shaped bifurcation. Each distal end has a balloon that can be guided into the right and left main bronchus. This device comes with its own multiport adapter and is used through an 8.0 SLT. The end of the Y sits in the tracheal carina. Each distal end is positioned into the right and left bronchus, and the bronchial balloon is inflated in the operative side for lung isolation.

If hypoxia is encountered during one lung ventilation, PEEP applied (or titrated upward) to the ___________ lung acts to recruit collapsed airways, increasing lung compliance and FRC. A. Dependent B. Nondependent

A. Dependent - Besides shunt flow through the operative lung, atelectasis and reduced FRC in the dependent lung may also degrade the PaO2. If CPAP to the nondependent lung does not improve oxygenation, PEEP applied (or titrated upward) to the dependent, ventilated lung acts to recruit collapsed airways, increasing lung compliance and FRC. Excessive PEEP may detrimentally reduce cardiac output. Combined with a fast respiratory rate and/or high inspiratory/expiratory ratio (I:E ratio), PEEP may impair adequate exhalation, leading to a net volume increase through auto-PEEP and the potential for volutrauma to the dependent lung. The actual end-expiratory pressure should be monitored during OLV to ensure that it does not significantly exceed the intended level of PEEP. - Other methods of improving oxygenation during OLV include combining PEEP and CPAP to the respective lungs, and intermittent reinflation of the nondependent lung. Innovative ventilatory approaches such as high-frequency jet ventilation to the operative lung and selective oxygenation to nonoperative lobes of the operative lung via a bronchial blocker or bronchoscope are also used.155-158 Jet ventilation is effective at reducing the shunt, but lung movement can be deleterious, which makes monitoring CO2 levels and effecting CO2 removal challenging.159

The most common problems and complications associated with the use of a DLT are (Select 2) A. Barotrauma B. Airway trauma C. Malpositioning D. Infection

B & C Problems Related to Double-Lumen Tubes - The most common problems and complications associated with the use of a DLT are malpositioning and airway trauma. A malpositioned DLT will fail to allow collapse of the lung, causing gas trapping during positive-pressure ventilation, or it may partially collapse the ventilated or dependent lung, producing hypoxemia. A common cause of malpositioning is dislodgment of the endobronchial cuff because of overinflation, surgical manipulation of the bronchus, or extension of the head and neck during or after patient positioning. Fiberoptic bronchoscopy is the recommended method to diagnose and correct intraoperative malpositioning of DLTs, along with proper recognition of tracheobronchial anatomy. If a DLT is malpositioned with the patient in the supine or lateral decubitus position, there is a greater likelihood of hypoxemia during OLV. If a DLT is in the optimal position, but lung deflation is not completely achieved, a suction catheter should be passed to the side where lung collapse is supposed to occur. This suction will expedite lung deflation. The suction catheter must then be removed to avoid including it in a suture line. - Airway trauma and rupture of the membranous part of the trachea or the bronchus is a potential complication with the use of DLTs.83 Airway trauma can occur from an oversized DLT or when an undersized DLT migrates distally into the lobar bronchus and the main (i.e., tracheal) body of the DLT comes into the bronchus, producing lacerations or rupture of the airway. Airway damage during the use of DLTs can present an unexpected air leak, 1961subcutaneous emphysema, massive airway bleeding into the lumen of the DLT, or protrusion of the endotracheal or endobronchial cuffs into the surgical field, with visualization of this by the surgeon. If any of the aforementioned problems occur, a bronchoscopic examination and surgical repair should be performed. Another potential problem is the development of a tension pneumothorax in the dependent, ventilated lung during OLV.84

Hypoxic pulmonary vasoconstriction can be overriden by A. Hypovolemia B. Hypervolemia C. High cardiac output D. Low cardiac output

B & C. - HPV is effective in decreasing shunt flow, and attempting to avoid drugs or events that inhibit this mechanism is important (Box 30.7). Alveolar and intravascular volume derangements can inhibit the effects of HPV. Hypervolemia or high cardiac output may override HPV by recruiting constricted vessels.112 Conversely, hypovolemia may trigger adrenergic vasoconstriction, reducing flow to well-ventilated portions of lung.113 Overdistention of alveoli may also reduce perfusion to well-ventilated lung areas by creating the "zone 1" ventilation-perfusion scenario. For these reasons, normal fluid volume should be maintained during OLV; moderate VTs (6 mL/kg) should be used, and excessive PEEP should be avoided.89 Hypocapnia and alkalosis decrease HPV. - Many vasodilatory drugs inhibit HPV, including nitroglycerin, nitroprusside, dobutamine, some calcium channel blockers (e.g., nifedipine nicardipine, and verapamil), and some β2-agonists (such as isoproterenol).89 Vasoconstrictive drugs, including dopamine, epinephrine, and phenylephrine, may preferentially constrict normally oxygenated pulmonary vessels and reestablish the shunt flow, opposing the effects of HPV.111

Indications for a Right-Sided Double-Lumen Tube include the following except A. External or intraluminal tumor compression B. Ascending thoracic aortic aneurysm C. Left lung transplantation D. Right-sided tracheobronchial disruption

B & D Indications for a Right-Sided Double-Lumen Tube∗ • Distorted anatomy of the entrance of left mainstem bronchus • External or intraluminal tumor compression • Descending thoracic aortic aneurysm • Site of surgery involving the left mainstem bronchus • Left lung transplantation • Left-sided tracheobronchial disruption • Left-sided pneumonectomy† • Left-sided sleeve resectionon

Which of the following is not a proper venitlation parameter for one-lung ventilation A. TV 5-6 ml/lg B. PEEP 10 cm H2O C. RR 12 bpm D. Volume control

B.

There is an _________ correlation between perioperative mortality and surgical volume, and the cure rate of esophageal cancer with esophagectomy is between 10% and 50% A. Direct B. Inverse

B. Anesthetic Management for Specific Surgical Procedures Esophageal Surgery - Esophageal surgery is performed for both malignant and benign disease and may be curative or palliative. General 1985considerations that apply to almost all esophageal surgery patients include an increased risk of aspiration caused by esophageal dysfunction and the possibility of malnutrition. Esophagectomy - Esophagectomy is a palliative and potentially curative treatment for esophageal cancer and may occasionally be required for some benign obstructive lesions that do not respond to conservative therapy. It is a major surgical procedure and is associated with high morbidity and mortality rates (10%-15%). There is an inverse correlation between perioperative mortality and surgical volume, and the cure rate of esophageal cancer with esophagectomy is between 10% and 50%. There are multiple surgical procedures for esophageal cancer (Table 66-11) that combine some or all of three fundamental approaches: (1) a transthoracic approach, (2) a transhiatal approach, and (3) minimally invasive surgery (laparoscopic/thoracoscopic or robotic esophagectomy).197 The incidence of respiratory complications has been reported to be between 18% and 26% for both the transthoracic and transhiatal esophagectomy approaches.198 One study showed that the development of acute respiratory distress syndrome occurred in 14.5% of patients and acute lung injury occurred in 24%.199 Complications associated with the gastroesophageal anastomosis are: anastomotic leakage/dehiscence (5%-26%) and stenosis (12%-40%). Outcomes are improved with a multimodal anesthetic management protocol using fluid restriction, early extubation, thoracic epidural analgesia, and vasopressor/inotrope infusions to support blood pressure.200 Hypotension decreases the blood flow to the esophagogastric anastomosis. The use of vasopressors or inotropes, in normovolemic patients, restores the systemic pressure and the anastomotic blood flow.201 Fluid management for esophageal surgery is essentially the same as for pulmonary resection surgery.

When ventilation to the nondependent lung is ceased, ventilation is directed to the dependent lung. The remaining perfusion to the nondependent lung creates a shunt, but hypoxic pulmonary vasoconstriction reduces the shunt by __% by diverting much of that blood toward the dependent lung. A. 25 B. 50 C. 75 D. 100

B. Anesthetized Open-Chest, With One-Lung Ventilation - The succeeding cascade of changes leading up to the anesthetized open-chest results in significant ventilation-perfusion mismatch. There is little resistance to ventilation of the nondependent lung, whereas the effects of gravity promote perfusion to the dependent lung. When ventilation to the nondependent lung is ceased, ventilation is directed to the dependent lung. The remaining perfusion to the nondependent lung creates a shunt, but hypoxic pulmonary vasoconstriction reduces the shunt by 50% by diverting much of that blood toward the dependent lung. The PaO2 is higher during OLV in the lateral position than it is in the supine position, as clamping the airway to the nondependent lung reverses some of the changes that cause the ventilation-perfusion (V/Q) disparity in the anesthetized patient.

Which of the following dysrhythmia has the highest incidence in patients undergoing thoracic surgery A. Vtach B. Afib C. Aflutter D. Vfib

B. Arrhythmia - Dysrhythmias (see Chapter 47) are a common complication of pulmonary resection surgery, and it occurs is 30% to 50% of patients within the first week postoperatively, 1946when Holter monitoring is used.18 Of these arrhythmias, 60% to 70% are atrial fibrillation. Several factors correlate with an increased incidence of arrhythmias, including extent of lung resection (pneumonectomy 60% versus lobectomy 40% versus nonresection thoracotomy 30%), intrapericardial dissection, intraoperative blood loss, and age of the patient. Extrapleural pneumonectomy patients are a particularly high-risk group.19 - Two factors in the early postthoracotomy period interact to produce atrial arrythmias: (1) increased flow resistance through the pulmonary vascular bed because of permanent (lung resection) or transient (atelectasis, hypoxemia) causes, with attendant strain on the right side of the heart; and (2) increased sympathetic stimuli and oxygen requirements, which are maximal on the second postoperative day as patients become more mobile. - In some patients undergoing a pneumonectomy, the right heart may not be able to increase its output adequately to meet the usual postoperative stress. Transthoracic echocardiographic studies have shown that pneumonectomy patients develop an increase in right ventricular systolic pressure as measured by the tricuspid regurgitation jet on postoperative day 2 but not on postoperative day 1. An increase in tricuspid regurgitation jet velocity is associated with postthoracotomy supraventricular tachyarrhythmias.20 Preoperative exercise testing, which assesses the cardiopulmonary interaction, can predict postthoracotomy arrhythmias. Patients with COPD are more resistant to pharmacologic-induced heart rate control when they develop postthoracotomy atrial fibrillation and often require multiple drugs.21 - Many antiarrhythmics are used to decrease the incidence of atrial arrhythmias after lung surgery, of which digoxin preparations are the most common. Digoxin does not prevent arrhythmias after pneumonectomy or other intrathoracic procedures. Other antiarrhythmics that have been used to prevent postthoracotomy arrhythmias include β-adrenergic blockers, verapamil, and amiodarone. Although all of these drugs decrease arrhythmias in thoracic patients, their side effects preclude their widespread application in this surgical population. At present, diltiazem is the most useful drug for postthoracotomy arrhythmia prophylaxis.19 It seems that atrial arrhythmias are a sign of a dysfunctional right heart, for which prevention does not solve the underlying problem.

The incidence of developing second primary lung tumors is estimated at __% per year. A. 1 B. 2 C. 3 D. 4

B. Assessment for Repeat Thoracic Surgery - Patients who survive lung cancer surgery form a high-risk cohort to have a recurrence of the original tumor or to develop a second primary tumor. The incidence of developing second primary lung tumors is estimated at 2% per year. The use of routine screening low-dose spiral CT scans probably increases the rate of early detection.64 Patients who present for repeat thoracotomy should be assessed using the same framework as those who present for surgery the first time. Predicted values for postoperative respiratory function based on the preoperative lung mechanics, parenchymal function, exercise tolerance, and the amount of functioning lung tissue resected should be calculated and used to identify patients at increased risk.

To detect auto-PEEP, the respiratory circuit must be held closed during A. Beginning of a normal expiration B. End of a normal expiration C. Beginning of a normal inspiration D. End of a normal expiration

B. Auto-PEEP - Auto-PEEP occurs in patients with decreased lung elastic recoil, such as in older patients or those with emphysema.134 Auto-PEEP increases as the inspiratory/expiratory (I:E) ratio increases (i.e., as the time of expiration decreases). This auto-PEEP, which averages 4 to 6 cm H2O in most series of lung cancer patients studied, opposes the previously mentioned factors which tend to diminish dependent-lung FRC during OLV. The effects of applying external PEEP through the ventilator circuit to the lung in the presence of auto-PEEP are complex (Fig. 66-30). Patients with a very low auto-PEEP (<2 cm H2O) will have a greater increase in total PEEP from a moderate (5 cm H2O) external PEEP than those with a high level of auto PEEP (>10 cm H2O). Whether the application of PEEP during OLV will improve a patient's gas exchange depends on the individual's lung mechanics. If the application of PEEP tends to shift the expiratory equilibration position on the compliance curve toward the lower inflection point of the curve (i.e., toward the FRC), then external PEEP is of benefit (Fig. 66-31). However, if the application of PEEP raises the equilibration point such that it is further from the lower inflection point, then gas exchange deteriorates. - Auto-PEEP is difficult to detect and measure using currently available anesthetic ventilators. To detect auto-PEEP, the respiratory circuit must be held closed at the end of a normal expiration until an equilibrium appears in the airway pressure.135 Most current intensive care ventilators can be used to measure auto-PEEP.

In an awake lateral position, volume and perfusion is the greatest in the A. Independent B. Dependent

B. Awake Lateral Position - In the lateral position, less vertical distance is present to cause differences in the intrapleural pressure and blood pressure gradients (Fig. 30.4). Abdominal contents displace the diaphragm in a cephalad direction on the dependent side. Starting from a higher position in the thorax, the dependent hemidiaphragm can contract further. During inspiration, therefore, contraction of the diaphragm causes more of the VT to fill the dependent lung. Perfusion is dependent upon gravity and, when in the lateral position, it is also greatest in the dependent lung (Fig. 30.5). Overall, the relationship of greater ventilation and perfusion in the dependent lung is unchanged, and gas exchange remains efficient.

The safest method of obtaining lung isolation is A. Glidescope awake B. Fiberoptic awake C. ETT asleep D. Nasal RAE asleep

B. Bronchopleural Fistula - A bronchopleural fistula may be caused by (1) rupture of a lung abscess, bronchus, bulla, cyst, or parenchymal tissue into the pleural space; (2) the erosion of a bronchus by carcinoma or chronic inflammatory disease; or (3) stump dehiscence of a bronchial suture line after pulmonary resection. Pneumonectomy patients have an incidence of bronchopleural fistula ranging from 2% to 11%,218 with a mortality ranging from 5% to 70%. - The diagnosis of bronchopleural fistula is usually made clinically. After pneumonectomy, the diagnosis is based on the sudden onset of dyspnea, subcutaneous emphysema, contralateral deviation of the trachea, and a decrease of fluid level on serial radiographs of the chest (Fig. 66-47). In lobectomy patients, persistent air leak, purulent drainage, and expectoration of purulent material are usually diagnostic indicators of a bronchopleural fistula. When the fistula appears after removal of a chest tube, the diagnosis of bronchopleural fistula is made on the basis of fever, purulent sputum, and a new air-fluid level in the pleural cavity on the chest radiograph. The diagnosis is confirmed by bronchoscopic examination. In addition, bronchography and sinograms of the fistula may be used to confirm the diagnosis. Other diagnostic methods include the injection of an indicator, such as methylene blue, into the pleural space and subsequent recovery of the indicator from sputum. 1990Accumulation of radionuclide in the pleural space after inhalation of xenon or a mixture of O2 and N2O to detect the presence of a bronchopleural fistula can also be used as indicators.219 - If the disruption occurs early in postpneumonectomy patients it can be life threatening and it is possible to resuture the stump. Late or chronic postpneumonectomy bronchial disruption is managed with drainage or with the Clagett procedure, which includes open pleural drainage and the use of a muscle flap to reinforce the bronchial stump. In nonpneumonectomy cases, if the lung expands to fill the thoracic cavity, the air leak can usually be controlled with chest tube drainage alone. However, if the fistula is large and a significant leak through a large persistent pleural space occurs, it is unlikely that the fistula will close, and surgical resection is necessary Anesthetic Management - The patient with a bronchopleural fistula presents several intraoperative challenges for the anesthesiologist, including: (1) the need for lung isolation to protect healthy lung regions, (2) the possibility of tension pneumothorax with positive-pressure ventilation, and (3) the possibility of inadequate ventilation caused by air-leak from the fistula. - Preoperatively, it is useful to estimate the loss of tidal volume through the bronchopleural fistula, which may be done in two ways. First, one should determine whether air bubbles flow intermittently or continuously through the chest tube. If air bubbles flow intermittently, then the fistula is small. In contrast, when a patient has a large bronchopleural fistula or bronchial rupture, air will bubble continuously through the water-seal chamber of the chest tube drainage system. Second, the size of the bronchopleural fistula may be quantified by the difference between inhaled and exhaled tidal volumes. In a nonintubated patient, this may be determined with a tight-fitting mask and a fast-responding spirometer. In an intubated patient, it is determined by direct attachment of the spirometer to the endotracheal tube. The larger the air leak, the greater the need to isolate the bronchopleural fistula with the use of a lung isolation device (a DLT or an independent bronchial blocker). - Several nonsurgical approaches (i.e., the use of various mechanical ventilation-chest tube drainage systems) have been used for the treatment of patients with a bronchopleural fistula. These approaches consist of OLV and differential lung ventilation, including HFV, PEEP to the pleural cavity equal to intrathoracic PEEP, and unidirectional chest tube values. One-way endobronchial valves have been used successfully in patients with bronchopleural fistula who were considered unfit for surgery.220 - For patients undergoing operative repair, the ability to adequately deliver positive-pressure ventilation intraoperatively must be carefully considered before surgery. A chest drain should be placed before induction to avoid the possibility of tension pneumothorax with positive-pressure ventilation. A DLT is the best choice for delivering positive-pressure ventilation. The DLT can provide positive-pressure ventilation to the normal lung without loss of minute ventilation through the fistula and prevent the hazard of contamination of the uninfected lung with infected material when the patient is turned to the lateral decubitus position. - The safest method of obtaining lung isolation is awake fiberoptic intubation with a DLT.221 However, this requires a cooperative patient and excellent topical anesthesia and is often not an option. Another option is to maintain spontaneous ventilation during induction and intubation until lung isolation is secured. This avoids the possibility of inadequate positive-pressure ventilation because of air-leak but is not well tolerated in older patients with significant comorbidity. If the patient has a postpneumonectomy fistula, the DLT or SLT must be guided under direct vision with the assistance fiberoptic bronchoscopy, and the tip of the endobronchial lumen should be placed in the noninvolved lung (i.e., for right-sided fistula, use a left-sided tube). Whatever anesthetic technique is used, the principle of anesthetic management for a bronchopleural fistula is: lung isolation before positive-pressure ventilation or repositioning the patient. - One option to avoid instrumentation of the airway in patients with bronchopleural fistula after pneumonectomy is the use of thoracic epidural anesthesia with intravenous sedation during minimally invasive surgery.222 An alternative method of ventilating patients with multiple bronchopleural fistulas is the use of high-frequency oscillatory ventilation with permissive hypercapnia. This avoids barotrauma to the nonoperative lung, decreases bronchopleural fistula air-leak, and optimizes the operative outcome.223 The advantage of high-frequency oscillatory ventilation over conventional mechanical ventilation is that it uses lower peak airway pressure and 1991higher minimum airway pressure and may decrease the air-leak across the fistula. Early extubation in the operating room should be considered in all patients undergoing fistula repair to avoid barotrauma to the surgical stump from positive-pressure ventilation in the postoperative period.

This are thin-walled, air-filled intraparenchymal lung spaces caused by the loss of alveolar structural tissue A. Bleb B. Bullae C. Cysts D. Pneumatocele

B. Bullae - Bullae are thin-walled, air-filled intraparenchymal lung spaces caused by the loss of alveolar structural tissue (Fig. 66-48). These are usually associated with emphysema but their exact cause is unclear. Although there is some confusion over terminology in this area, bullous-like lesions of the lung associated with congenital malformations or secondary to trauma or infection are more correctly termed pneumatoceles or cysts. There are no universally accepted surgical indications for resection of lung bullae. A patient with symptomatic dyspnea and a giant bulla (or bullae) that occupies more than 30% of a hemithorax and in whom radiography and CT scans suggest that reasonably functional lung tissue can be restored to a more anatomically favorable position should be considered for bullectomy. Factors that support the effect of the bullae as a cause of a patient's dyspnea are a restrictive pattern of spirometry (proportional decrease of both FEV1 and FVC) and a discrepancy in lung volume studies in which the FRC measured by plethysmography exceeds the FRC measured by helium dilution by more than 2 liters. In the usual tidal volume range, bullae are more compliant than normal lung and fill preferentially during spontaneous ventilation. However, beyond the normal tidal volume range, bullae become much less compliant and the intrabulla pressure rises acutely as airway pressure increases. Measurement of in vivo intrabullae pressures in patients using fine needles both before and during anesthesia showed no evidence of a valve mechanism.37 All bullae studied communicated with the central airways, although some did this very slowly. The typical compression pattern seen on radiographs or CT scans is most likely caused by secondary elastic recoil of normal lung regions (see Fig. 66-6). Intrabulla pressure at FRC corresponds with the mean airway pressure averaged over the respiratory cycle. Thus during spontaneous ventilation, the intrabulla pressure will be negative with respect to the surrounding lung tissue. However, whenever positive pressure is used, the intrabulla pressure will rise in relation to surrounding lung regions. There is a risk of hyperinflation and rupture whenever positive pressure is used. The complications of bulla rupture can be life threatening because of hemodynamic collapse from tension pneumothorax or inadequate ventilation owing to resultant bronchopleural fistula. - Relief of dyspnea symptoms and improved pulmonary function are well documented in patients after resection of giant bullae, with most patients showing an increase in FEV1 of more than 0.3 L and excellent short-term improvement in quality of life, but with a decrease in improvement by 3 years.224 Hypercapnia is not a contraindication for bulla resection. Lung infection must be meticulously treated preoperatively. Outcome depends on patient age, smoking history, and cardiac status. In the postoperative period, lung air-leaks are the major complication. - The surgical approach can be by traditional sternotomy or by VATS. Laser resection may decrease the incidence of air-leaks. Various nonsurgical thoracoscopic 1992and bronchoscopic procedures such as the subsegmental injections of fibrin glue have been used to deal with these air-leaks. - The anesthetic considerations for bullectomy are similar to those for a patient with a bronchopleural fistula, with the exception that it is best to not place a chest drain prophylactically because this may enter the bulla and create a fistula, and there is not the risk of soiling healthy lung regions from extrapleural fluid that there is with fistulas. For induction of anesthesia, it is optimal to maintain spontaneous ventilation until the lung or lobe with the bulla or bleb is isolated.225 When there is a risk of aspiration or it is felt that the patient's gas exchange or hemodynamics may not permit spontaneous ventilation for induction, the anesthesiologist will have to use small tidal volumes and low airway pressures during positive-pressure ventilation until the airway is isolated.

As the patient is turned to the lateral position, the PetCO2 of the nondependent lung will ___________ to the dependent lung, reflecting increased perfusion of the dependent lung and increased dead space of the nondependent lung. A. increase B. decrease

B. Capnometry - The end-tidal CO2 (PetCO2) is a less reliable indicator of the PaCO2 during OLV than during TLV, and the PaCO2-PetCO2 gradient tends to increase during OLV. Although the PetCO2 is less directly correlated with alveolar minute ventilation during OLV, because the PetCO2 also reflects lung perfusion and cardiac output, it gives an indication of the relative changes in perfusion of the two lungs independently during position changes and during OLV.65 As the patient is turned to the lateral position, the PetCO2 of the nondependent lung will fall relative to the dependent lung, reflecting increased perfusion of the dependent lung and increased dead space of the nondependent lung. However, the fractional excretion of CO2 will be higher from the nondependent lung in most patients because of the increased fractional ventilation of this lung. At the onset of OLV, the PetCO2 of the dependent lung will usually fall transiently as all of the minute ventilation is transferred to this lung. The PetCO2 will then rise as the fractional perfusion is increased to this dependent lung by collapse and pulmonary vasoconstriction of the nonventilated lung. If there is no correction of minute ventilation, the net result will be both an increased baseline PaCO2 and PetCO2 with an increased gradient. Severe (>5 mm Hg) or prolonged decreases in PetCO2 can indicate a maldistribution of perfusion between ventilated and nonventilated lungs and may be an early warning sign of a patient who will subsequently desaturate during OLV.a.

The most useful predictor of difficult endobronchial intubation is A. CT scan B. Chest Xray C. MRI D. Bronchoscopy

B. Difficult Endobronchial Intubation - The most useful predictor of difficult endobronchial intubation is the plain chest radiograph (Fig. 66-7). - The anesthesiologist should view the chest imaging before induction of anesthesia because neither the radiologist's nor the surgeon's report of the imaging is made with the specific consideration of lung isolation in mind. Distal airway problems not detectable on the plain chest film can sometimes be visualized on the computed tomography (CT) scan: a side-to-side compression of the distal trachea, the so-called saber-sheath trachea 1953can cause obstruction of the tracheal lumen of a left-sided DLT during ventilation of the dependent lung for a left thoracotomy.59 Similarly, extrinsic compression or intraluminal obstruction of a mainstem bronchus that can interfere with endobronchial tube placement may only be evident on the CT scan. The major factors in successful lower airway management are anticipation and preparation based on the preoperative assessment. Management of lung isolation in patients with difficult upper and lower airways is discussed later in this chapter. Final Preanesthetic Assessment for Thoracic Surgery 1. Review initial assessment and test results 2. Assess difficulty of lung isolation: examine chest radiograph and computed tomographic scan 3. Assess risk of hypoxemia during one-lung ventilation

DLCO less than __% of the predicted value has been associated with increased complications following pulmonary surgery. A. 20 B. 40 C. 60 D. 80

B. Diffusion Capacity - Diffusion capacity tests the lungs' ability to allow transport of gas across the alveolar-capillary membrane. It is difficult to measure the diffusing capacity of oxygen, and thus carbon monoxide (CO) is used. The patient inhales a small amount of CO, holds the breath for 10 seconds, exhales, and the amount of CO in the exhaled breath is measured. After subtracting the amount of CO that should be expired with dead space air, the amount exhaled provides an indicator of the diffusion of gases in the lung. A DLCO less than 40% of the predicted value has been associated with increased complications following pulmonary surgery. However, DLCO has been found to have good specificity but low sensitivity as an independent measurement. The product of the predicted values for DLCO and FEV1 may provide better reliability than single measures. This measurement, called the predicted postoperative product, was found by Pierce to be less than 1650 in 75% of those who died.64

The blue cuff in a Double Lumen Endobronchial Tubes corresponds to A. Tracheal lumen B. Bronchial lumen

B. Double Lumen Endobronchial Tubes - DLTs consist of a single tube with two lumens. The bronchial lumen is identified by its blue cuff and is designed to be inserted into either the left or right mainstem bronchus. The bronchial lumen will be used to initially ventilate the corresponding lung. The tracheal lumen is positioned midtrachea, and the corresponding port will ventilate the opposite lung. The bronchial lumen does not necessarily have to be placed within the bronchus of the operative lung

In the blind technique: the DLT is passed by direct laryngoscopy and then turned 90 degrees __________ (for a left-sided DLT placement) after the endobronchial cuff has passed beyond the vocal cords. A. Clockwise B. Counterclockwise

B. Double-Lumen Endotracheal Tubes - The design of the Carlens DLT for lung surgery in 1950 was a landmark in the development of thoracic anesthesia because it allowed anesthesiologists to obtain reliable lung isolation in a majority of patients using only laryngoscopy and auscultation. However, the Carlens tube had a high flow resistance owing to the narrow lumina and the carinal hook was difficult to pass through the glottis in some patients. In the 1960s, Robertshaw introduced design modifications for separate left- and right-sided DLTs, removing the carinal hook and using larger lumina. In the 1980s, manufacturers introduced disposable DLTs made of polyvinyl chloride based on the design of the Robertshaw DLT. Among other subsequent improvements is the inclusion of radiographic markers near the endotracheal and endobronchial cuffs and a radiographic marker surrounding the ventilation slot for the right upper lobe bronchus for the right-sided DLT version. Bright blue, low-volume, low-pressure endobronchial cuffs are incorporated for easier visualization during fiberoptic bronchoscopy. Table 66-5 lists the sizes of DLTs, the 1956appropriate fiberoptic bronchoscope sizes to be used, and the comparable diameters of SLTs. Size Selection - A properly sized, left-sided DLT should have a bronchial tip 1 to 2 mm smaller than the patient's left bronchus diameter to allow for the space occupied by the deflated bronchial cuff. A study by Eberle and associates75 used a three-dimensional image reconstruction of tracheobronchial anatomy from the spiral CT scans combined with superimposed transparencies of DLTs to predict proper size for a right- or left-sided DLT. Chest radiographs and CT scans are valuable tools for selection of proper DLT size in addition to their proven value in assessment of abnormal tracheobronchial anatomy and should be reviewed before the placement of the DLT (Fig. 66-10). 1957A multidetector CT scan (MDCT) of the chest allows appreciation of any abnormal tracheobronchial anatomy before placement of a DLT. A simplified method of selection of DLT size is given in Table 66-6. It is important to appreciate that compared with SLTs, DLTs have a large external diameter (Fig. 66-11) and they should not be advanced against significant resistance. Methods of Insertion - Two techniques are commonly used when inserting and placing a DLT. One is the blind technique: the DLT is passed by direct laryngoscopy and then turned 90 degrees counterclockwise (for a left-sided DLT placement) after the endobronchial cuff has passed beyond the vocal cords. The DLT should pass the glottis without any resistance. Seymour76 showed that the mean diameter of the cricoid ring is approximately the same as that of the left mainstem bronchus. The optimal depth of insertion for a left-sided DLT is correlated with the patient's height in average-sized adults. In adults, depth, measured at the teeth for a properly positioned DLT, will be approximately 12 + (patient height/10) cm.77 In patients of Asian 1958descent, many of whom are of shorter stature (<155 cm), patient height is not a good predictor of depth of insertion of a DLT.78 An inadvertently deep insertion of a DLT can lead to serious complications, including rupture of the left mainstem bronchus. Figure 66-12 depicts the blind method technique for insertion of a left-sided DLT. - The direct vision technique uses bronchoscopic guidance, in which the tip of the endobronchial lumen is guided into the correct bronchus after the DLT passes the vocal cords using direct vision with a flexible fiberoptic bronchoscope. A study by Boucek and associates79 comparing the blind technique versus fiberoptic bronchoscopy-guided technique showed that of the 32 patients who underwent the blind technique approach, primary success occurred in 27 patients and eventual success occurred in 30 patients. In contrast, in the 27 patients using the bronchoscopy-guided technique, primary success was achieved only in 21 patients and eventual success in 25 patients. Although both methods resulted in successful left mainstem bronchus placement in all patients, more time was required when the fiberoptic bronchoscopy guidance technique was used (181 versus 88 seconds).

Signs of right ventricular hypertrophy in the ECG include the following except A. tall R wave in V1 (> 6 mm) B. ST segment depression C. R/S ratio greater than 1 in lead V1 D. ratio less than 1 in V6

B. Electrocardiogram - An electrocardiogram (ECG) is useful for assessing signs of right ventricular hypertrophy. In such a case, the ECG shows a tall R wave in V1 (> 6 mm), R/S ratio greater than 1 in lead V1, and a ratio less than 1 in V6, along with a right-axis deviation and diminished amplitude limb leads.33-35 Right atrial hypertrophy causes the initial component of a biphasic P wave in lead V1 to be larger than the second component. Strain characteristics such as ST segment depression and T-wave inversion, as well as incomplete or complete right bundle branch block, may be observed. The ECG has excellent specificity for identifying left ventricular hypertrophy (LVH), but less sensitivity for detecting it.36,37 Therefore, if ECG criteria are inconclusive, echocardiography may be helpful to further elucidate the status of the right ventricle. Findings of pathologic Q-waves and evidence of LVH preoperatively correlate with an increased incidence of postoperative ischemia and infarction.38 Echocardiogram Echocardiographic findings that are consistent with pulmonary disease are increased thickness of the right ventricular free wall, chamber enlargement, septal shift, tricuspic regurgitation, and a decreased right ventricular ejection fraction. In response to chronic hypoxia, hypoxic pulmonary vasoconstriction causes elevated pulmonary artery pressures. This results in increased right ventricular afterload, which can lead to right ventricular dysfunction. Increased PVR and right ventricular strain cause concern in patients undergoing pneumonectomy or extensive partial resection because of the added resistance produced by clamping the vasculature of one lung in the surgical procedure. Echocardiography is the best initial tool for assessing pulmonary hypertension, but additional studies with pulmonary angiography, ventilation-perfusion scintigraphy, computed tomography (CT), and magnetic resonance imaging (MRI) may also be used for more in-depth evaluation.39

This is a therapeutic option for selected patients with malignant pleural mesothelioma. A. Lobectomy B. Extrapleural pneumonectomy C. Sleeve pneumonectomy D. Sleeve lobectomy

B. Extrapleural Pneumonectomy - Extrapleural pneumonectomy is a therapeutic option for selected patients with malignant pleural mesothelioma.193 1984Significant improvement in survival has been achieved in patients who have an advanced malignant pleural mesothelioma with extrapleural pneumonectomy and high-dose radiotherapy in the postoperative period. Extrapleural pneumonectomy involves an extensive resection that may include lymph nodes, pericardium, diaphragm, parietal pleura, and the chest wall. The anesthetic management of the extrapleural pneumonectomy patient is characterized by significant loss of blood caused by chest wall vessel involvement. In these patients, it is recommended that a CVP catheter be used to guide intravascular fluid administration and ensure large-bore IV access. During tumor dissection, venous return to the heart may be compromised owing to multiple factors including blood loss, compression effect by the tumor in superior vena cava, or surgical causes. If there is an excessive loss of blood, it must be replaced to maintain an acceptable hematocrit level and the coagulation profile kept within normal limits. Because of extensive tumor resection and the potential for a pericardial resection in right-sided surgery, cardiac herniation or hemodynamic instability can appear postoperatively after the patient is turned from the lateral decubitus to the supine position. It is common to ventilate these patients for a short period postoperatively because of the extended duration of the surgery and the large fluid shifts. If a DLT is used intraoperatively, the DLT is usually replaced at the end of the case with an SLT.

If hypoxia is encountered during one lung ventilation, CPAP to the ______________ lung is almost 100% efficacious in increasing PaO2. A. Dependent B. Nondependent

B. Hypoxemia During One-Lung Ventilation - Hypoxemia occurs in 5% to 10% of patients under OLV.3 Although hypoxic pulmonary vasoconstriction attempts to normalize the relationship between ventilation and perfusion, it is not 100% effective at doing so. Certain characteristics predict the degree of hypoxemia that will be exhibited by a patient under OLV. Anatomically, the right lung is larger than the left lung, and thus, proportionally, there is a greater amount of perfusion to the right lung. This is the reason that there is an increased incidence of hypoxemia during right-sided surgery (see Fig. 30.15).151 The usual detrimental effect of lateral positioning on oxygenation paradoxically benefits the patient during OLV. Lateral positioning with mechanical ventilation normally imbalances V/Q matching by distributing more ventilation to the nondependent lung, while gravity encourages more perfusion to the dependent lung. During lateral-positioned OLV, direction of all ventilation to the dependent lung creates a more beneficial match of ventilation and perfusion. In fact, PaO2 is found to be significantly worse in procedures (such as lung transplant) where OLV is performed in supine patients.116 Some patient data also predict the degree of hypoxemia that will be encountered during OLV. The reduction in FEV1, paradoxically, is sometimes inverse to the degree of hypoxemia experienced. As a measure of disease progress, a lower FEV1 indicates worse disease; but as a measure of air-trapping, patients with a lower FEV1 sometimes develop intrinsic or auto-PEEP that helps keep their airways patent and beneficially reduces hypoxemia during OLV.151 Ventilation-perfusion scanning, if completed preoperatively, can help the anesthetist to predict the potential for intraoperative shunt under OLV, because perfusion to the operative lung is inverse to the potential shunt.152 A last-minute predictor is the end-tidal CO2 (ETCO2). Being dependent on blood flow to the lung, ETCO2 is a surrogate measure of perfusion.153 The degree of decline in ETCO2 when switching from two-lung ventilation to OLV indicates the degree of blood perfusing the nondependent lung, and therefore a greater initial decline predicts inferior oxygenation during OLV. Alveolar recruitment maneuvers and the use of subsequent PEEP at 8 cm H20 improves arterial oxygenation during OLV.154 - Should hypoxemia occur during OLV, the anesthetist should assess for physiologic causes or tube malpositioning. Physiologic causes include bronchospasm, decreased cardiac output, hypoventilation, low FiO2, or pneumothorax of the dependent lung. Tube malpositioning implies that movement of the DLT may have excluded a portion of dependent lung, usually the upper lobe. Assessing the position of the DLT should be the initial intervention, as a large proportion of hypoxemic episodes are remedied by tube repositioning.3 If physiologic causes have been ruled out, and adequate lung separation and ventilation have been determined, one or more of the following interventions will help improve PaO2. First, continuous positive airway pressure (CPAP) to the nondependent, nonventilated lung is almost 100% efficacious in increasing PaO2. This can be accomplished with a compact breathing system, such as a Mapleson C with a manometer for pressure determination, attached to the lumen of the deflated lung (Fig. 30.17), or with a calibrated, adjustable device made specifically for this purpose. Application of CPAP should help to oxygenate the persistent blood flow through the nondependent lung, but too much pressure will cause the lung to inflate, reducing surgical exposure. The lowest level of effective CPAP should be sought (start at 2 cm H2O). The reservoir bag on the CPAP device can also be used to provide intermittent ventilation to the operative lung, should that intervention become necessary. Providing gentle ventilation with a separate system will minimize the diminution of surgical exposure, as opposed to ventilating the lung with the same vigor as that required for the dependent lung. As an alternative to CPAP, a small catheter can be used to deliver low-flow oxygen insufflation to the nondependent lung without generating pressure. This approach may be adequate to reduce the shunt and reverse hypoxemia in mild cases.155

HPV is thought to be able to decrease the blood flow to the nonventilated lung by __% A. 25 B. 50 C. 75 D. 100

B. Hypoxic Pulmonary Vasoconstriction - HPV is thought to be able to decrease the blood flow to the nonventilated lung by 50%.117 The stimulus for HPV is primarily the alveolar oxygen tension (PAO2), which stimulates precapillary vasoconstriction, redistributing pulmonary blood flow away from hypoxemic lung regions via a pathway involving nitric oxide and/or cyclooxygenase synthesis inhibition.118 The mixed venous PO2 (PVO2) is also a stimulus, although considerably weaker than PAO2. HPV has a biphasic temporal response to alveolar hypoxia. The rapid-onset phase begins immediately and reaches a plateau by 20 to 30 minutes. The second (delayed) phase begins after 40 minutes and plateaus after 2 hours (Fig. 66-28).119 The offset of HPV is also biphasic, and pulmonary vascular resistance may not return to baseline for several hours after a prolonged period of OLV. This may contribute to increased desaturation during the collapse of the second lung during bilateral thoracic procedures. HPV is also a reflex that has a preconditioning effect, and the response to a second hypoxic challenge will be greater than to the first challenge.120 - The surgical trauma to the lung can affect pulmonary blood flow redistribution. Surgery may oppose HPV by release of vasoactive metabolites locally in the lung. Conversely, surgery can dramatically decrease blood flow to the nonventilated lung by deliberately or accidentally mechanically interfering with either the unilateral pulmonary arterial or venous blood flow.121 Ventilation increases blood flow through a hypoxic lung more than in a normoxic lung, which is generally not of clinical relevance but does complicate studies of HPV. HPV is decreased by vasodilators such as nitroglycerin and nitroprusside. In general, vasodilators can be expected to cause deterioration in PaO2 during OLV. Thoracic epidural sympathetic blockade probably has little or no direct effect on HPV, which is a localized chemical response in the lung.122 However, thoracic epidural anesthesia can have an indirect effect on oxygenation during OLV if it is allowed to cause hypotension and a fall in cardiac output (see Cardiac Output later in this chapter).

The left lung is __% smaller than the right lung, there is less shunt when the left lung is collapsed A. 5 B. 10 C. 15 D. 20

B. Prediction of Desaturation During One-Lung Ventilation - In the vast majority of cases, it is possible to determine the patients who are most at risk of desaturation during OLV for thoracic surgery. The factors that correlate with desaturation during OLV are listed in Box 66-4. In patients at high-risk of desaturation, prophylactic measures can be used during OLV to decrease this risk. The most useful prophylactic measures are the use of CPAP (2 to 5 cm H2O of oxygen) to the nonventilated lung or PEEP to the dependent lung (see Treatment of Hypoxemia During OLV) (also see Chapter 19 regarding more detailed physiology.). - The most important predictor of PaO2 during OLV is the PaO2 during two-lung ventilation (TLV), specifically the intraoperative PaO2 during TLV in the lateral position before OLV.60 The proportion of perfusion or ventilation to the nonoperated lung on preoperative scans also correlates with the PaO2 during OLV.61 If the operative lung has little perfusion preoperatively because of unilateral disease, the patient is unlikely to desaturate during OLV. The side of the thoracotomy has an effect on PaO2 during OLV. Because the left lung is 10% smaller than the right lung, there is less shunt when the left lung is collapsed. In a series of patients, the mean PaO2 during left thoracotomy was approximately 70 mm Hg higher than during right thoracotomy.62 Finally, the degree of obstructive lung disease correlates in an inverse fashion with PaO2 during OLV. Other factors being equal, patients with more severe airflow limitation on preoperative spirometry tend to have a better PaO2 during OLV than patients with normal spirometry (this is discussed later in Anesthetic Management). Factors That Correlate with an Increased Risk of Desaturation During One-Lung Ventilation 1. High percentage of ventilation or perfusion to the operative lung on preoperative scan 2. Poor PaO2 during two-lung ventilation, particularly in the lateral position intraoperatively 3. Right-sided thoracotomy 4. Normal preoperative spirometry (FEV1 or FVC) or restrictive lung disease 5. Supine position during one-lung ventilation

A patient with a PaO2 of ____ mm Hg is prone to desaturate during OLV A. 100 B. 200 C. 300 D. 400

B. Intraoperative Monitoring (See Chapters 44 and 51) - A few points specific to intraoperative monitoring of the thoracic surgical patient should be emphasized. The majority of these operations are major procedures of moderate duration (2 to 4 hours) and are performed with the patient in the lateral position and the hemithorax open. Thus consideration for monitoring and maintenance of body temperature and fluid volume should be given to all of these cases. Because surgery is usually performed with the patient in the lateral position, monitors are initially placed with the patient in the supine position and have to be rechecked and repositioned after the patient is turned. It is difficult to add additional monitoring, particularly invasive vascular monitoring, after the case is started if complications arise. Thus, the risk-benefit ratio often tends to favor being overly invasive at the outset. Choice of monitoring should be guided by a knowledge of which complications are likely to occur (Table 66-3). Oxygenation - Significant desaturation (SpO2 <90%) during OLV occurs in 1% to 10% of the surgical population despite a high 1954FiO2 (1.0) (see Management of OLV later in the chapter). Pulse oximetry (SpO2) has not negated the need for direct measurement of arterial PaO2 via intermittent blood gases in the majority of thoracotomy patients. The PaO2 value offers a more useful estimate of the margin of safety above desaturation than the SpO2. A patient with a TLV PaO2 greater than 400 mm Hg with an FiO2 of 1.0 (or an equivalent PaO2/FiO2 ratio) is unlikely to desaturate during OLV, whereas a patient with a PaO2 of 200 mm Hg is prone to desaturate during OLV, although both may have SpO2 values of 99% to 100%.

General anesthesia can therefore be very dangerous in patients with tumors in the A. Thyroid B. Anterior mediastinum C. Thoracic cavity D. Abdominal cavity

B. Mediastinal Masses - Masses in the mediastinum can compress vital structures and cause changes in cardiac output, obstruct airflow, induce atelectasis, or produce central nervous system changes. Masses can include benign or cancerous tumors, thymomas, substernal thyroid masses, vascular aneurysms, lymphomas, teratomas, and neuromas (Box 30.9). Surgical procedures for diagnosis or treatment of these masses may include thoracotomy, thoracoscopy, and mediastinoscopy. - Tumors within the anterior mediastinum can cause compression of the trachea or bronchi, increasing resistance to airflow. Changes in airway dynamics with supine positioning, induction of anesthesia, and positive-pressure ventilation can cause collapse of the airway with total obstruction to flow. General anesthesia can therefore be very dangerous in these patients. Total airway obstruction can occur at any phase of anesthesia and through the recovery phase. Positive-pressure ventilation may be impossible, even with a properly placed ETT, if the mass encroaches on the airway distal to the ETT. The airway can collapse, even when spontaneous respiration is maintained.165 To anticipate this potential, anesthetic preparation should include the availability of a rigid bronchoscope and readiness to turn the patient lateral or prone in case of airway collapse. Cannulation for potential emergency femoral-femoral bypass should be considered if the tumor is large or if the patient becomes symptomatic.166,167 Localization of the mass by CT or bronchoscopy may facilitate placement of the ETT distal to the mass. A major anesthetic goal is to maintain spontaneous ventilation, which retains normal airway-distending pressure gradients and can maintain airway patency when positive-pressure ventilation will not.168 - Signs and symptoms associated with respiratory tract compression should be determined preoperatively. Many patients with mediastinal masses are asymptomatic or characterized by vague signs such as dyspnea, cough, hoarseness, or chest pain. Wheezing may represent airflow past a mechanical obstruction rather than bronchospasm. Symptoms may be positional, worsening in the supine position or other positions. A chest radiograph may show airway compression or deviation. CT, transesophageal echocardiography, and MRI may further delineate the size and effects of masses. Subclinical airway obstruction may be revealed by flow-volume loops, which demonstrate changes in flow rates at different lung volumes. A decreased maximal inspiratory or expiratory flow rate alerts the anesthetist to an increased risk of obstruction perioperatively. Comparison of flow rates obtained with the patient in the upright and supine positions can reveal whether the supine position will exacerbate the obstruction intraoperatively (Fig. 30.18 and Box 30.10).169 Common Tumors of the Anterior Mediastinum: "The 4 Ts" • Thymoma • Thyroid • Teratoma • "Terrible" lymphoma

During mediastonoscopy, The mediastinoscope can place pressure on the innominate (brachiocephalic) artery; therefore, Monitoring perfusion to the ________ arm with a pulse oximeter or radial artery catheter can detect innominate artery compression A. Left B. Right

B. Mediastinoscopy - Mediastinoscopy involves insertion of a scope into the mediastinum via an incision made above the sternal notch. The scope is passed anterior to the trachea in close proximity to the left common carotid artery, the left subclavian artery, the innominate artery, the innominate veins, the vagus nerve, the left recurrent laryngeal nerve, the thoracic duct, the superior vena cava, and the aortic arch. - Complications associated with mediastinoscopy include hemorrhage resulting from disruption of major vessels; pneumothorax; dysrhythmias; bronchospasm; left recurrent laryngeal nerve palsy; laceration of the trachea or esophagus; and chylothorax secondary to laceration of the thoracic duct.174 Large-bore IV access should be in place, and banked blood should be immediately available in the event of a tear in a major blood vessel. Air embolism is also a risk if a venous tear occurs. Dysrhythmias such as bradycardia are possible with manipulation of the aorta or trachea during blunt dissection. - The mediastinoscope can place pressure on the innominate (brachiocephalic) artery prior to its division into the right common carotid artery and right subclavian artery. This can cause decreased blood flow to the right common carotid artery and right vertebral artery, and decreased right subclavian blood flow to the right arm (Fig. 30.19).117 The decrease in cerebral flow can cause an acute ischemic stroke, especially if the patient has a history of cerebrovascular disease. Monitoring perfusion to the right arm with a pulse oximeter or radial artery catheter can detect decreased flow to the right arm and signal concurrent loss of flow to the brain via innominate artery compression. Repositioning of the mediastinoscope is required to reestablish flow to the brain. A noninvasive blood pressure cuff placed on the left arm enables continued monitoring of systemic blood pressure during periods of innominate artery compression.

In medistinoscopy, it is mandatory to monitor the pulse in the _____ arm A. Left B. Right

B. Mediastinoscopy - Mediastinoscopy is the standard method for the evaluation of mediastinal lymph nodes in the staging of NSCLC. In addition, mediastinoscopy is used to aid in the diagnosis of anterior/superior mediastinal masses.169 The most common mediastinal diagnostic procedure is a cervical mediastinoscopy, in which a small transverse incision (2 to 3 cm) is made in the midline of the lower neck in the suprasternal notch. The pretracheal fascial plane is dissected bluntly and the mediastinoscope inserted toward the carina. An alternative procedure is a parasternal (or anterior) mediastinoscopy with a small incision made through the interchondral space or the space of the excised second costal cartilage. - Morbidity related to mediastinoscopy ranges from 2% to 8%. The most severe complication of mediastinoscopy is major hemorrhage, which may require emergent thoracotomy. Other potential complications include airway obstruction, compression of the innominate artery, pneumothorax, paresis of the recurrent laryngeal, phrenic nerve injury, esophageal injury, chylothorax, and air embolism.170 Anesthetic Management - For patients undergoing cervical mediastinoscopy, the chest radiograph and CT scan should be reviewed for the presence of a mass that might obstruct the airway during the preoperative evaluation. It is possible to perform mediastinoscopy (particularly anterior mediastinoscopy) with local anesthesia. This may be an option with an anterior mediastinal mass in a cooperative adult with a compromised airway. However, patient coughing or movement could result in surgical complications. The majority of these patients require general anesthesia with placement of an SLT. An arterial line is not necessarily used in these cases. However, it is mandatory to monitor the pulse in the right arm (pulse oximeter on the right hand, arterial line, or anesthesiologist's finger) because compression of the innominate artery by the mediastinoscope may occur and the surgeon usually is not aware that this is happening. The innominate artery supplies blood not only to the right arm but also to the right common carotid. Patients who do not have good cerebral collateral circulation (it is generally not possible to predict who these patients are) are at risk for cerebrovascular ischemia with innominate compression. A noninvasive blood pressure cuff is placed on the left arm to confirm the correct systolic pressure in case of suspected innominate compression. - Mild mediastinal hemorrhage may respond to conservative measures: the patient can be placed in the head-up position, the systolic pressure kept in the 90s, and tamponading the wound with surgical sponges. However, massive hemorrhage requires an emergent sternotomy or thoracotomy to stop the bleeding (Box 66-10). A bronchial blocker can be passed through the lumen of the existing lung isolation device if lung isolation is required because it is often difficult to change to a DLT while the surgeon is tamponading the wound. An arterial line should be placed (if it was not placed previously) to measure arterial blood pressure. If hemorrhage originates from a tear in the superior vena cava, volume replacement and drug treatment may be lost into the surgical field unless they are administered through a peripheral IV catheter placed in the lower extremity. - Pneumothorax is an infrequent complication of mediastinoscopy. Pneumothorax that occurs intraoperatively (as evidenced by increased peak inspiratory pressure, tracheal shift, distant breath sounds, hypotension, and cyanosis) requires immediate treatment by chest tube decompression. All patients must have a chest radiograph taken in the postanesthesia care unit after mediastinoscopy to rule out pneumothorax. - When mediastinoscopy causes injury to the recurrent laryngeal nerve, it can be permanent in approximately 50% of the cases. If injury to the recurrent laryngeal nerve is suspected, the vocal cords should be visualized while the patient is spontaneously breathing. If the vocal cords do not move or are in a midline position, consideration has to be given to the problem of postoperative laryngeal obstruction. - During mediastinoscopy, the tip of the mediastinoscope is located intrathoracically and therefore it is directly exposed to pleural pressure. A venous air embolus can occur if venous bleeding occurs and patients are breathing spontaneously because of the development of negative intrathoracic pressure during inspiration. Autonomic reflexes may result from compression or stretching of the trachea, vagus nerve, or great vessels. With an uncomplicated mediastinoscopy, the patient can be extubated in the operating room and discharged home the same day.

Most research indicates that smoking cessation for less than __ weeks prior to surgery does not alter risk of complications at all. A. 2 B. 4 C. 6 D. 8

B. Patient Optimization - Aggressive treatment of acute or reversible components of respiratory disease greatly decreases the risk of postoperative complications. Treatable preoperative conditions include infections, excess bronchial secretions, bronchospasm, dehydration, electrolyte imbalance, cigarette smoking, alcohol abuse, and malnutrition. - Smoking is not only a major risk factor for chronic lung disease, but is also a strong predictor of perioperative complications. Among patients undergoing noncardiac surgery, pulmonary complications occurred in 22% of smokers, 13% of past smokers, and only 5% of nonsmokers.69 In a study of lung cancer patients, 87% were smokers, and the smokers had a 1.5% rate of mortality, whereas nonsmokers had only a 0.4% rate of mortality. Smokers in that study demonstrated twice the rate of complications as did nonsmokers.70 The correlation of complications with the duration of smoking history can be calculated using the pack-year index (the product of the packs per day smoked times the years of smoking at that rate). Patients with greater than a 20 pack-year history have demonstrated increased incidence of complications compared with those who have a more modest smoking history.71,72 - Smoking cessation may reduce postoperative complications; however, the timing of this intervention is important. Most research indicates that smoking cessation for less than 4 weeks prior to surgery does not alter risk of complications at all.69,73,74 Some data suggest that short-term smoking cessation (less than 1 month) may cause increases in mucus production, which may actually increase complications69,75; however, research findings are equivocal on the concept of increasing complications.74,75 Carboxyhemoglobin levels have shown a significant decrease rapidly after smoking cessation, and improvement of nasal mucociliary clearance occurs within 1 to 2 weeks; however, the implications of these findings for postoperative complications are unclear.76,77 The rates of complications are reduced in proportion to the amount of time after quitting, with a threshold of at least 4 weeks to observe improvement and even more improvement noted after 8 weeks.73,74 Only in one very large study was a trend toward slight reduction in complications noticed with less than 1 month of smoking cessation, and complications further decreased in proportion to the total duration of cessation.70 Smoking cessation counseling and intervention is a widely recommended strategy for medical management of COPD patients; however, owing to the urgent nature of treating pulmonary carcinoma, delaying surgery to allow for an adequate period of smoking cessation is an impractical goal.

During two-lung ventilation, blood flow to the dependent lung averages approximately __% A. 20 B. 40 C. 60 D. 80

B. Physiology of One-Lung Ventilation - During two-lung ventilation, blood flow to the dependent lung averages approximately 60% (Fig. 30.15). When one lung is allowed to deflate and OLV is started, any blood flow to the deflated lung becomes shunt flow, causing the PaO2 to decrease. Without autoregulation of pulmonary blood flow, a 40% shunt would be anticipated. The lungs have a compensatory mechanism of increasing vascular resistance in hypoxic areas of the lungs, and this diverts some blood flow to areas of better ventilation and oxygenation. This mechanism, which is present in most mammals, is termed hypoxic pulmonary vasoconstriction (HPV). HPV is a reflex intrapulmonary feedback mechanism in inhomogeneous lungs that improves gas exchange and arterial oxygenation. Whereas hypoxemia causes vasodilation in the general circulation, alveolar hypoxia has the opposite effect on pulmonary arteries. HPV is a unique compensatory mechanism, suited specifically to matching pulmonary blood flow with well-oxygenated areas of lung. - The cellular mechanism for HPV involves a redox-based oxygen sensor in smooth muscle cells of the pulmonary arteries (probably focused on the electron transport chain of the mitochondria of these cells). Hypoxia reduces production of activated oxygen species (AOS) such as H2O2. These AOS act as second messengers from the oxygen sensors, and reduction in their outflow leads to inhibition of voltage-dependent potassium channels. The result is an influx of extracellular calcium, which causes vasoconstriction.107,108 Box 30.6 lists the characteristics of HPV. - HPV during OLV is effective in decreasing the cardiac output to the nonventilated lung by approximately 50% (Fig. 30.16).109 HPV can increase the PVR by 50% to 300%, and the response can persist for long periods of time in the face of chronic hypoxia.107 In fact, the chronic increase in PVR from COPD can be responsible for pulmonary vascular remodeling that leads to cor pulmonale.110 HPV occurs whether the lung is rendered hypoxic by atelectasis or by ventilation with a hypoxic mixture. It is initiated within seconds of hypoxia and reaches its maximum effect in approximately 15 minutes. HPV improves arterial oxygenation when the amount of hypoxic lung is between 20% and 80%, which occurs during OLV.111 When less than 20% of the lung is hypoxic, the total amount of shunt is not significant. When more than 80% of the lung is hypoxic, HPV increases PVR, but the amount of well-perfused lung is not sufficient to accept enough diverted flow to maintain arterial oxygenation.

This can reduce opioid consumption more than 30% after thoracotomy and are particularly useful in treating the ipsilateral shoulder pain that is often present postoperatively and is poorly controlled with epidural analgesia. A. Opoid B. NSAID C. Ketamine D. Dexmedetomidine

B. Postoperative Analgesia (See Chapter 98) - Studies before 1990 consistently report a 15% to 20% rate of major respiratory complications (i.e., atelectasis, pneumonia, respiratory failure) within the first 3 days after thoracic surgery.1 This time of onset may relate to the unique pattern of recovery of pulmonary function after thoracotomy, which shows a delay in the initial 72-hour postoperative period that is not seen with other major surgical incisions.269 The incidence of postthoracotomy respiratory complications has shown an overall decline to less than 10%, whereas the cardiac complication rate has not changed.2 Improvements in postoperative care, specifically pain management, are the major cause of this decline. There are multiple sensory afferents that transmit nociceptive stimuli after thoracotomy (Fig. 66-52). These include the incision (intercostal nerves T4-T6), chest drains (intercostal nerves T7-T8), mediastinal pleura (vagus nerve, CN X), central diaphragmatic pleura (phrenic nerve, C3-C5),270 and ipsilateral shoulder (brachial plexus). There is no one analgesic technique that can block all of these various pain afferents, so analgesia should be multimodal. The optimal choice for an individual patient depends on patient factors (i.e., contraindications, preferences), surgical factors (i.e., type of incision), and system factors (i.e., available equipment, monitoring, nursing support). The ideal postthoracotomy analgesic technique will include three classes of drugs: opioids, antiinflammatory agents, and local anesthetics. Systemic Analgesia Opioids - Systemic opioids alone are effective in controlling background pain, but the acute pain component associated with cough or movement requires plasma levels that produce sedation and hypoventilation in most patients. Even when administered by patient-controlled devices, pain control is generally poor271 and patients have interrupted sleep patterns when serum opioid levels fall below the therapeutic range. Nonsteroidal Antiinflammatory Drugs - NSAIDs can reduce opioid consumption more than 30% after thoracotomy and are particularly useful in treating the ipsilateral shoulder pain that is often present postoperatively and is poorly controlled with epidural analgesia. NSAIDs act through reversible inhibition of cyclooxygenase (COX), which has antiinflammatory and analgesic effects but can also be associated with decreased platelet function, gastric erosions, increased bronchial reactivity, and decreased renal function. Acetaminophen is an antipyretic/analgesic with weak COX inhibition and can be administered orally or rectally in doses of up to 4 g/day. It is effective against shoulder pain and has a low toxicity compared with more potent COX-inhibiting NSAIDs.272 2001 Ketamine - Low-dose intramuscular ketamine (1 mg/kg) is equivalent to the same dose of meperidine and causes less respiratory depression. Ketamine can also be administered as a low-dose intravenous infusion and may be useful in patients who are refractory to other therapies or if there is a contraindication to more common techniques.273 The possibility of psychomimetic effects with ketamine is always a concern but is rarely seen with analgesic, subanesthetic doses. Dexmedetomidine - Dexmedetomidine, a selective adrenergic α2-receptor agonist, has been reported as a useful adjunct for postthoracotomy analgesia and can significantly decrease the requirement for opioids when used in combination with epidural local anesthetics. Maintenance infusion doses for analgesia in children and adults are in the range of 0.3 to 0.4 μg/kg/h.274 It is associated with some hypotension, but it seems to preserve renal function.

Massive hemoptysis is defined as expectoration of more than ___ mL of blood in 24 to 48 hours. A. 100 B. 200 C. 300 D. 400

B. Pulmonary Hemorrhage - Massive hemoptysis is defined as expectoration of more than 200 mL of blood in 24 to 48 hours. The commonest causes are carcinoma, bronchiectasis, and trauma (blunt, penetrating, or secondary to a pulmonary artery catheter). Death can occur quickly as a result of asphyxia. Management requires four sequential steps: lung isolation, resuscitation, diagnosis, and definitive treatment. The anesthesiologist is often called to deal with these cases outside of the operating room. There is no consensus on the best method of lung isolation for these cases. The initial method for lung isolation will depend on the availability of appropriate equipment and an assessment of the patient's airway. All three basic methods of lung isolation have been used: DLTs, SLTs, and bronchial blockers. Fiberoptic bronchoscopy is usually not helpful to position endobronchial tubes or blockers in the presence of torrential pulmonary hemorrhage, and lung isolation must be guided by clinical signs (primarily auscultation). DLTs will achieve rapid and secure lung isolation. Even if a left-sided tube enters the right mainstem bronchus, only the right upper lobe will be obstructed. However, suctioning large amounts of blood or clots is difficult through the narrow lumina of a DLT. An option is initial placement of the SLT for oxygenation and suctioning and then replacement with a DLT either by laryngoscopy or with an appropriate tube exchanger. An uncut single-lumen ETT can be advanced directly into the right mainstem bronchus or rotated 90 degrees counterclockwise for advancement into the left mainstem bronchus.237 A bronchial blocker will normally pass easily into the right mainstem bronchus and is useful for right-sided hemorrhage (90% of pulmonary artery catheter-induced hemorrhages are right-sided). Except for cases with blunt or penetrating trauma, after lung isolation and resuscitation 1995have been achieved, diagnosis and definitive therapy of massive hemoptysis are now most commonly performed by coiling of the pulmonary artery false aneurysm in interventional radiology.238

This has traditionally been considered the technique of choice for the preoperative diagnostic assessment of an airway obstruction involving the trachea and in the therapy of massive hemoptysis and foreign bodies in the airway. A. Fiberoptic bronchoscopy B. Rigid bronchoscopy C. Mediastinoscopy D. EBUS

B. Rigid Bronchoscopy - Rigid bronchoscopy has traditionally been considered the technique of choice for the preoperative diagnostic assessment of an airway obstruction involving the trachea and in the therapy of massive hemoptysis and foreign bodies in the airway. The role of interventional bronchoscopy with laser, bronchial dilation, or stent insertion is well established for the treatment of malignant and benign central airway and endobronchial lesions (Fig. 66-38).164 Rigid bronchoscopy is the procedure of choice for operative procedures such as dilation of tracheal stenosis. Anesthetic Management Patients undergoing rigid bronchoscopy should have a complete preoperative evaluation including radiologic studies. Chest radiographs and chest CT scans should be reviewed in the preoperative evaluation. If time permits, it is recommended that patients with severe stridor receive pharmacologic interventions for temporary stabilization of the condition. Treatments may include inspired cool saline mist, nebulized racemic epinephrine, and the use of systemic steroids.165 There are four basic methods of ventilation management for rigid bronchoscopy: 1. Spontaneous ventilation. The addition of topical anesthesia or nerve blocks to the airway decreases the tendency to breath-hold and cough when volatile anesthetics are used. 2. Apneic oxygenation (with/without insufflation of oxygen). This requires thorough preoxygenation, and the anesthesiologist will have to interrupt surgery to ventilate the patient before desaturation occurs. This should allow the surgeon working intervals of 3 minutes or longer depending on the underlying condition of the patient. 3. Positive-pressure ventilation via a ventilating bronchoscope (Fig. 66-39). This allows the use of a standard anesthetic circuit but may cause significant air leaks if there is a discrepancy between the size of a smaller bronchoscope and a larger airway. 4. Jet ventilation. This can be performed with a handheld injector such as the Sanders injector (Sulz, Germany)166 or with a high-frequency ventilator. These techniques 1978are most useful with intravenous anesthesia because they entrain gas from either the room air or an attached anesthetic circuit, and the dose of any volatile agent delivered will be very uncertain. - The use of anticholinergic agents (e.g., 0.2 mg intravenous [IV] glycopyrrolate ) before manipulation of the airway will decrease secretions during the bronchoscopic examination. For a patient undergoing rigid bronchoscopy, the surgeon must be at the bedside for the induction of anesthesia and be prepared to establish airway control with the rigid bronchoscope. Anesthesia in children for rigid bronchoscopy is most commonly done with spontaneous ventilation and a volatile anesthetic. In adults, IV anesthesia and the use of muscle relaxants is more common. In cases for which the use of neuromuscular blocking drugs is not contraindicated, succinylcholine can be used initially to facilitate intubation with either a small SLT or the rigid bronchoscope. Nondepolarizing neuromuscular blocking drugs (see Chapter 34) may be needed for prolonged procedures such as stent placement or tumor resection. Mouthguards should be used to protect the upper and lower teeth and gums from the pressure of the bronchoscope. Remifentanil and propofol infusions can be administered if an IV regimen is the planned anesthetic.167 This is a useful technique if the surgeon needs repeated access (for suction or instrumentation) to the open airway because it maintains the level of anesthesia and avoids contaminating the operating room with exhaled anesthetic vapors. For cases in which a neodymium-doped yttrium aluminium garnet (Nd:YAG) laser is used, the inspired fraction of oxygen should be maintained in the lowest acceptable range (i.e., <30% if possible) according to patient oxygen saturation, to avoid the potential for fire in the airway. Because any common material (including porcelain and metal) can be perforated by the Nd:YAG laser, it is best to avoid any potentially combustible substance in the airway when the Nd:YAG laser is used.168 Because of its high energy and short wavelength, the Nd:YAG laser has several advantages for distal airway surgery over the CO2 laser, which is used in upper airway surgery. The Nd:YAG laser penetrates tissue more deeply so it causes more coagulation in vascular tumors, and it can be refracted and passed in fibers through a flexible or rigid bronchoscope. However, there is a higher potential for accidental reflected laser strikes and there is more delayed airway edema. - Rigid bronchoscopes have different sizes, commonly from 3.5- to 9-mm diameters, with a ventilating side port to facilitate ventilation when the bronchoscope is placed into the airway. If excessive leak of tidal volume occurs around the bronchoscope with positive-pressure ventilation, it may be necessary to place throat packs to facilitate ventilation. Continuous communication with the surgeon or pulmonologist is necessary in case desaturation occurs. If desaturation does occur, it must be corrected by stopping surgery and allowing the anesthesiologist to ventilate and oxygenate the patient, either via the rigid bronchoscope or by removing the bronchoscope and ventilating with a mask, LMA, or ETT. - Pulse oximetry is vital during rigid bronchoscopy because there is a high risk of desaturation. There is no simple way to monitor end-tidal CO2 or volatile anesthetics because the airway remains essentially open to atmosphere. For prolonged procedures, it is useful to perform repeated arterial blood gas analysis to confirm the adequacy of ventilation. An alternative is to interrupt surgery and ventilate the patient with a standard circuit and a mask or ETT to assess the end-tidal CO2. - Unlike during fiberoptic bronchoscopy via an ETT, with rigid the bronchoscopy the airway is never completely secure and there is always the potential for aspiration in patients at increased risk, such as those with a full stomach, hiatal hernia, morbid obesity, and so on. It is always best to defer rigid bronchoscopy to decrease the aspiration risk if possible in these patients. When there is no benefit to be gained by deferring and/or the airway risk is acute (e.g., aspiration of an obstructing foreign body), there is no simple solution and each case will need to be managed on an individual basis depending on the context and weighing the competing risks. Other uses of the rigid bronchoscope that require anesthesia include dilation for benign airway stenosis, coring-out of malignant lesions in the trachea, laser ablation of endobronchial and carinal tumors, and therapeutic bronchoscopic interventions before surgical resection of lung cancer. In addition, interventional bronchoscopy is often used for the management of airway complications after lung transplantation. - Complications of rigid bronchoscopy include airway perforation, mucosal damage, hemorrhage, postmanipulation airway edema, and potential airway loss at the end of the procedure. In some situations, it may be necessary to keep the patient intubated with a small (i.e., 6.0-mm ID) SLT after a rigid bronchoscopy if an edematous airway is suspected or if the patient is not able to be extubated. These patients may require the use of steroids, nebulized racemic epinephrine, or helium-oxygen mixtures to treat stridor in the postoperative period

Pulmonary complications are decreased in thoracic surgical patients who cease smoking for more than __ weeks before surgery. A. 2 B. 4 C. 6 D. 8

B. Smoking - Pulmonary complications are decreased in thoracic surgical patients who cease smoking for more than 4 weeks before surgery.44 Carboxyhemoglobin concentrations decrease if smoking is stopped more than 12 hours.45 It is extremely important for patients to avoid smoking postoperatively. Smoking leads to a prolonged period of tissue hypoxemia. Wound tissue oxygen tension correlates with wound healing and resistance to infection. There is no rebound increase in pulmonary complications if patients stop for shorter (<8 weeks) periods before surgery.46 Intensive smoking-cessation interventions are the most successful.47

Thymoectomy may be done for which condition A. Multiple sclerosis B. Myasthenia gravis C. Eaton-Lambert Syndrome D. Duchenne Muscular dystrophy

B. Thymectomy for Myasthenia Gravis - Myasthenia gravis is a disease of the neuromuscular junction; affected patients have weakness caused by a decreased number of acetylcholine receptors at the motor 1999end plate.258 Patients may or may not have an associated thymoma. Thymectomy is frequently performed to induce clinical remission, even in the absence of a thymoma. The effects of muscle relaxants are modified by this disease; myasthenic patients are resistant to succinylcholine and are extremely sensitive to nondepolarizing blockers. Thymectomy may be performed via full or partial sternotomy, or with a minimally invasive approach via a transcervical incision or VATS. For thymectomy with a thymoma, a sternotomy is used. In the absence of an identifiable tumor, minimally invasive techniques are commonly used. Ideally, the use of neuromuscular relaxation is avoided. Induction of anesthesia with propofol, remifentanil, and topical anesthesia of the airway facilitates intubation without the use of muscle relaxants. Alternatively, inhalational induction with a halogenated agent such as sevoflurane may be performed.259 For sternotomy, combined general and thoracic epidural anesthesia is a useful technique. - Most patients take pyridostigmine, an oral anticholinesterase, and many patients take immunosuppressive medication (e.g., corticosteroids). On the day of surgery, pyridostigmine dosing should ensure that the patient's usual regimen is provided during the immediate perioperative period. A few patients require IV dosing with neostigmine until they are able to resume oral intake of pyridostigmine. A scoring system was devised for prediction of the need for prolonged mechanical ventilation after thymectomy via sternotomy.260 Among the criteria that contributed to the predicted need for support were disease duration longer than 6 years, chronic respiratory illness, pyridostigmine dosage greater than 750 mg/day, and vital capacity less than 2.9 L. The relevance of this score has diminished with better preoperative preparation and minimally invasive surgery.261 Referral for surgery early in the course of the disease and stabilization of symptoms with medication and plasmapheresis, combined with an increased use of minimally invasive approaches, have made the need for postoperative ventilation infrequent. In optimized patients, mean hospital length of stay can be reduced to 1 day after minimally invasive, and 3 days after transsternal thymectomy.262 Patients should remain on their full medical regimen postoperatively. The remission from myasthenia gravis after thymectomy occurs slowly over a period of months to years.

Overall, the effects of hyperventilation will usually tend to ____________ pulmonary vascular pressures. A. Increase B. Decrease

B. Ventilation Strategies During One-Lung Ventilation - The strategy used to manage the ventilated lung during OLV plays an important part in the distribution of pulmonary blood flow between the lungs. It has been the practice of many anesthesiologists to use the same large tidal volume (e.g., 10 mL/kg ideal body weight) during OLV as during TLV. This strategy probably decreases hypoxemia by recurrently recruiting atelectatic regions in the dependent lung and may result in higher PaO2 values during OLV compared with smaller 1971tidal volumes.130 However, there is a trend to use smaller tidal volumes with PEEP during OLV for several reasons. First, the incidence of hypoxemia during OLV is much lower than it was 20 to 30 years ago. Second, there is a risk of causing acute injury to the ventilated lung with prolonged use of large tidal volumes. And finally, third, a ventilation pattern that allows recurrent atelectasis and recruitment of lung parenchyma seems to be injurious.131 The ventilation technique needs to be individualized depending on the patient's underlying lung mechanics. Respiratory Acid-Base Status (See Chapter 60) - The efficacy of HPV in a hypoxic lung region is increased in the presence of respiratory acidosis and is inhibited by respiratory alkalosis. However, there is no net benefit to gas exchange during OLV from hypoventilation because the respiratory acidosis preferentially increases the pulmonary vascular tone of the well-oxygenated lung, and this opposes any clinically useful pulmonary blood flow redistribution.132 Overall, the effects of hyperventilation will usually tend to decrease pulmonary vascular pressures. - Positive End-Expiratory Pressure Resistance to blood flow through the lung is related to lung volume in a biphasic pattern and is lowest when the lung is at its FRC (see Fig. 66-26). Keeping the ventilated lung as close as possible to its normal FRC favorably encourages pulmonary blood flow to this lung. Several intraoperative factors known to alter FRC tend to cause the FRC of the ventilated lung to fall below its normal level; these include lateral position, paralysis, and opening the nondependent hemithorax, which allows the weight of the mediastinum to compress the dependent lung. Attempts to measure FRC in human patients during OLV have been complicated by the presence of a persistent end-expiratory airflow in patients with COPD.133 Many patients do not actually reach their end-expiratory equilibrium FRC lung volume as they try to exhale a relatively large tidal volume through one lumen of a DLT. These patients develop dynamic hyperinflation and an occult positive end-expiratory pressure (auto-PEEP).41

Lateral flexion of the neck can cause compression of the A. Subclavian vein B. Jugular Vein C. Vertebral artery D. Carotid artery

B. & C. Lateral Decubitus Position - The most frequent position chosen for surgical exposure during thoracotomy is the lateral decubitus position. A roll is placed beneath the torso just caudal to the axilla to prevent compression of the neurovascular bundle and forward rotation of the humeral head. It is important to note that, whereas this roll is commonly called an axillary roll, is better considered an axillary support roll, because positioning it in the axilla may cause neurovascular compression. Hyperabduction of the arms is prevented to keep the brachial plexus from stretching against the humeral head. Arms can be separately padded and extended forward on arm boards. Strategies for supporting the nondependent arm may include a pillow between the arms, a padded Mayo stand (which provides good access to intravenous (IV) or arterial lines in the dependent arm), or specially made double arm boards. Pulse oximetry or frequent palpation of the radial pulse ensures the integrity of circulation to the hand. - The head is supported on pillows to maintain alignment of the head and neck with the spine. Lateral flexion of the neck can cause compression of the jugular veins or vertebral arteries, compromising cerebral circulation. The dependent ear can be compressed by the weight of the head. Careful padding or use of a foam doughnut relieves this pressure, but care must be taken to prevent corneal abrasion and retinal ischemia by avoiding pressure on the eyes. Because the brachial plexus arises from the cervical vertebrae, stretching of one side of the neck can occur if the head is not maintained in a neutral position resulting in neuropathy. A complete discussion of positioning and potential nerve injuries is presented in Chapter 23. Another pressure point of concern with the lateral position is the region near the common peroneal nerve. These pressure points are located near the fibular head of the dependent leg and the femoral head of the nondependent leg if a stabilizing strap is placed over the patient.

Ventilation in an anesthetized, open-chest patient causes a A. Increase dead space B. Decrease dead space C. Increase shunt D. Decrease shunt

C Anesthetized, Open-Chest - Upon opening the thorax, there is an immediate decrease in resistance to gas flow in the nondependent lung, as the lung detaches from the chest wall. This causes further loss of ventilation to the dependent lung, in preference for the nondependent lung. The mediastinum also further shifts downward because of loss of negative intrapleural pressure in the nondependent lung, which helped to distend it. Ventilation to the dependent lung is decreased in proportion to the displacement of the lung by the mediastinal structures. Compression of the great vessels may cause a decrease in cardiac output and circulatory compromise. Any spontaneous respiration becomes very inefficient as paradoxical movement of air occurs on inspiration from the open-chest lung into the dependent lung, which has greater negative intrapleural pressure. Upon expiration, gas exits the dependent lung and enters both the trachea and the open-chest lung, causing the lung to expand (Fig. 30.8). Paradoxical respiration compromises fresh gas exchange in the dependent lung as part of the VT moves between the lungs. Positive-pressure ventilation diminishes the effects of mediastinal shift and paradoxical respiration. However, during mechanical ventilation, the open chest provides no resistance, and the greatly increased compliance of that lung allows a higher proportion of ventilation to go to the nondependent lung, which is the least perfused area of the thorax. The less ventilated, better-perfused, dependent lung contributes to physiologic shunt, as blood flows through atelectatic areas without acquiring oxygen. Although the prevalence of different zones is not as evenly distributed as diagrams would suggest, the lateral, anesthetized, paralyzed, open-chest patient does exhibit the epitome of significant regional areas of disparity between ventilation and perfusion.

This combination has been shown to improve oxygenation in ventilated intensive care unit patients with adult respiratory distress syndrome, and this may have applications in OLV. A. Almitrene B. L-NAME C. Phenylephrine D. NO

C & D Pharmacologic Manipulations - Eliminating known potent vasodilators such as nitroglycerin and halothane and large doses of other volatile anesthetics will improve oxygenation during OLV.150 The selective administration of the vasodilator prostaglandin E1 to the ventilated lung151 or a NO synthase inhibitor (L-NAME)152 to a hypoxic lobe results in improved redistribution of pulmonary blood flow in animal models. This is not currently applicable in humans. Selective administration of NO alone to the ventilated lung was not shown to be of benefit in humans.153 The combination of NO (20 ppm) to the nonventilated lung and an intravenous infusion of almitrene, which enhances HPV, has been shown to restore PaO2 values during OLV in humans to essentially the same levels as during TLV. However, this may have been primarily a result of the augmentation of HPV by almitrene.154 It is unlikely that almitrene, which was previously available in North America as a respiratory stimulant, will be reintroduced to this market because of side effects such as hepatic enzyme changes and lactic acidosis. However, the combination of NO and other pulmonary vasoconstrictors such as phenylephrine has been shown to improve oxygenation in ventilated intensive care unit patients with adult respiratory distress syndrome,155 and this may have applications in OLV. Intermittent Reinflation of the nonventilated Lung - HPV becomes more effective during repeated hypoxic exposure. Often after reinflation, the oxygen saturation will be more acceptable during a second period of lung collapse. Reexpansion can be performed by regular reexpansion of the operative lung via an attached CPAP circuit.

Which of the following is not an absolute indication for lung separation A. Infection B. Massive hemorrhage C. Upper lobectomy D. Severe hypoxemia related to unilateral lung disease

C & D. One-Lung Ventilation Indications for Lung Separation - The ability to provide distinct ventilation to the separate lungs facilitates pulmonary surgery by providing a quiet surgical field. This is particularly helpful in the case of thoracoscopic surgery, in which visualization and the ability to manipulate the operative lung are limited. Thoracic surgeons will commonly consider lung separation an absolute requirement for pulmonary surgery. However, surgery can be performed on a lung that is being ventilated, and thoracic surgery alone is not an absolute indication for OLV. In fact, in the case of pediatric patients, gentle dual-lung ventilation is indicated and appropriate in either the presence of airway concerns that preclude use of a lung-separating device or the inability to maintain oxygenation with OLV. Certain situations, such as infectious contamination of one lung, are absolute indications for OLV, but most common thoracic surgeries create relative indications for lung separation in that they can safely be accomplished without it. Indications for lung separation are noted in Box 30.3. Indications for Lung Separation I. Absolute A. Isolation of one lung from the other to avoid spillage or contamination 1. Infection 2. Massive hemorrhage B. Control of the distribution of ventilation 1. Bronchopleural fistula 2. Bronchopleural cutaneous fistula 3. Surgical opening of a major conducting airway 4. Giant unilateral lung cyst or bulla 5. Tracheobronchial tree disruption 6. Life-threatening hypoxemia related to unilateral lung disease C. Unilateral bronchopulmonary lavage 1. Pulmonary alveolar proteinosis II. Relative A. Surgical exposure—high priority 1. Thoracic aortic aneurysm 2. Pneumonectomy 3. Thoracoscopy 4. Upper lobectomy 5. Mediastinal exposure B. Surgical exposure—medium (lower) priority 1. Middle and lower lobectomies and subsegmental resections 2. Esophageal resection 3. Procedures on the thoracic spine C. Postcardiopulmonary bypass pulmonary edema/hemorrhage after removal of totally occluding unilateral chronic pulmonary emboli D. Severe hypoxemia related to unilateral lung disease

This involves the use of laparoscopic, thoracoscopic, or robotic surgical approaches perform esophagectomy A. Transthoracic B. Transhiatal C. Minimally Invasive Approach

C. Transhiatal Approach - Airway management is done with an SLT. Apart from this, anesthetic management is essentially the same as for a transthoracic approach. Of special concern is that the blunt/blind manual dissection of the thoracic esophagus by the surgeon through the hiatus during this approach is often associated with cardiac compression and sudden severe hypotension. In addition, this blind dissection can cause vascular or distal airway injuries if the tumor is adherent.203 It is good practice to not cut the endotracheal tube for this procedure in case of surgical perforation of the trachea or bronchus necessitating advancement of the endotracheal tube into a mainstem bronchus for emergent OLV Minimally Invasive Approach Minimally invasive esophagectomy involves the use of laparoscopic, thoracoscopic, or robotic surgical approaches. For a laparoscopic approach, distention of the peritoneum may produce hemodynamic changes because of the intragastric pressure generated by carbon dioxide insufflation. In these cases, it is important to adjust ventilatory parameters to achieve an optimal PaCO2. For the thoracoscopic approach, a left-sided DLT or a bronchial blocker is required. During robotic surgery, the use of a lung isolation device is required to achieve OLV. Special considerations for robotic surgery include protecting the patients against any injury related to the robot and not moving the operating room table while the robot is being used. The thoracoscopic-assisted esophagectomy has several advantages including less blood loss, less pain, and a shorter length of hospitalization. This method does, however, require a prolonged duration of surgery. - All patients undergoing esophagectomy require a nasogastric tube, which must be well-secured at the end of the operation. Respiratory complications, including the development of an acute lung injury, may be present after an esophagectomy. Intrathoracic anastomotic leakage is a feared major complication after esophageal surgery and carries a high mortality rate of 4% to 30%.204 To treat this potential complication, nasogastric decompression and nutritional support should be used. Severe leakage usually occurs in the early postoperative period as a consequence of gastric necrosis, and it may present with respiratory symptoms and signs of shock. Even though there is a very high mortality rate, prompt surgical intervention is recommended. Patients older than 80 years have an increased risk of mortality after esophagectomy, independent of comorbidity.205

Which of the following is an advantage of the bronchial blocker A. Lesser incidence of malpositioning B. Decrease time of insertion C. Indicated for difficult airway D. Better lung deflation

C. Advantages and Disadvantages - Because insertion of a DLT is more complicated, bronchial blockers are more useful in patients with a difficult airway or a tracheostomy. These devices are beneficial for patients who are already intubated, when changing to another tube would compromise ventilation. They can also be used for pediatric lung separation, even in children younger than 2 years of age, with a special small-sized blocker.104,105 - Positioning the bronchial blocker requires more time than the DLT, and placement is dependent on the use of a fiberscope. In comparison to DLT, blockers have a greater incidence of becoming malpositioned. Because a bronchial blocker affords little conduit for egress of gas, lung deflation is often less effective than with a DLT. This can be particularly troublesome in thoracoscopic surgery. Suction of the blocker port can be helpful, but excessive negative pressure or duration can cause damage. Blockers also do not allow suctioning, particularly when separation is indicated by unilateral infection or bleeding. In that case, removal must follow special procedures to avoid contaminating the opposite lung.106 The bronchial blocker is plagued with the same challenge as the DLT in blocking the right bronchus, but the DLT has the advantage of being able to exclude either lung with a left-sided device. Therefore, bronchial blockers are reserved by some exclusively for left-sided surgery. Insertion of Bronchial Blockers - The method used for inserting the bronchial blocker depends on the device. Typically, it involves basic tracheal intubation and insertion of the blocker through the tube, followed by the fiberscope, which is used for final positioning. Variations include wire-guided catheters, which are already mounted on the fiberscope when it is inserted, and pediatric extraluminal blockers, which are inserted in the trachea, followed by intubation, followed by fiberoptic inspection and positioning of the blocker.

HPV can be expected to remain intact if volatile agents are administered at less than __ of the minimum alveolar concentration. A. 0.5 B. 1 C. 1.5 D. 2

C. Anesthetic Management During One-Lung Ventilation Choice of Anesthetic - Side effects associated with general anesthesia stem from a variety of mechanisms and can negatively impact intraoperative and postoperative pulmonary function. They include impairment of HPV, disruption of V/Q matching, neural and pain-induced hypoventilation, postoperative residual muscle relaxation, and atelectasis. Few studies have been able to demonstrate clear differences in patient outcomes based on anesthetic technique alone. However, some factors have emerged as important in the pulmonary surgical patient. - Clinical doses of potent inhalation agents do not significantly alter the mechanism of HPV. Volatile agents offer several benefits in thoracic surgery. They allow the use of a high fraction of inspired oxygen (FiO2) to help prevent hypoxemia during OLV. They produce bronchodilatory effects and decrease airway irritability in patients who will be subjected to direct manipulation of lung tissue. In some studies, volatile agents resulted in a decreased inflammatory response compared to IV agents.114,115 In contrast, the dose of narcotics required to obtund airway reflexes could depress ventilation and necessitate postoperative ventilation. Volatile agents are rapidly eliminated at the end of surgery, and allow for early extubation. For these reasons, volatile agents are usually chosen as the primary anesthetics during thoracic surgery. - IV anesthetics do not inhibit HPV, and, whereas all volatile agents theoretically do, the volatiles act as vasodilators in a dose-dependent manner.116 HPV can be expected to remain intact if volatile agents are administered at less than 1.5 of the minimum alveolar concentration.117-119 Most analyses fail to demonstrate a difference in gas exchange between IV and inhaled techniques.120 Furthermore, there is no difference in perioperative morbidity and mortality whether inhalational anesthesia or total IV anesthesia is administered.121 - To prevent hypoxia and any significant increase in PVR, N2O is generally avoided in favor of an air-oxygen mixture. N2O increases PVR in healthy patients, as well as in those with preexisting pulmonary hypertension.87 This is cause for even greater concern with concurrent right ventricular dysfunction. Another indication to avoid N2O includes patients with bullous or emphysematous lungs. Because N2O is highly diffusible, increased air can be trapped within the lung. An air-oxygen mixture prolongs emptying of the operative lung, in contrast to pure oxygen or a N2O mixture.122 - The choice of specific opioids or hypnotics will not influence pulmonary outcomes, but there is concern related to the choice of muscle relaxants used. Postoperative residual neuromuscular blockade can occur with both intermediate and long-acting relaxants and is present regardless of the use of neuromuscular monitoring.123 However, the incidence of residual relaxation associated with the use of long-acting relaxants (e.g., pancuronium) is significantly higher, and complications from it are more frequent compared with the use of intermediate-acting relaxants.124 Muscle weakness may follow long surgeries, even with adequate tests of neuromuscular recovery.125 Therefore, the use of shorter-acting muscle relaxants and conservative monitoring, dosing, and reversal practices are indicated in the pulmonary surgical patient.

Paravertebral blocks have been used with a single dose of local anesthetic and have been shown to reduce pain after thoracoscopic surgery for __ hours A. 2 B. 4 C. 6 D. 8

C. Anesthetic Technique - Thoracoscopic surgery can be performed under local, regional, or general anesthesia with OLV or TLV.179 For minor diagnostic procedures, VATS can be done in the awake patient. Intercostal nerve blocks performed at the level of the incision and two interspaces above and below provide adequate analgesia. Partial collapse of the lung on the side of surgery occurs when air enters the pleural cavity. When using local anesthesia with the patient awake, it is hazardous to insufflate gases under pressure into the hemithorax in an attempt to increase visualization of the pleural space. Although many patients suffer from advanced pulmonary disease, changes in PaO2, PaCO2, and cardiac rhythm are usually minimal during the procedure when it is performed under local anesthesia and the patient is breathing spontaneously.180 However, it is recommended that a high FiO2 is delivered via a facemask to overcome the shunt because of the loss in lung volume caused by the unavoidable pneumothorax. - For most invasive procedures, VATS is performed under general anesthesia with a DLT or a bronchial blocker to achieve OLV. If the procedure is short in duration and the lung needs to be deflated for only a brief period, blood gases are not routinely monitored during the procedure. However, for patients undergoing prolonged VATS procedures such as lobectomy or for patients with marginal pulmonary status, an arterial line and measurement of arterial blood gases is required. Paravertebral blocks have been used with a single dose of local anesthetic and have been shown to reduce pain after thoracoscopic surgery for 6 hours.181 - Anesthetic complications are rare during this procedure, although it is possible that any structure that the surgeon has to manipulate may be damaged. The anesthesiologist needs to be aware of the potential for conversion to open thoracotomy if massive bleeding ensues or if the surgeon is unable to localize the lung nodule to be biopsied. The majority of thoracoscopic surgery requires placement of a chest tube postoperatively. It is important to have a functional chest tube with underwater seal drainage so that extubation can be performed safely.

This is a disease that causes localized, irreversible dilatation of part of the bronchial tree. A. lung abscess B. Empyema C. Bronchiectasis D. Pleural effusion

C. Bronchiectasis/Lung Abscess/Empyema - Bronchiectasis is a disease that causes localized, irreversible dilatation of part of the bronchial tree. Involved bronchi are inflamed and easily collapsible, resulting in airflow obstruction and impaired clearance of secretions. Bronchiectasis is associated with a wide range of disorders, but it usually results from necrotizing bacterial infections. Bronchiectasis may require surgery if it causes hemoptysis or recurrent pneumonia. An abscess is a nonanatomic area of liquefactive necrosis of the lung, often distal to an obstruction or after a bout of pneumonia (Fig. 66-46). An empyema is a collection of pus between the visceral and parietal pleural layers, often a complication of pneumonia or surgery. Empyema complicating lung resections occurs in 2% to 16% of cases and with a 40% increase in the associated perioperative mortality rate. Mortality further increases when the empyema is associated with a bronchopleural fistula. Surgical interventions for patients with empyema include: decortication (the method of choice when the underlying lung is unable to expand because of a thick inflammatory coat) or open-window thoracostomy (the ideal method for drainage of the pleural cavity to control septic symptoms in patients with postpulmonary resection empyema).217 In less severe cases, tube drainage, antibiotic irrigation, and debridement may be sufficient. - All of these infective indications for thoracic surgery are less common in the developed world since the introduction of antibiotics. Anesthetic considerations during surgery for these infective indications include the need for lung isolation to protect uninvolved lung regions from soiling by pus in the infected areas. The risk of soiling occurs if the patient is repositioned for surgery after induction of anesthesia before the lung is adequately isolated. Because of the inflammation, surgery is technically more difficult and there is a greater risk of massive hemorrhage. Anesthetic Management - Some of these patients may present with sepsis at the time of surgery. In these patients, the placement of a thoracic epidural catheter is not recommended. They require lung isolation, preferably with a DLT. The DLT facilitates suctioning of debris and copious secretions that are present in the tracheobronchial tree. Patients undergoing a decortication may have massive blood loss. If the lung has been chronically collapsed, expansion should be done gradually to avoid the development of pulmonary edema upon reexpansion. Extubation in the operating room is encouraged if the patient meets the standard criteria for extubation.

Which of the following is a Characteristics of Hypoxic Pulmonary Vasoconstriction A. Vasoconstriction in proximal pulmonary arteries B. Triggered by arterial hypoxemia C. Peak effect in 15 minutes D. Onset and resolution are within minutes following changes in partial pressure of oxygen (PO2)

C. Characteristics of Hypoxic Pulmonary Vasoconstriction • A local reaction occurring in hypoxic areas of lung. May be very localized due to regional atelectasis (as in one-lung ventilation), or affect both lungs entirely in hypoxic situations (such as those leading to high altitude pulmonary edema). • Opposite to systemic reaction to hypoxia, causes vasoconstriction in all but very proximal pulmonary arteries. • Triggered by alveolar hypoxia, not arterial hypoxemia. • Onset and resolution are within seconds following changes in partial pressure of oxygen (PO2). • Peak effect occurs within 15 minutes. • May be inhibited by vasodilators and augmented by chemoreceptor agonists (almitrine).

The most common complication associated with a DLT is A. Breaking of a carinal hook B. Damage to the vocal cords C. Malpositioning D. Barotrauma

C. Complications Associated With Double-Lumen Tubes The most common complication associated with a DLT is malpositioning. Fig. 30.10 demonstrates some variations of DLT positioning. Rupture of a thoracic aneurysm is possible with a left-sided DLT if the aneurysm compresses the left mainstem bronchus. Damage to the vocal cords or arytenoid cartilages is possible from a carinal hook. A carinal hook can also break off, requiring retrieval with a bronchoscope. Bronchial rupture, which was thought to be caused by overinflation of the bronchial cuff, has been reported.97,98 Owing to the possibility of its being inserted too deeply, a DLT can also cause the entire VT to be delivered to a single lung lobe, creating the potential for barotrauma. The larger size of the DLT is probably also responsible for the slightly increased incidence of hoarseness and vocal cord lesions observed in patients following DLT, versus using a bronchial blocker for lung separation.99

The F scale refers to ___ times the external diameter of the ETT (in millimeters). A. 1 B. 2 C. 3 D. 4

C. Features - Several types of DLTs are used in thoracic surgery. DLTs are designed for insertion either in the right or the left bronchus. Right-sided tubes include features to accommodate the proximity of the upper lobe bronchus. Disposable polyvinyl chloride tubes are available in French (F) sizes 26, 28, 35, 37, 39, and 41. The F scale refers to three times the external diameter of the ETT (in millimeters). The internal lumen diameters range from 3.4 mm to 6.6 mm,91 although there is wide size variation between manufacturers.92,93 It would be most important for the anesthetist to ascertain the appropriate size fiberscope that will fit through the intended tube lumens. Despite the perceived smaller size of the lumens, DLTs do not increase breathing resistance significantly in comparison to single lumen tubes.88,94

Which of the following is true regarding Fluid Management for Pulmonary Resection Surgery A. Total positive fluid balance in the first 24-hour perioperative period should not exceed 30 mL/kg. B. For an average adult patient, crystalloid administration should be limited to less than 5 L in the first 24 hours. C. No fluid administration for third-space fluid losses during pulmonary resection D. Urine output greater than 0.5 mL/kg/h is necessary

C. Fluid Management for Pulmonary Resection Surgery • Total positive fluid balance in the first 24-hour perioperative period should not exceed 20 mL/kg. • For an average adult patient, crystalloid administration should be limited to less than 3 L in the first 24 hours. • No fluid administration for third-space fluid losses during pulmonary resection • Urine output greater than 0.5 mL/kg/h is unnecessary. • If increased tissue perfusion is needed postoperatively, it is preferable to use invasive monitoring and inotropes rather than to cause fluid overload.

Which of the following is the grading scale for the statement below as far as Symptoms in Patients with an Anterior or Superior Mediastinal Mass: The most important diagnostic tool for the patient with a mediastinal mass is A. Asymptomatic B. Mild C. Moderate D. Severe

C. Grading Scale for Symptoms in Patients with an Anterior or Superior Mediastinal Mass • Asymptomatic • Mild: Can lie supine with some cough/pressure sensation • Moderate: Can lie supine for short periods but not indefinitely • Severe: Cannot tolerate supine position Stratification of Patients with Mediastinal Masses Regarding Safety for General Anesthesia • Safe: (I) Asymptomatic adult with CT minimal tracheal/bronchial diameter >50% of normal • Unsafe: (I) Severely symptomatic adult or child; (II) children with CT tracheal/bronchial diameter <50% of normal, regardless of symptoms • Uncertain: (I) Mild/moderate symptomatic child with CT tracheal/bronchial diameter >50% of normal; (II) mild/moderate symptomatic adult with CT tracheal/bronchial diameter <50% of normal; (III) adult or child unable to give history Management for All Patients with a Mediastinal Mass and an Uncertain Airway for General Anesthesia • Determine optimal positioning of patient preoperatively. • Secure airway beyond stenosis when patient is awake, if feasible. • Have rigid bronchoscope and surgeon available at induction of anesthesia. • Maintain spontaneous ventilation if possible (noli pontes ignii consumere). • Monitor for airway compromise postoperatively.

Which of the following is a bronchial blocker A. Bronchocath B. Carlens C. Coopdech D. White

C. Methods of Lung Separation - Several devices have been developed to enable isolation of one lung and ventilation of the other. The single-lumen endobronchial tube was developed in 1931 to isolate an infected lung.90 Mimicking this simple approach by advancing a 7.5-mm wide, 32-cm long ETT over a fiberoptic scope into one bronchus may still be used in some circumstances. A disadvantage of using a single-lumen tube for OLV is that the ability to ventilate or suction the operative lung is lost. Another disadvantage is that use of a single-lumen tube in the right lung would probably occlude the right upper lobe orifice. However, in an emergent situation, use of a single-lumen tube advanced blindly down the right bronchus or placed into the left bronchus aided by a bronchoscope can be lifesaving. A summary of lung separation devices is presented in Table 30.2. Given that each approach (DLT vs. bronchial blocker) has relative merits, neither can be recommended as a superior method of lung separation for thoracic surgery.

This is the site of the majority of intraoperative nerve injuries related to the lateral position A. Common peroneal B. Lumbar plexus C. Brachial plexus D. Sacral plexus

C. Neurovascular Complications - There are a specific set of nerve and vascular injuries related to the lateral position that must be appreciated. The brachial plexus is the site of the majority of intraoperative nerve injuries related to the lateral position.93 These are basically of two varieties: the majority are compression injuries of the brachial plexus of the dependent arm, but there is also a significant risk of stretch injuries to the brachial plexus of the nondependent arm. The brachial plexus is fixed at two points: proximally by the transverse process of the cervical vertebrae and distally by the axillary fascia. This two-point fixation, plus the extreme mobility of neighboring skeletal and muscular structures, makes the brachial plexus extremely liable to injury (Box 66-6). The patient should be positioned with padding under the dependent thorax (Fig. 66-21) to keep the weight of the upper body off the dependent arm brachial plexus. However, this padding will exacerbate the pressure on the brachial plexus if it migrates superiorly into the axilla. - The arms should not be abducted beyond 90 degrees and should not be extended posteriorly beyond the neutral position nor flexed anteriorly more than 90 degrees. Fortunately, the majority of these nerve injuries resolve spontaneously over a period of months. Anterior flexion of the arm at the shoulder (circumduction) across the chest or lateral flexion of the neck toward the opposite side can cause a traction injury of the suprascapular nerve.94 This causes a deep, poorly circumscribed pain of posterior and lateral aspects of the shoulder and may be responsible for some cases of postthoracotomy shoulder pain. - It is very easy, after repositioning the patient in the lateral decubitus position, to cause excessive lateral flexion of the cervical spine because of improper positioning of the patient's head. This malpositioning, which exacerbates brachial plexus traction, can cause a "whiplash" syndrome and can be difficult to appreciate from the head of the operating table, particularly after the surgical drapes have been placed. It is useful for the anesthesiologist to survey the patient from the side of the table immediately after turning to ensure that the entire vertebral column is aligned properly. - The dependent leg should be slightly flexed with padding under the knee to protect the peroneal nerve lateral to the proximal head of the fibula. The nondependent leg is placed in a neutral extended position and padding placed between it and the dependent leg. The dependent leg must be observed for vascular compression. Excessively tight strapping at the hip level can compress the sciatic nerve of the nondependent leg. Other sites particularly liable for neurovascular injury in the lateral position are the dependent ear pinna and eye. A head-to-toe protocol to monitor for possible neurovascular injuries related to the lateral decubitus position is presented in Box 66-7.

These is a potential space deep to the endothoracic fascia that the intercostal nerve traverses as it passes from the intervertebral foramen en route to the intercostal space A. Intrapleural B. Epidural C. Paravertebral D. Intercostal

C. Paravertebral Block - The paravertebral space is a potential space deep to the endothoracic fascia that the intercostal nerve traverses as it passes from the intervertebral foramen en route to the intercostal space (Fig. 66-54). A catheter can be placed in the thoracic paravertebral space either percutaneously or by approaching the space anteriorly and directly when the chest is open intraoperatively. - There is also a combined percutaneous/direct-vision method in which the tip of the Tuohy needle is advanced percutaneously into the paravertebral space under direct vision either during open thoracotomy or VATS. The tip of the needle is seen to enter the paravertebral space and the pleura is not punctured. Saline is injected via the Tuohy needle to hydrodissect the paravertebral space, and an epidural catheter is passed into the pocket that has been created in the paravertebral space and then secured at the skin. The use of ultrasonographic guidance has been a major advance for percutaneous paravertebral injections and catheter placement (Fig. 66-55).284 - Paravertebral local anesthetics provide a reliable multilevel intercostal blockade that tends to be unilateral, with a low tendency to spread to the epidural space. Clinically the analgesia is comparable with that from epidural local anesthetics.285 Studies comparing paravertebral versus thoracic epidural analgesia for thoracotomies have suggested the following advantages for paravertebral blockade286: comparable analgesia; fewer failed blocks; decreased risk of neuraxial hematoma; and less hypotension, nausea, or urinary retention. Because there is an option to place the paravertebral catheter under direct vision, this may contribute to the lower incidence of failed blocks versus thoracic epidural analgesia. - Paravertebral infusions in combination with NSAIDs and systemic opioids are a reasonable alternative to epidural techniques in children or patients with contraindications to neuraxial blockade.57 Using common therapeutic doses (e.g., 0.1 mL/kg/h of bupivacaine 0.5%), serum bupivacaine levels can approach toxic levels by 4 days.287 An alternative regime for paravertebral infusions is lidocaine 1% (1 mL/10 kg/h; maximum 7 mL/h). It has not yet been demonstrated whether paravertebral analgesia can contribute to a decrease in respiratory morbidity in high-risk cases, which has been shown for thoracic epidural analgesia.288

The majority of thoracic procedures are performed in which position A. Prone B. Supine C. Lateral D. Sitting

C. Positioning - The majority of thoracic procedures are performed in the lateral position, most often the lateral decubitus position, but depending on the surgical technique, a supine, semisupine, or semiprone lateral position may be used. These lateral positions have specific implications for the anesthesiologist. Position Change (See Chapter 41) - It is awkward to induce anesthesia with the patient in the lateral position. Thus monitors will be placed and anesthesia will usually be induced in the supine position and the anesthetized patient will then be repositioned for surgery. It is possible to induce anesthesia in the lateral position, and this may rarely be indicated with unilateral lung diseases such as bronchiectasis or hemoptysis until lung isolation can be achieved. However, even these patients will then have to be repositioned and the diseased lung turned to the nondependent side. - Because of the loss of venous vascular tone in the anesthetized patient, it is not uncommon to observe hypotension in the patient when turning them to or from the 1965lateral position. All lines and monitors will have to be secured during positioning changes and their function reassessed after repositioning. The anesthesiologist should take responsibility for the head, neck, and airway during position changes and must be in charge of the operating team to direct repositioning. It is useful to make an initial "head-to-toe" survey of the patient after induction and intubation to check oxygenation, ventilation, hemodynamics, lines, monitors, and potential nerve injuries. This survey then must be repeated after repositioning. It is nearly impossible to avoid some movement of a DLT or bronchial blocker during repositioning. Certainly the patient's head, neck, and endobronchial tube should be turned "en-bloc" with the patient's thoracolumbar spine. However, the margin of error in positioning endobronchial tubes or blockers is often so narrow that even very small movements can have significant clinical implications. The carina and mediastinum may shift independently with repositioning and this can lead to proximal misplacement of a previously well-positioned tube. Endobronchial tube/blocker position and the adequacy of ventilation must be rechecked by auscultation and fiberoptic bronchoscopy after patient repositioning.

These complications are the major cause of morbidity and mortality after thoracic surgery. A. Cardiac B. Renal C. Respiratory D. Vascular

C. Premedication - We do not routinely order preoperative sedation or analgesia for pulmonary resection patients. Mild sedation such as an intravenous short-acting benzodiazepine is often given immediately before placement of invasive monitoring lines and catheters. In patients with copious secretions, an antisialogogue (e.g., glycopyrrolate) is useful to facilitate fiberoptic bronchoscopy for positioning of a double-lumen endobronchial tube (DLT) or bronchial blocker. To avoid an intramuscular injection, this can be given orally or intravenously immediately after placement of the intravenous catheter. It is a common practice to use short-term intravenous antibacterial prophylaxis such as a cephalosporin in thoracic surgical patients. If it is the local practice to administer these drugs before admission to the operating room, they will have to be ordered preoperatively. Consideration for patients allergic to cephalosporins or penicillin should be made at the time of the initial preoperative visit. Summary of the Initial Preoperative Assessment - The anesthetic considerations that should be addressed at the time of the initial preoperative assessment are summarized in Box 66-2. Patients need to be specifically assessed for risk factors associated with respiratory complications, which are the major cause of morbidity and mortality afte thoracic surgery. Final Preoperative Assessment - The final preoperative anesthetic assessment for the majority of thoracic surgical patients is carried out immediately before admission of the patient to the operating room. At this time, it is important to review the data from the initial prethoracotomy assessment and the results of tests ordered at that time. In addition, two other specific areas affecting thoracic anesthesia need to be assessed: (1) the potential for difficult lung isolation and (2) the risk of desaturation during one-lung ventilation (OLV) (Box 66-3). Initial Preanesthetic Assessment for Thoracic Surgery 1. All patients: Assess exercise tolerance, estimate ppoFEV1%, discuss postoperative analgesia, discontinue smoking 2. Patients with ppoFEV1 < 40%: DLco, scan, VO2max 3. Cancer patients: consider the four Ms: mass effects, metabolic effects, metastases, medications 4. COPD patients: arterial blood gas, physiotherapy, bronchodilators 5. Increased renal risk: measure creatinine and blood urea nitrogen levels

This the most potent bronchodilator of the volatile anesthetics A. Desflurane B. Isoflurane C. Sevoflurane D. Nitrous oxide

C. Prevention of Bronchospasm - Because of the frequent incidence of coexisting reactive airways disease in the thoracic surgical population, an anesthetic technique that decreases bronchial irritability should be chosen. This is particularly important because the added airway manipulation caused by placement of a DLT or bronchial blocker is a potent trigger for bronchoconstriction. The principles of anesthetic management are the same as they are for any asthmatic patient: avoid manipulation of the airway in a lightly anesthetized patient, use bronchodilating anesthetics, and avoid drugs that release histamine. For intravenous induction of anesthesia, either propofol or ketamine can be expected to diminish bronchospasm. This benefit will not be seen with barbiturate, opioid, benzodiazepine, or etomidate intravenous induction of anesthesia. For maintenance of anesthesia, propofol and/or any of the volatile anesthetics will diminish bronchial reactivity. Sevoflurane may be the most potent bronchodilator of the volatile anesthetics.106 Coronary Artery Disease (See Chapter 47) - Because the lung resection population is largely elderly and/or smokers, there is a high coincidence of coronary artery disease. This consideration will be a major factor in the choice of the anesthetic technique for most thoracic patients. The anesthetic technique should optimize the myocardial oxygen supply/demand ratio by maintaining arterial oxygenation and diastolic blood pressure while avoiding unnecessary increases in cardiac output and heart rate. Thoracic epidural anesthesia/analgesia may aid in this (see Postoperative Analgesia later in this chapter). - The ability to provide effective regional anesthesia with thoracic epidural blockade has opened the possibility of performing some thoracic surgical procedures in awake patients. A wide variety of thoracic operations have been reported using this technique, including thoracoscopy, thoracotomy, and sternotomy.107 Epidural block is often combined with ipsilateral stellate ganglion block to decrease the cough reflex from stimulation of the hilum. The specific indications to use these techniques do not arise commonly but may occur in severely compromised patients with airway management issues.108 Obviously the original concerns with the pneumothorax syndrome apply to prolonged surgery with these techniques. However, during VATS, the operative hemithorax is effectively closed and gas exchange is usually adequate during spontaneous ventilation with supplemental oxygen.

The only therapy that improves long-term survival and decreases right-sided heart strain in COPD is A. Antibiotic B. Corticosteroids C. Oxygen D. Vasodilators

C. Right Ventricular Dysfunction - Right ventricular (RV) dysfunction occurs in as many as 50% of COPD patients. The dysfunctional right ventricle is poorly tolerant of sudden increases in afterload,34 such as the change from spontaneous to controlled ventilation.35 RV function becomes critical in maintaining cardiac output as the pulmonary artery pressure rises. The RV ejection fraction does not increase with exercise in COPD patients as it does in normal patients. Chronic recurrent hypoxemia is the cause of the RV dysfunction and the subsequent progression to cor pulmonale. Patients who have 1948episodic hypoxemia in spite of normal lungs (e.g., central alveolar hypoventilation, SAHS)36 develop the same secondary cardiac problems as COPD patients. The only therapy that improves long-term survival and decreases right-sided heart strain in COPD is administration of increasing concentrations of oxygen. COPD patients who have a resting PaO2 less than 55 mm Hg, as well as those who have decreases to less than 44 mm Hg with exercise, should receive supplemental oxygen at home. The goal of supplemental oxygen is to maintain a PaO2 of 60 to 65 mm Hg. Compared with patients with chronic bronchitis, emphysematous COPD patients tend to have a decreased cardiac output and mixed venous oxygen tension while maintaining lower pulmonary artery pressures. Bullae - Many patients with moderate or severe COPD develop cystic air spaces in the lung parenchyma known as bullae. These bullae are often asymptomatic unless they occupy more than 50% of the hemithorax, in which case the patient presents with findings of restrictive respiratory disease in addition to their obstructive disease. A bulla is a localized area of loss of structural support tissue in the lung with elastic recoil of surrounding parenchyma (Fig. 66-6). The pressure in a bulla is actually the mean pressure in the surrounding alveoli averaged over the respiratory cycle. This means that during normal spontaneous ventilation, the intrabulla pressure is actually slightly negative in comparison with the surrounding parenchyma.37 However, whenever positive-pressure ventilation is used, the pressure in a bulla becomes positive in relation to the adjacent lung tissue, and the bulla expands with the attendant risk of rupture, tension pneumothorax, and bronchopleural fistula. Positive-pressure ventilation can be used safely in patients with bullae provided the airway pressures are kept low and there is adequate expertise and equipment immediately available to insert a chest drain and obtain lung isolation if necessary. Management of patients for bullectomy is discussed later.

The right upper lobe bronchus originates at a distance of _____ to ___ cm from the carina A. 0.5-1 B. 1-1.5 C. 1.5-2 D. 2-2.5

C. Right-Sided Double-Lumen Endobronchial Tubes - Although a left-sided DLT is used more commonly for most elective thoracic procedures,80 there are specific clinical situations in which the use of a right-sided DLT is indicated (Box 66-5). The anatomic differences between the right and the left mainstem bronchus are reflected in the fundamentally different designs of the right-sided and left-sided DLTs. Because the right mainstem bronchus is shorter than the left bronchus, and because the right upper lobe bronchus originates at a distance of 1.5 to 2 cm from the carina, techniques using right endobronchial intubation must take into account the location and potential for obstruction of the orifice of the right upper 1959lobe bronchus. The right-sided DLT incorporates a modified cuff and slot on the endobronchial lumen that allows ventilation for the right upper lobe

Which of the following is not an appropriate intervention during OLV A. FIO2 1.0 B. PEEP on dependent lung C. CPAP on dependent lung D. Apply Recruitement maneuver

C. Treatment of Hypoxemia During One-Lung Ventilation - During OLV there will be a fall in arterial oxygenation that usually reaches its nadir 20 to 30 minutes after the initiation of OLV, and then the saturation will stabilize or may rise slightly as HPV increases over the next 2 hours. The majority of patients who desaturate do so quickly and within the first 10 minutes of OLV. Hypoxemia during OLV responds readily to treatment in the vast majority of cases. Potential therapies are outlined in Box 66-9. 1. Resume two-lung ventilation. Reinflate the nonventilated lung and deflate the bronchial cuff of the DLT or the bronchial blocker. This will necessitate interruption of surgery but is necessary in case of severe or precipitate desaturation. After an adequate level of oxygenation is obtained, the diagnosis of the cause of desaturation can be made and prophylactic measures instituted (see later) before another trial of OLV is attempted. Therapies for Desaturation during One-Lung Ventilation • Severe or precipitous desaturation: resume two-lung ventilation (if possible). • Gradual desaturation • Ensure that delivered FiO2 is 1.0. • Check position of double-lumen tube or blocker with fiberoptic bronchoscopy. • Ensure that cardiac output is optimal; decrease volatile anesthetics to <1 MAC. • Apply a recruitment maneuver to the ventilated lung (this will transiently make the hypoxemia worse). • Apply PEEP 5 cm H2O to the ventilated lung (except in patients with emphysematous pathology). • Apply CPAP 1-2 cm H2O to the nonventilated lung (apply a recruitment maneuver to this lung immediately before CPAP). • Use intermittent reinflation of the nonventilated lung. • Partial ventilation techniques of the nonventilated lung: • Lung oxygen insufflation • Lobar insufflation • Lobar collapse (using a bronchial blocker) • Use mechanical restriction of the blood flow to the nonventilated lung PEEP, positive end-expiratory pressure; CPAP, continuous positive airway pressure.

The most important assessment of respiratory function is A. Respiratory mechanics B. Lung parenchymal function C. Cardiopulmonary interaction D. Ventilation-perfusion scintigraphy

Cardiopulmonary Interaction - The final and perhaps most important assessment of respiratory function is an assessment of the cardiopulmonary interaction. Formal laboratory exercise testing is currently the gold standard for assessment of cardiopulmonary function,8 and the maximal oxygen consumption (VO2max) is the most useful predictor of postthoracotomy outcome. The risk of morbidity and mortality is increased if the preoperative VO2max is less than 15 mL/kg/min and is considered very high if it is less than 10 mL/kg/min.9 Few patients with a VO2max greater than 20 mL/kg/min have respiratory complications (for comparison, the highest VO2max recorded is 85 mL/kg/min by the American cyclist Lance Armstrong in 200510). Complete laboratory exercise testing is expensive. Several alternatives have been demonstrated to be valid surrogate tests for prethoracotomy assessment. - The distance that a patient with chronic obstructive pulmonary disease (COPD) can walk during a 6-minute test shows an excellent correlation with VO2max and requires little or no laboratory equipment.11 VO2max can be estimated from the 6-minute walk test distance in meters divided by 30 (i.e., 6-minute walk text of 450 m: estimated VO2max = 450/30 = 15 mL/kg/min). Patients with a decrease of oxygen saturation (SpO2) greater than 4% during exercise12 are also at increased risk. The traditional exercise test in ambulatory patients is stair climbing. The ability to climb five flights of stairs correlates with a VO2max more than 20 mL/kg/min, and climbing two flights corresponds to a VO2max of 12 mL/kg/min. - After pulmonary resection, right ventricular dysfunction appears to be in proportion to the amount of functioning pulmonary vascular bed that has been removed. The exact etiology and duration of this dysfunction remains unknown. Clinical evidence of this hemodynamic problem is minimal when the patient is at rest but is dramatic when the patient exercises, leading to elevation of pulmonary vascular pressures, limitation of cardiac output, and absence of the normal decrease in pulmonary vascular resistance usually seen with exertion.13

Left-sided DLT is contraindicated in the following except A. Internal lesions of the airway B. Compression of the trachea or main bronchi by an external mass C. Presence of a descending thoracic aortic aneurysm D. Left-sided hemorrhage

D. Advantages and Disadvantages - Although the presence of two lumens limits the internal diameter of each, the external diameter of a DLT is large. The 37F DLT has an outer diameter equivalent to that of a standard 11-mm internal diameter (ID) ETT. For this reason, DLTs are not used for small children; the external diameter of the 26F DLT, which is the smallest version, is 7.5 mm.95 Sizing of DLTs is determined by patient height, usually leading to the use of 35F to 37F tubes in females, and 39F to 41F tubes in males. More specifically, DLT use can include 35F for women up to 160 cm, 37F for women over 160 cm, 37/39F for men less than 175 cm, and 39/41F for men over 175 cm.96 - The distance from the carinal bifurcation to the right upper lobe is 1.5-2 cm, as compared with a 4-5 cm left mainstem bronchus. Modifications have been made in right-sided tubes to allow ventilation through a slot in the endobronchial cuff or to use two bronchial cuffs, but even slight movement of the right DLT can lead to malpositioning. Many practitioners have resolved to use left-sided DLTs for all right and left thoracotomies unless a left-sided tube is contraindicated by internal lesions of the airway, compression of the trachea or main bronchi by an external mass, or the presence of a descending thoracic aortic aneurysm, which can compress or erode the left main bronchus. This is due to the close proximity of the right upper bronchi to the carina. The bronchial cuff of a right-sided DLT is more likely to occlude the right upper lung lobe and further decrease ventilation. Intubation with the large DLT can pose a challenge, even in patients with a normal airway; insertion in those with poor airway anatomy may be particularly challenging.

Predictive factors for renal dysfunction include the following A. Preoperative hypertension B. Angiotensin II receptor blockers C. Hyoxyethel starch D. Closed thoracotomies

D. Age-Related Factors (See Chapter 80) - There is no maximum age that is a cutoff for pulmonary resection.4 The operative mortality in a group of patients 80 to 92 years of age was 3%, a very respectable figure, in one series.22 However, the rate of respiratory complications (40%) was double that expected in a younger population, and the rate of cardiac complications (40%), particularly arrhythmias, was nearly triple that which would be seen in younger patients. In older patients, thoracotomy should be considered a high-risk procedure for cardiac complications, and cardiopulmonary function is the most important part of the preoperative assessment. An algorithm for the cardiac assessment of the older patient for thoracic surgery is presented in Figure 66-5. Although the mortality resulting from lobectomy among the older patients is acceptable, the mortality from pneumonectomy, particularly right pneumonectomy, is excessive.23 Quality of life after pneumonectomy is significantly worse than it is after lesser pulmonary resections.24 For these reasons, lung-sparing pulmonary resections are performed whenever possible. As a proportion of all lung cancer resections, pneumonectomy has decreased to approximately one third of its share of 15 years ago.25 Exercise tolerance seems to be the primary determinant of outcome in older patients (see Chapter 80). Older patients should have, as a minimum cardiac investigation, a transthoracic echocardiogram to rule out pulmonary hypertension. Renal Dysfunction - Renal dysfunction after pulmonary resection surgery was previously associated with a high mortality rate. Gollege and Goldstraw26 reported a perioperative mortality of 19% (6/31) in patients in whom a significant elevation of serum creatinine developed in the postthoracotomy period, compared with 0% (0/99) in those who did not show any renal dysfunction. More recently, postthoracotomy renal dysfunction, as assessed by significant increases in serum creatinine levels, has been found to be associated with prolonged length of stay but not increased mortality.27 Predictive factors for renal dysfunction include preoperative hypertension, angiotensin II receptor blockers, use of hydoxyethel starch, and open thoracotomies.

Which of the following are preoperative Factors That Characterize Elevated Risk for postoperative complications A. PPO FEV1 > 80% of predicted B. V̇O2 max > 15 mL/kg per min C. FEV1 > 2 L or 80% of predicted D. PPO FEV1 < 40% of predicted

D. Summary of Preoperative Factors That Predict Postoperative Complications Factors That Characterize Average Risk • FEV1 > 2 L or 80% of predicted • PPO FEV1 > 80% of predicted • PPO FEV1 + PPO DLCO both > 40% • V̇O2 max > 15 mL/kg per min • Ability to climb three flights of stairs Factors That Characterize Elevated Risk • FEV1 < 2 L or < 40% of predicted • PPO FEV1 < 40% of predicted • PPO DLCO < 40% of predicted • PPO product (FEV1 × DLCO) < 1,650 • V̇O2 max < 10 mL/kg per min • Inability to climb one flight of stairs • Oxygen desaturation > 4% during exercise

Which chemotherapy agent is associated with pulmonary toxicity from high inspired oxygen concentrations A. Cisplatin B. Doxorubicin C. Methotrexate D. Bleomycin

D. Assessment of the Patient with Lung Cancer (See Chapter 38) - At the time of initial assessment, cancer patients should be assessed for the "4 Ms" associated with malignancy (Box 66-1): mass effects, metabolic abnormalities, metastases, and medications. The prior use of medications that can exacerbate oxygen-induced pulmonary toxicity, such as bleomycin, should be considered.56 Bleomycin is not used to treat primary lung cancers, but patients presenting for excision of lung metastases from germ-cell tumors often have received prior bleomycin therapy. Although the association between previous bleomycin therapy and pulmonary toxicity from high inspired oxygen concentrations is well documented, none of the details of the association are understood (i.e., safe doses of oxygen or safe period after bleomycin exposure). The safest anesthetic management is to use the lowest FiO2 consistent with patient safety and to closely monitor oximetry in any patient who has received bleomycin. We have seen lung cancer patients who received preoperative chemotherapy with cisplatin and then developed an elevation of serum creatinine when they received nonsteroidal antiinflammatory drugs (NSAIDs) postoperatively. For this reason, we do not routinely administer NSAIDs to patients who have been treated recently with cisplatin.

Which of the following about bronchial blockers is false A. The smallest internal diameter (ID) of an endotracheal tube that will allow passage of both a bronchial blocker and a fiberoptic bronchoscope depends on the diameters of the bronchoscope and blocker. B. or standard adult 9-Fr blockers, an endotracheal tube greater than or equal to 7.0 mm ID can be used with a bronchoscope less than 4.0 mm in diameter. C. All blockers need to be well lubricated before placement. D. Larger bronchoscopes will require an endotracheal tube greater than 8.5 mm ID

D. Bronchial Blockers - An alternative method to achieve lung separation involves blockade of a mainstem bronchus to allow lung collapse distal to the occlusion (Fig. 66-17). Bronchial blockers also can be used selectively to achieve lobar collapse if necessary. Currently, there are several different bronchial blockers available to facilitate lung separation. These devices are either within a modified SLT as an enclosed bronchial blocker (Torque Control Blocker Univent; Vitaid, Lewinston, N.Y.) or are used independently with a conventional SLT, the Arndt wire-guided endobronchial blocker (Cook Critical Care, Bloomington, Ind), the Cohen tip-deflecting endobronchial blocker (Cook Critical Care), the Fuji Uniblocker (Vitaid), or the EZ Blocker (Teleflex, Dresden, Germany). - There are specific conditions in which a bronchial blocker may be preferred to a DLT, such as patients with previous oral or neck surgery who present with a challenging airway and require lung separation for intrathoracic surgery. In these cases, the use of an SLT during an awake nasotracheal or orotracheal intubation or via tracheostomy secures the airway, and thereafter an independent bronchial blocker can be placed to achieve lung separation. Another group of patients who may benefit from the use of bronchial blockers are cancer patients who have undergone a previous contralateral pulmonary resection. In such cases, selective lobar blockade with a bronchial blocker in the ipsilateral side improves oxygenation and facilitates surgical exposure. Blockers can be advanced over a guidewire placed with a fiberoptic bronchoscope into the required lobar bronchus. Bronchial blockers are most commonly used intraluminally (coaxially) with the SLT. The Cohen and Fuji Uniblockers can also be placed separately through the glottis or tracheostomy exterior to the SLT. This allows the use of a smaller SLT and is often an option in pediatrics. Another advantage of the bronchial blockers is when postoperative mechanical ventilation is being considered after prolonged thoracic or esophageal surgery. In many instances, these patients have an edematous upper airway at the end of the procedure. If a bronchial blocker is used, there is no need to change the SLT and there is no compromise of the airway if mechanical ventilation is needed in the postoperative period. Table 66-7 describes the characteristics of current 1962bronchial blockers. The smallest internal diameter (ID) of an endotracheal tube that will allow passage of both a bronchial blocker and a fiberoptic bronchoscope depends on the diameters of the bronchoscope and blocker. For standard adult 9-Fr blockers, an endotracheal tube greater than or equal to 7.0 mm ID can be used with a bronchoscope less than 4.0 mm in diameter. Larger bronchoscopes will require an endotracheal tube greater than 7.5 mm ID. All blockers need to be well lubricated before placement.

More than __% of pulmonary artery catheters float into the right lung A. 30 B. 50 C. 70 D. 90

D. Central Venous Pressure Monitoring - Central venous pressure (CVP) monitoring is not required for routine thoracotomies, but may be indicated if the patient's volume status is unclear or if large fluid shifts are anticipated. In complex cases, a CVP line may help manage fluid status. Increased filling pressures (CVP or pulmonary capillary wedge) have been associated with greater lung injury and prolonged mechanical ventilation following complex pulmonary surgery.83 In addition, a large-bore CVP line can provide access for rapid infusion, and an access site should transvenous pacing or pulmonary artery pressure monitoring become necessary. - The CVP line can be inserted via the external or internal jugular veins or the subclavian veins. An external jugular line is more easily kinked in the lateral position. One should remain alert to the possibility of pneumothorax with the insertion of central lines. A pneumothorax on the ventilated (nonoperative) side can lead to severe hypoxemia during OLV. If a subclavian approach to venous cannulation is planned, the insertion site should be on the same side as the planned thoracotomy. Cardiac Performance Monitoring -Pulmonary artery pressure monitoring is intended to provide estimation of left ventricular pressures and guide the support of cardiac performance with fluids and cardiovascular drugs. In spite of its past popularity, pulmonary artery catheterization has not been demonstrated to improve patient outcomes in either cardiac or noncardiac surgery,84 nor is it helpful in predicting postoperative complications.53,85 There have even been suggestions that right heart catheterization may promote cardiac complications.86 If pulmonary artery monitoring is deemed useful, the anesthetist must be cautious in interpreting values when pulmonary disease is present because the normal correlation of right and left ventricular pressures may be inaccurate. The use of pulmonary artery catheters has specific limitations during OLV. Lung pathology or hypoxic pulmonary vasoconstriction may alter the resistance in pulmonary vessels and reduce the correlation between pulmonary artery occlusion pressure and left ventricular pressure.87 More than 90% of pulmonary artery catheters float into the right lung.88 During right thoracotomy, then, the catheter will likely be in the nondependent, collapsed lung and give a false low reading for cardiac output. Finally, care must be taken to ensure that a pulmonary artery catheter is not situated in a vessel that will be clamped during the course of lung resection. A better evaluation of cardiac filling, contractility, and valvular performance would come by way of echocardiography. Preoperative echocardiography is indicated when suspicion exists of valvular disease, outflow tract obstruction, ventricular dysfunction, or pulmonary hypertension.17,53

Which of the following has the least ability to inhibit HPV A. Halothane B. Enflurane C. isoflurane D. Sevoflurane

D. Choice of Anesthetic - All of the volatile anesthetics inhibit HPV in a dose-dependent fashion with halothane > enflurane > isoflurane.123 The older anesthetics were potent inhibitors of HPV and this may have contributed to the high incidence of hypoxemia reported during OLV in the 1960s and 1970s (see earlier); many of these studies used 2- to 3-MAC doses of halothane. - In doses of less than or equal to 1 MAC, the modern volatile anesthetics (isoflurane, sevoflurane,124 and desflurane125) are weak, and equipotent, inhibitors of HPV. The inhibition of the HPV response by 1 MAC of a volatile anesthetic such as isoflurane is approximately 20% of the total HPV response, and this could account for only a net 4% increase in total arteriovenous shunt during OLV, which is a difference too small to be detected in most clinical studies.126 In addition, volatile anesthetics cause less inhibition of HPV when delivered to the active site of vasoconstriction via the pulmonary arterial blood than via the alveolus. This pattern is similar to the HPV stimulus characteristics of oxygen. During established OLV, the volatile agent only reaches the hypoxic lung pulmonary capillaries via the mixed venous blood. No clinical benefit in oxygenation during OLV has been shown for total intravenous anesthesia above that seen with 1 MAC of the modern volatile anesthetics.127 - The use of nitrous oxide/oxygen (N2O/O2) mixtures is associated with a higher incidence of postthoracotomy radiographic atelectasis (51%) in the dependent lung than when air/oxygen mixtures are used (24%). Nitrous oxide also tends to increase pulmonary artery pressures in patients who have pulmonary hypertension, and N2O inhibits HPV. For these reasons, N2O is usually avoided during thoracic anesthesia.

Relevant signs associated with increased pulmonary vascular resistance (PVR) include the following except A. Prominent pulmonary arteries B. rapid tapering of the vasculature C. widened right heart border. D. widened left heart border

D. Diagnostic Data Chest Radiograph - A chest radiograph should be obtained to determine the presence of associated disease and COPD complications (tumor infringement on airway or vascular structures, bullous disease, congestive heart failure, or pneumothorax). The radiograph does not provide abundant information regarding the degree of COPD, but findings characteristic of COPD include hyperinflation, increased anteroposterior thoracic diameter, and diaphragm flattening. Emphysema-related bullae may be present, and infection or pleural effusions may be noted preoperatively and treated to improve the postoperative course. The locations of masses can be identified. In some patients, it can be ascertained whether lesions compress mediastinal structures, cause tracheal shift, or invade the airway. This information is important to predict whether intubation will be difficult, whether induction of anesthesia could cause collapse of the airway, or whether surgical dissection may be difficult and potentially involve excessive bleeding. Evidence of increased pulmonary vascular resistance (PVR) resulting from compression of the vascular bed increases the likelihood of right ventricular failure and worsens the prognosis following lung resection. Relevant signs include prominent pulmonary arteries with rapid tapering of the vasculature, and a widened right heart border.

he most common complications from type II hernias are the following except A. Blood loss B. Anemia C. Gastric volvulus D. Aspiration

D. Esophageal Surgery for Benign Disease Hiatal Hernia - Although most patients with gastroesophageal reflux have a hiatal hernia, most patients with a hiatal hernia do not have significant reflux.206 Patients with heartburn have a lowered barrier pressure and may be at increased risk for regurgitation of gastric contents. Two types of hiatal hernia have been described. Type I hernias, also called "sliding hernias," make up approximately 90% of esophageal hiatal hernias. In this type, the esophagogastric junction and fundus of the stomach have herniated axially through the esophageal hiatus into the thorax (Fig. 66-44). The term sliding refers to the presence of a sac of parietal peritoneum. The lower esophageal sphincter is cephalad to the diaphragm and may not respond appropriately to increased abdominal pressure. Thus a reduced barrier pressure during coughing or breathing leads to regurgitation. The type II or "paraesophageal hiatal hernia," is characterized by portions of the stomach herniating into the thorax next to the esophagus. In the presence of a type II hernia, the esophagogastric junction is still located in the abdomen. The most common complications from type II hernias are blood loss, anemia, and gastric volvulus. The goal of surgical repair of a sliding hernia is to obtain competence of the gastroesophageal junction. Because restoration of the normal anatomy is not always successful in preventing subsequent reflux, several antireflux operations have been developed, such as the Nissen fundoplication. Repair of a hiatal hernia can be performed via a thoracotomy or laparotomy, or minimally invasively. Benign Esophageal Stricture - Chronic reflux of acidic gastric contents can lead to ulceration, inflammation, and eventually stricture of the esophagus. The pathologic 1987changes are reversible if the acidic gastric contents cease their contact with the esophageal mucosa. Surgery may be necessary if medical treatment and dilatations are inadequate. There are two types of surgical repair, both of which are usually approached via a left thoracoabdominal incision. Gastroplasty after esophageal dilatation interposes the fundus of the stomach between the esophageal mucosa and the acidic milieu of the stomach. The remaining fundus may be sewn to the lower esophagus to create a valvelike effect. The second type of repair is resection of the stricture and the creation of a thoracic end-to-side esophagogastrostomy. Vagotomy and antrectomy are performed to eliminate stomach acidity, and a Roux-en-Y gastric drainage procedure is performed to prevent alkaline intestinal reflux.

Which of the following is considered a stretch injury of the brachial plexus in the lateral position A. Pressure on clavicle into retroclavicular space B. Cranial migration of thorax padding into the axilla C. Arm directly under thorax D. Lateral flexion of cervical spine

D. Factors That Contribute to Brachial Plexus Injury in the Lateral Position • Dependent arm (compression injuries) • Arm directly under thorax • Pressure on clavicle into retroclavicular space • Cervical rib • Cranial migration of thorax padding into the axilla∗ • Nondependent arm (stretch injuries) • Lateral flexion of cervical spine • Excessive abduction of arm (>90 degrees) • Semiprone or semisupine repositioning after arm fixed to a support

Which of the following are Factors That Reduce effectiveness of Hypoxic Pulmonary Vasoconstriction A. Vasoconstriction B. Hemoconcentration C. Acidosis D. Hypothermia

D. Factors That Reduce Effectiveness of Hypoxic Pulmonary Vasoconstriction • Alkalosis • Excessive tidal volume or PEEP • Hemodilution • Hypervolemia (LAP > 25 mm Hg), atrial natriuretic peptide • Hypocapnia • Hypothermia • Prostacyclin • Shunt fraction < 20% or > 80% • Vasodilators, phosphodiesterase inhibitors, and calcium channel blockers • Volatile anesthetics > 1.5 MAC

Which of the following is associated with patients with lung cancer A. Addison's DIsease B. Hypocalcemia C. Myasthenia Gravis D. Carcinoid Syndrome

D. History and Physical Examination - Cancer patients who undergo lung resection typically have a history of multiple risk factors and signs of respiratory disease. Risk factors include cigarette smoking, air pollution, and industrial chemical exposure. The smoking history that is so commonly associated with these patients increases the incidence of ischemic heart disease or hypertension. Therefore patients must be evaluated for exertional dyspnea, productive cough, hemoptysis, cyanosis, poor exercise tolerance, and chest pain. - The presence of ischemic cardiac disease that is severe, unstable, or associated with dysrhythmias should indicate consultation from a cardiologist to help mitigate the risk of cardiac complications. Because severe COPD may significantly limit physical activity, a provocative pharmacologic cardiac stress test may be helpful to identify coronary insufficiency.15 The use of perioperative beta blockade is controversial, and nonselective agents may be particularly detrimental if they inhibit bronchodilation. However, cardioselective β-blockers have been found to be beneficial in COPD patients undergoing vascular surgery.16 Beta blockade may be considered to reduce cardiac risk;17 at the least, patients currently taking β-blockers should continue their regimen throughout the surgical encounter. The surgical plan must be carefully considered for patients with coronary diffusion defects that show greater than 20% reversibility. In cases of high risk cardiac disease, lung surgery may be delayed for 6 weeks to allow coronary artery bypass first.15 With the increasing popularity of off-bypass coronary revascularization, the two procedures can more easily be performed in a single surgical encounter.18 - Patients with lung cancer should be assessed for effects of the primary tumor, as well as effects of secondary pathologies and side effects of therapy (Table 30.1). Supine dyspnea may result from COPD or compression of the airway by a mediastinal mass. A high index of suspicion should also be maintained for hormonal abnormalities, because many lung tumors cause paraneoplastic syndromes characterized by the secretion of endocrine-like substances such as adrenocorticotropic hormone,19 antidiuretic hormone,20 serotonin, parathyroid-like hormone,21 and insulin-like growth factor,22 causing a variety of metabolic abnormalities.23 Cushing disease may lead to metabolic alkalosis, hypokalemia, and hyperglycemia.24,25 Hypercalcemia occurs in 10% to 25% of patients with lung cancer related to parathyroid-like hormone, increased calcitriol, or overactivity of osteoclasts. The finding of hypercalcemia carries a very poor prognosis. Clinical signs may include polyuria, polydipsia, confusion, vomiting, abdominal cramps, bradycardia, and mental status changes. Neuroendocrine tumors comprise 20% of lung cancers, and 5% of them produce carcinoid syndrome.26 In those patients, histamine-stimulating drugs and adrenergic agonists may precipitate the flushing, hypotension, and tachyarrhythmias related to serotonin release. Paraneoplastic neurologic syndromes represent autoimmune dysfunctions associated with cancers. In total, 1% to 2% of patients with small cell lung cancer develop Lambert-Eaton myasthenic syndrome (LEMS).27 In this syndrome, antibodies are formed that inhibit voltage-gated calcium channels. This decreases the release of acetylcholine from presynaptic nerve terminals,28 resulting in weakness and sensitivity to nondepolarizing muscle relaxants. Usually, the initial presentation with autonomic dysfunction (80% of patients with LEMS) includes orthostatic hypotension followed by weakness that progresses upward from the legs.29 In contrast to myasthenia gravis, LEMS involves dysfunction of the calcium channels. Repetitive stimulation or activity improves function (as more acetylcholine is mobilized), and anticholinesterase drugs are not an effective treatment. Treatment includes immunoglobulin, corticosteroids, or 3,4-diaminopyridine (3,4-DAP), which opens potassium channels and increases calcium concentrations in the nerve terminal. Other autoimmune channelopathies can affect voltage-gated potassium channels and prolong acetylcholine release, causing myotonia, or affect autonomic ganglia, causing orthostatic hypotension and arrhythmias.3 - Physical examination findings in COPD are commonly barrel chest deformity, accessory muscle use or paradoxical breathing movement, pursed-lip breathing, and tympanic percussion notes on the chest. Auscultation reveals rhonchi or wheezing. Signs of cor pulmonale include jugular vein distention or peripheral edema, split S2 heart sound, pulmonary or tricuspid valve insufficiency murmurs, and rales auscultated over the lungs.31 Nutritional status, commonly compromised in patients with cancer, is also important to note because hypoalbuminemia and malnutrition are associated with increased postoperative complications such as pneumonia.32 Box 30.2 lists important elements of the preoperative evaluation.

Which of the following is an absolute contraindication for lung transplantation A. Untreatable end-stage pulmonary disease B. Substantial limitation of daily activities C. Critical condition D. Substance addiction

D. Indications and Contraindications for Lung Transplantation Indications • Untreatable end-stage pulmonary, parenchymal, or vascular disease • Absence of other major medical illnesses • Substantial limitation of daily activities • Projected life expectancy less than 50% of 2- to 3-year predicted survival • New York Heart Association Class III or IV functional level • Rehabilitation potential • Satisfactory psychosocial profile and emotional support system • Acceptable nutritional status • Disease-specific mortality exceeding transplant-specific mortality over 1 to 2 years Relative Contraindications • Age > 65 years • Critical or unstable clinical conditions (e.g., shock, mechanical ventilation, extracorporeal membrane oxygenation) • Severely limited functional status with poor rehabilitation potential • Colonization with highly resistant or virulent bacteria, fungi, or mycobacteria • Severe obesity defined as a body mass index greater than >30 kg/m2 • Severe or symptomatic osteoporosis • Other medical conditions not resulting in end-organ damage (e.g., diabetes mellitus, systemic hypertension, peripheral vascular disease, gastroesophageal reflux, patients with coronary artery disease s/p coronary artery stenting or percutaneous transluminal coronary angioplasty) Absolute Contraindications • Untreatable advanced dysfunction of another major organ system (e.g., heart, liver, kidney) • Active malignancy within the previous 2 years • Noncurable chronic extrapulmonary infection • Chronic active viral hepatitis B, hepatitis C, and HIV • Significant chest wall or spinal deformity • Documented nonadherence or inability to follow through with medical therapy, office follow-up, or both • Untreatable psychiatric or psychological condition associated with inability to cooperate or comply with medical therapy • Absence of a consistent or reliable social support system • Substance addiction (e.g., alcohol, tobacco, narcotics) that is either currently active or was active within the previous 6 months

The alternatives to achieve OLV in a tracheostomized patient include the following except A. insertion of an SLT followed by an independent bronchial blocker passed coaxially or external to the SLT B. the use of a disposable cuffed tracheostomy cannula with an independent bronchial blocke C. replacement of the tracheostomy cannula with a specially designed short DLT such as the Naruke DLT, which is made for use in tracheostomized patients D. placement of a large DLT through the tracheostomy stoma E. if possible, oral access to the airway for standard placement of a DLT or blocker

D. Lung Isolation Techniques in Patients with a Tracheostomy in Place - Placement of a DLT through a tracheostomy stoma may be prone to malpositioning because the upper airway has been shortened and the conventional DLT may be too long. Before placing any lung isolation devices through a tracheostomy stoma, it is important to consider whether it is a fresh stoma (i.e., a few days old, when the airway can be lost immediately on decannulation) versus a chronic tracheostomy. The alternatives to achieve OLV in a tracheostomized patient include (1) insertion of an SLT followed by an independent bronchial blocker passed coaxially or external to the SLT91; (2) the use of a disposable cuffed tracheostomy cannula with an independent bronchial blocker; (3) replacement of the tracheostomy cannula with a specially designed short DLT such as the Naruke DLT, which is made for use in tracheostomized patients92; (4) placement of a small DLT through the tracheostomy stoma; or (5) if possible, oral access to the airway for standard placement of a DLT or blocker (this is occasionally an option in patients on prolonged mechanical ventilation for respiratory failure or postoperative complications). - In summary, the optimal method of lung isolation depends on a number of factors, including the patient's airway anatomy, the indication for lung isolation, the available equipment, and the training of the anesthesiologist. Suggested methods for lung isolation in specific clinical situations are listed in Table 66-8. Whichever method of lung isolation is used, the "ABCs" of lung isolation are: - Anatomy: Know the tracheobronchial anatomy. One of the major problems that many anesthesiologists have in achieving satisfactory lung isolation is a lack of familiarity with distal airway anatomy (Fig. 66-20). - Bronchoscopy: Whenever possible, use a fiberoptic bronchoscope to position endobronchial tubes and blockers. The ability to perform fiberoptic bronchoscopy is now a fundamental skill needed by all anesthesiologists 1964providing anesthesia for thoracic surgery. An online bronchoscopy simulator has been developed to help train anesthesiologists in positioning DLTs and blockers. This simulator, which uses real-time video, is available without cost at www.thoracicanesthesia.com. - Chest imaging: The anesthesiologist should always consult the chest imaging before placement of a DLT or blocker. Abnormalities of the lower airway can often be identified in advance, and this will affect the selection of the optimal method of lung isolation for a specific case

Which of the following is not an appropriate management of one-lung anesthesia A. Tidal volume: 6-8 mL/kg B. Rate: 12-15 (permissive hypercapnia acceptable C. FiO2: 0.4-0.8; maintain SpO2 > 90% D. Titrate PEEP in NDL

D. Management of One-Lung Anesthesia • Ventilate: • Tidal volume: 6-8 mL/kg • Rate: 12-15 (permissive hypercapnia acceptable • FiO2: 0.4-0.8; maintain SpO2 > 90% • PEEP: 5-10 cm H2O (2.5-5 if COPD) • I:E ratio: 1 : 2 (1 : 3 if COPD or intrinsic (auto) PEEP) • Consider alveolar recruitment maneuver prior to OLV • Assess ABG 15 minutes after OLV is initiated • Volatile anesthetics < 1-1.5 MAC or IV agents • Stepwise response to worsening hypoxemia: • Increase FiO2 • Confirm tube position with fiberoptic bronchoscope • Ensure adequacy of cardiac output • Remedy detrimental effects caused by anemia or vasodilators • Perform alveolar recruitment maneuver to DL • Titrate PEEP in DL • CPAP 5-10 cm H2O to NDL • Intermittent or continuous two-lung ventilation • Low- or no-pressure oxygen insufflation to NDL or selected lobe of NDL • Reposition to lateral decubitus position if supine • Alter perfusion with almitrine to NDL; nitric oxide to DL

Because of its low blood-gas solubility, this gas will delay collapse of the lung A. Oxygen B. Carbon dioxide C. Nitrous oxide D. Nitrogen

D. Management of One-Lung Ventilation - During OLV, the anesthesiologist has the unique and often conflicting goals of trying to maximize atelectasis in the nonventilated lung to improve surgical access while 1968trying to avoid atelectasis in the ventilated lung (usually the dependent lung) to optimize gas exchange. The gas mixture in the nonventilated lung immediately before OLV has a significant effect on the speed of collapse of this lung. Because of its low blood-gas solubility, nitrogen (or an air-oxygen mixture) will delay collapse of this lung (Fig. 66-24).109 This is particularly a problem at the start of VATS surgery when surgical visualization in the operative hemithorax is limited, and in patients with emphysema who have delayed collapse of the nonventilated lung because of decreased lung elastic recoil. It is important to thoroughly de-nitrogenate the operative lung, by ventilating with oxygen, immediately before it is allowed to collapse. Although nitrous oxide is even more effective than oxygen in speeding lung collapse, for the reasons just cited it is not commonly used in thoracic anesthesia because many patients may have blebs or bullae. - Also, during the period of two-lung anesthesia before the start of OLV, atelectasis will develop in the dependent lung. It is useful to perform a recruitment maneuver to the dependent lung (similar to a Valsalva maneuver, holding the lung at an end-inspiratory pressure of 20 cm H2O for 15 to 20 seconds) immediately after the start of OLV to decrease this atelectasis. Recruitment is important to maintain PaO2 levels during subsequent OLV.110

Neurovascular Injuries Specific to the Lateral Position: Routine "Head-to-Toe" Survey include the following except A. Dependent eye B. Dependent ear pinna C. Dependent peroneal nerve D. Dependent sciatic nerve

D. Neurovascular Injuries Specific to the Lateral Position: Routine "Head-to-Toe" Survey • Dependent eye • Dependent ear pinna • Cervical spine in line with thoracic spine • Dependent arm: (a) brachial plexus, (b) circulation • Nondependent arm: (a) brachial plexus, (b) circulation∗ • Dependent and nondependent suprascapular nerves • Nondependent leg sciatic nerve • Dependent leg: (a) peroneal nerve, (b) circulation

Which of the following is not a partial ventilation method for OLV A. Intermittent positive airway pressure to the nonventilated lung. B. Selective insufflation of oxygen to recruit lung segments on the side of surgery but remote from the site of surgery C. Selective lobar collapse of only the operative lobe in the open hemithorax.1 D. Continous positive airway pressure in the ventilated lung

D. Partial Ventilation Methods - Several alternative methods of OLV, all involving partial ventilation of the nonventilated lung, have been described and improve oxygenation during OLV. These techniques are useful in patients who are particularly at risk of desaturation, such as those who have had previous 1975pulmonary resections of the contralateral lung. These alternatives include: 1. Intermittent positive airway pressure to the nonventilated lung. This can be performed by a variety of methods. Russell described attaching a standard bacteriostatic filter to the nonventilated lumen of the DLT with a 2-L oxygen inflow attached to the CO2 port of the filter (Fig. 66-34).156 Manual occlusion of the filter for 2 seconds gives an insufflation of approximately 66 mL of oxygen to the nonventilated lung. This could be repeated at 10-second intervals with minimal interference with surgical exposure. 2. Selective insufflation of oxygen to recruit lung segments on the side of surgery but remote from the site of surgery (Fig. 66-35).157 A useful technique in minimally invasive surgery is intermittent insufflation of oxygen using a fiberoptic bronchoscope. A 5-L oxygen flow is attached to the suction port of a fiberoptic bronchoscope that is passed under direct vision into a segment of the lung remote from the site of surgery, which is then reinflated by triggering the suction on the fiberoptic bronchoscope. The surgeon aids this technique by 1976observing the lung inflation with the videoscope to avoid overdistention of the recruited segment(s). 3. Selective lobar collapse of only the operative lobe in the open hemithorax.158 This is accomplished by placement of a blocker in the appropriate lobar bronchus of the ipsilateral operative lung.

Which of the following is not included in the 4Ms as Anesthetic Considerations in Lung Cancer Patients A. Mass effect B. Metabolic effects C. Metastases D. Melanoma

D. Planning Postoperative Analgesia - The strategy for postoperative analgesia (see Chapter 98) should be developed and discussed with the patient during the initial preoperative assessment. A discussion of postoperative analgesia is presented at the end of this chapter. Many techniques are superior to the use of on-demand parenteral (intramuscular or intravenous) opioids alone in terms of pain control. These include the addition of neuraxial blockade, paravertebral blocks, and antiinflammatories to narcotic-based analgesia. However, only epidural techniques can decrease postthoracotomy respiratory complications in high-risk patients.2 As well, continuous paravertebral blockade may offer comparable analgesia with a less frequent rate of block failure and fewer side effects.57 - At the time of initial preanesthetic assessment, the risks and benefits of the various forms of postthoracotomy analgesia should be explained to the patient. Potential contraindications to specific methods of analgesia should 1952be determined, such as coagulation problems, sepsis, or neurologic disorders. If the patient is to receive prophylactic anticoagulants and it has been elected to use epidural analgesia, appropriate timing of anticoagulant administration and neuraxial catheter placement need to be arranged. American Society of Regional Anesthesia guidelines suggest an interval of 2 to 4 hours before or 1 hour after catheter placement for prophylactic heparin administration.58 Low-molecular-weight heparin (LMWH) recommendations and precautions are: (1) a minimal interval of 12 hours after low-dose LMWH and (2) 24 hours after higher-dose LMWH before catheter placement. Anesthetic Considerations in Lung Cancer Patients (the "4 Ms") 1. Mass effects: Obstructive pneumonia, lung abscess, SVC syndrome, tracheobronchial distortion, Pancoast syndrome, recurrent laryngeal nerve or phrenic nerve paresis, chest wall or mediastinal extension 2. Metabolic effects: Lambert-Eaton syndrome, hypercalcemia, hyponatremia, Cushing syndrome 3. Metastases: Particularly to brain, bone, liver, adrenal 4. Medications: Chemotherapy agents, pulmonary toxicity (bleomycin, mitomycin), cardiac toxicity (doxorubicin), renal toxicity (cisplatin)

After thoracotomy, as much as __% of patients with a functioning epidural describe ipsilateral shoulder pain. A. 20 B. 40 C. 60 D. 80

D. Postoperative Pain Management Problems Shoulder Pain - After thoracotomy, as much as 80% of patients with a functioning epidural describe ipsilateral shoulder pain. Shoulder pain occurs after both thoracotomy and VATS and is thought to be primarily referred pain of diaphragmatic irritation transmitted by phrenic nerve afferents.274 Other factors, including patient position during operation and major bronchus transection, have been incriminated, although the mechanisms are not clear. Several other causes of shoulder pain should be considered when evaluating a patient with shoulder pain postoperatively: 1. A chest drain placed too far into the apex of the hemithorax will irritate the parietal pleura and cause shoulder pain. The postoperative chest radiograph should be reviewed and the chest drain partially withdrawn if this is suspected as the cause. 2. The posterior end of a large posterolateral thoracotomy incision may not be well blocked by a functioning thoracic epidural, and this may be described as shoulder pain. The extent of the sensory block should be determined on the skin. If inadequate block is a possibility, the thoracic epidural analgesia can be "topped-up." 3. Patients with chronic arthralgia of the shoulder may experience an exacerbation from the intraoperative positioning of the ipsilateral arm. It is important to ask about shoulder problems preoperatively and to position the arms in a manner that does not exacerbate shoulder problems. Shoulder pain is refractory to thoracic epidural analgesia and requires antiinflammatory agents with or without opioids. Shoulder pain is usually transient and is often resolved completely by the second postoperative day. There are several reports of the use of nerve blocks to treat postthoracotomy shoulder pain. Phrenic nerve infiltration and interscalene brachial plexus block289 have had some success but carry a risk of causing diaphragm dysfunction. - Postthoracotomy Neuralgia and Chronic Incisional Pain The transition of acute pain syndromes, associated with thoracic surgery, to chronic ones such as postthoracotomy neuralgia, may be partially preventable with analgesic regimens that prophylactically block nerves and desensitize peripheral nerve endings and dorsal horn cells of nerves that are damaged by surgery.290 Management of Opioid-Tolerant Patients - The opioid-tolerant patient requiring thoracic surgery presents a significant challenge. Patients may be using physician-prescribed opioids for pain related to their thoracic pathology or other chronic pain syndromes. Active abusers of narcotics or those in a rehabilitation program who are receiving daily methadone are also included in this group. Whenever possible, patients should take their regular analgesia or methadone preoperatively; otherwise substitute opioids must be provided. The opioid doses required to produce adequate postoperative analgesia are increased. - A multimodal analgesic regimen is optimal. A choice must be made regarding the method of increased opioid delivery, either systemically or through an epidural solution. An increased narcotic dose may be provided in the epidural solution, or standard narcotic concentrations may be used in the epidural with additional systemic narcotic. DeLeon-Casasola and Yarussi291 report that higher epidural doses of opioids are able to curtail the appearance of narcotic withdrawal in most patients. More frequently, the patient receives a standard or slightly increased concentration of opioid in the epidural infusion and additional systemic opioid to minimize the occurrence of withdrawal. A convenient way to provide drug delivery in patients not immediately able to take oral medication is in the form of a transdermal fentanyl patch. Systemic 2004opioids can be provided as a continuous IV infusion or in oral form. - Patient-controlled analgesic techniques are often difficult to manage in these patients and they may be best managed with fixed dosage regimens that are modified as needed. Ultimately, after dose titration the patient may be receiving both increased epidural opioid and greater than preoperative doses of systemic opioid, without significant side effects. Patients in whom epidural bupivacaine-morphine analgesia is inadequate may respond to a switch to bupivacaine-sufentanil.292 Patients in drug rehabilitation programs with methadone may be reluctant to modify their methadone dose perioperatively, having struggled to establish a stable dose in the past. They frequently can take their full methadone dose throughout the perioperative period. - Supplemental therapies to be considered for these patients include adding epinephrine 5 μg/mL to the epidural infusion solution and the addition of low-dose continuous IV ketamine infusions.293 All opioid-tolerant patients require frequent adjustment of analgesic doses. Despite this, pain scores of 4 to 5 out of 10 with movement are often the lowest that are achievable. The increased analgesic requirements of opioid-tolerant patients are for a longer duration postoperatively than the usual need for analgesia in opioid-naive patients.

This is the most frequent indication for a sleeve lobectomy A. Carcinoid tumors B. Endobronchial metastases C. Primary airway tumors D. Bronchogenic carcinoma

D. Sleeve Lobectomy - Bronchial sleeve resections are performed for neoplasms or benign strictures. Bronchogenic carcinoma is the most frequent indication for a sleeve lobectomy, followed by carcinoid tumors, endobronchial metastases, primary airway tumors, and bronchial adenomas. Sleeve lobectomy involving parenchyma-sparing techniques in patients with a limited pulmonary reserve, is an alternative procedure for patients who cannot tolerate a pneumonectomy. The sleeve technique involves mainstem bronchial resection without parenchymal involvement and possibly resection of pulmonary arteries to avoid pneumonectomy (Fig. 66-42). - Patients undergoing sleeve lobectomy require lung isolation with a contralateral DLT or endobronchial tube (i.e., use a right-sided DLT for a left sleeve lobectomy). High-frequency jet ventilation can be used for resections done close to the tracheal carina. For a sleeve lobectomy involving resectioning of vessels, heparinization is necessary. In these cases, thoracic epidural catheters should not be manipulated for 24 hours after heparin administration. During pulmonary arterioplasty, uncontrollable bleeding may occur. For this reason, large-bore intravenous catheters should be used. Patients undergoing sleeve lobectomy are usually extubated in the operating room before transfer to the postanesthesia recovery room. Immediate and long-term survival is better after sleeve lobectomy compared with right pneumonectomy for comparable stages of right upper lobe cancer.182

Prediction of postresection pulmonary function can be further refined by assessment of the preoperative contribution of the lung or lobe to be resected using A. Respiratory mechanics B. Lung parenchymal function C. Cardiopulmonary interaction D. Ventilation-perfusion scintigraphy

D. Ventilation-Perfusion Scintigraphy - Prediction of postresection pulmonary function can be further refined by assessment of the preoperative contribution of the lung or lobe to be resected using ventilation-perfusion () lung scanning. If the lung region to be 1945resected is nonfunctioning or minimally functioning, the prediction of postoperative function can be modified accordingly. This is particularly useful in pneumonectomy patients14 and scanning should be considered for any pneumonectomy patient who has a preoperative FEV1 and/or DLco less than 80%. However, scanning is of limited usefulness to predict postlobectomy function. Combination of Tests - No single test of respiratory function has shown adequate validity as a sole preoperative assessment. Before surgery, an estimate of respiratory function in all three areas—lung mechanics, parenchymal function, and cardiopulmonary interaction—should be made for each patient. These three aspects of pulmonary function form the "three-legged stool" that is the foundation of prethoracotomy respiratory assessment (Fig. 66-2) that can be used to plan intraoperative and postoperative management (Fig. 66-3). These plans and intraoperative surgical factors sometimes necessitate that a resection becomes more extensive than was foreseen. If a patient has a ppoFEV1 greater than 40%, the trachea usually can be extubated in the operating room at the conclusion of surgery assuming the patient is alert, warm, and comfortable ("AWaC"). If the ppoFEV1 is greater than 30% and exercise tolerance and lung parenchymal function exceed the increased-risk thresholds, then tracheal extubation can be done in the operating room depending on the status of associated medical conditions. Patients in this subgroup who do not meet the minimal criteria for cardiopulmonary and parenchymal function should be considered for staged weaning from mechanical ventilation postoperatively (see Chapter 103). Patients with a ppoFEV1 of 20% to 30% and favorable predicted cardiorespiratory and parenchymal function can be considered for early tracheal extubation if thoracic epidural analgesia is used or if the resection is performed with VATS. In the increased-risk group, the presence of several associated factors and diseases should be documented during the preoperative assessment (see Chapter 3) and will enter into the consideration for postoperative management (discussed later).

The most valid single test for postthoracotomy respiratory complications is A. forced expiratory volume in one second (FEV1) B. forced vital capacity (FVC) C. maximal voluntary ventilation (MVV) D. residual volume/total lung capacity ratio (RV/TLC) E. predicted postoperative FEV1 (ppoFEV1 %)

E. Assessment of Respiratory Function (See Chapter 19) - The best assessment of respiratory function comes from a detailed history of the patient's quality of life. All patients undergoing a pulmonary resection should have baseline simple spirometry done preoperatively.3 Objective measures of pulmonary function are required to guide anesthetic management and to have this information in a format that can be easily transmitted between members of the health care team. Respiratory function can be divided into three related but somewhat independent areas: respiratory mechanics, gas exchange, and cardiorespiratory interaction. The basic functional units of extracellular respiration are to move the oxygen: (1) into the alveoli, (2) into the blood, and (3) into the tissues (the process is reversed for carbon dioxide removal). Respiratory Mechanics - Many tests of respiratory mechanics and volumes show correlation with postthoracotomy outcome: forced 1944expiratory volume in one second (FEV1), forced vital capacity (FVC), maximal voluntary ventilation (MVV), residual volume/total lung capacity ratio (RV/TLC), and so on (see Chapter 19 on pulmonary function testing). It is useful to express these as a percent of predicted volumes corrected for age, sex, and height (e.g., FEV1 %). Of these, the most valid single test for postthoracotomy respiratory complications is the predicted postoperative FEV1 (ppoFEV1 %),4 which is calculated as: - One method of estimating the percent of functional lung tissue is based on a calculation of the number of functioning subsegments of the lung removed (Fig. 66-1). Patients with a ppoFEV1 greater than 40% are at low risk for postresection respiratory complications. The risk of major respiratory complications is increased in the subgroup with a ppoFEV1 less than 40% (although not all patients in this subgroup develop respiratory complications), and patients with a ppoFEV1 less than 30% are at high risk.

Protective ventilation strategy during one-lung ventilation include the following except A. end-expiratory pressures above the lower inflection point on the pressure-volume curve B. a VT of less than 6 mL/kg C. inspiratory pressures of less than 20 cm H2O above the PEEP value D. permissive hypercapnia E. volume control ventilatory mode

E. Management of One-Lung Ventilation Ventilation Modes - The primary goal during OLV is to maintain adequate arterial oxygenation and protecting the lung, while providing a surgical field favorable for visualization and manipulation of the operative lung (Table 30.3). In the past, large VTs of 10-15 mL/kg were recommended to prevent atelectasis in the dependent lung and maintain an adequate FRC. Contemporary understanding is that high volumes predispose to volutrauma, which is associated with increases in cytokine inflammatory mediators, alveolar fibrin deposition, and other markers of procoagulant effect that contribute to acute lung injury.133,134 - During OLV, patients frequently develop "auto-PEEP" and have an increased FRC, such that large VTs are unnecessary. Understanding the detrimental effects of high VTs has led to the more contemporary approach of using more physiologic volumes (e.g., 6 mL/kg on the left and 8 mL/kg on the right), adding PEEP to those patients without auto-PEEP, and limiting peak inspiratory pressures to less than 25 cm H2O.128,133-135 With this approach, patients will maintain adequate or even improved oxygenation (as compared with using a higher VT) and minimal elevations in PaCO2.89,136 It is preferable to allow permissive hypercapnia, rather than aggressively attempting to maintain a normal PaCO2, as hypercapnia supports HPV and directly reduces the cytokine response.137,138 The PaCO2 should be maintained below 60-70 mm Hg to reduce the incidence of dysrhythmias, hypotension, and pulmonary hypertension. Although a high FiO2 should induce vasodilation in the dependent lung and improve blood flow, hypocapnia causes vasoconstriction, and should be avoided. PEEP must be employed when using a low VT, otherwise alveolar derecruitment and atelectasis will occur, reducing PaO2.139 Implementation of an alveolar recruitment maneuver prior to initiating OLV can help to support better PaO2 throughout the OLV period.140 Recruitment maneuvers are not universally accepted. Besides opening airways, they may also translocate cytokines into the circulation,141 impede hemodynamics, and provide only a transient effect.142 - An appropriate air-oxygen mixture, at times as high as an FiO2 of 1, is necessary to maximize the PaO2. However, considering the potential for oxygen toxicity (particularly in a setting of absorptive atelectasis or history of chemotherapeutic administration),143 FiO2 should be maintained at the lowest level that will support adequate SpO2.116 This approach also leaves a "reserve" intervention in the case of hypoxemia. Increasing the FiO2 in the face of declining oxygenation allows time to plan other interventions (such as checking tube placement). On 100% oxygen, by the time the SpO2 falls, the insult will be more advanced, and the saturation will continue to decline during diagnosis and management of the issue. - Research validates the "protective ventilation" strategy, including end-expiratory pressures above the lower inflection point on the pressure-volume curve, a VT of less than 6 mL/kg, inspiratory pressures of less than 20 cm H2O above the PEEP value, permissive hypercapnia, and preferential use of pressure-limited ventilatory modes.144-146 The superiority of pressure or volume modes of ventilation has not been definitively determined. It is compelling to consider that pressure-controlled ventilation limits maximal airway pressure, and that the square pressure waveform provides more widespread alveolar recruitment, but research has been equivocal about demonstrating consistently better outcomes based on the ventilation mode.147-150

True or False Increasing cardiac output has been shown to be associated with decreased arteriovenous shunt during OLV.

Fa;se Cardiac Output - The effects of alterations of cardiac output during OLV are complex (Fig. 66-29). Increasing cardiac output tends to cause increased pulmonary artery pressures and passive dilation of the pulmonary vascular bed, which in turn opposes HPV and has been shown to be associated with increased arteriovenous shunt () during OLV.128 However, in patients with a relatively fixed oxygen consumption, as is seen during stable anesthesia, the effect of an increase in cardiac output is to increase the mixed venous oxygen saturation (SvO2). Thus increasing cardiac output during OLV tends to increase both shunt and SvO2, which have opposing effects on PaO2. There is a ceiling effect to the amount that SvO2 can be increased. Increasing the cardiac output to supranormal levels by administering inotropes such as dopamine tends to have an overall negative effect on PaO2.129 Conversely, allowing the cardiac output to fall will lead to decreases in both shunt and SvO2, with a net effect of decreasing PaO2. Because even with optimal anesthetic management there is usually a shunt of 20% to 30% during OLV, it is very important to maintain cardiac output.

True or False There is a report of less malpositions with the use of bronchial blockers when compared with DLTs

False Complications Related to the Bronchial Blockers - Failure to achieve lung separation because of abnormal anatomy or lack of a seal within the bronchus has been reported.85 Inclusion of the bronchial blocker or the distal wire loop of an Arndt blocker into the stapling line has been reported during a lobectomy86 and required surgical reexploration after unsuccessful removal of the bronchial blocker after extubation. To avoid these mishaps, communication with the surgical team regarding the presence of a bronchial blocker in the surgical side is crucial. Clearly, the bronchial blocker needs to be withdrawn a few centimeters before stapling. - Another potentially dangerous complication with all bronchial blockers is that the inflated balloon can move and lodge above the carina or be accidentally inflated in the trachea. This leads to an inability to ventilate and possibly the development of hypoxia and potentially cardiorespiratory arrest unless quickly recognized and the blocker deflated.87 There is a report of more malpositions with the use of bronchial blockers when compared with DLTs.88

True or False A comprehensive program of pulmonary rehabilitation involving physiotherapy, exercise, nutrition, and education can improve functional capacity for patients with severe COPD.

False Preoperative Therapy of COPD - There are four treatable complications of COPD that must be actively sought and therapy begun at the time of the initial prethoracotomy assessment: atelectasis, bronchospasm, respiratory tract infections, and pulmonary edema (Table 66-1). Atelectasis impairs local lung lymphocyte and macrophage function, predisposing to infection. Pulmonary edema can be very difficult to diagnose by auscultation in the presence of COPD and may present very abnormal radiologic distributions (e.g., unilateral, upper lobes). Bronchial hyperreactivity may be a symptom of congestive failure or may represent an exacerbation of reversible airways obstruction. All COPD patients should receive maximal bronchodilator therapy as guided by their symptoms. Only 20% to 25% of COPD patients respond to corticosteroids. In a patient who is poorly controlled on sympathomimetic and anticholinergic bronchodilators, a trial of corticosteroids may be beneficial. Physiotherapy - Patients with COPD have fewer postoperative pulmonary complications when a perioperative program of intensive chest physiotherapy is initiated preoperatively.42 Among the different modalities available (cough and deep breathing, incentive spirometry, PEEP, continuous positive airway pressure [CPAP]), there is no clearly proven superior method. Family members or nonphysiotherapy hospital staff can easily be trained to perform effective preoperative chest physiotherapy, and this should be arranged at the time of the initial preoperative assessment. Even in patients with the most severe COPD, exercise tolerance with a physiotherapy program will improve. Little improvement is seen before 1 month. Among COPD patients, those with excessive sputum benefit the most from chest physiotherapy. - A comprehensive program of pulmonary rehabilitation involving physiotherapy, exercise, nutrition, and education can improve functional capacity for patients with severe COPD.43 These programs are usually of several months duration and are generally not an option in resections for malignancy, although for nonmalignant resections in severe COPD patients, rehabilitation should be considered. The benefits of short-duration rehabilitation programs before malignancy resection have not been fully assessed.

True or False Epidural anesthesia reduces hypoxic pulmonary vasoconstriction

False The Role of Regional Anesthesia - When compared with general anesthesia, regional anesthesia may be beneficial in reducing atelectasis, pneumonia, respiratory failure, and other pulmonary complications.126 Unfortunately, regional anesthesia as the sole anesthetic technique is impractical for open-lung cases; however, regional anesthesia can offer postoperative analgesia without the respiratory depressant effects of systemic opioids, which confers great benefits. Epidural anesthesia was found to reduce complications both for patients with normal FEV1, and those with airflow obstruction.127 The sympathectomy that is associated with neuraxial blockade causes vasodilation. Some have questioned whether epidural anesthesia would inhibit HPV, but because HPV is a locally-mediated event, epidural anesthesia has been found to be similar to IV or balanced techniques with regard to pulmonary gas exchange.128

True or False VATS is associated with lesser anti-angiogenic responsiveness in patients with non-small cell lung cancer as compared to open thoracotomy.

False Thoracoscopy - Advances in videoscopic technology have led to the increased use of thoracoscopy to replace open thoracotomy for a variety of diagnostic and therapeutic intrathoracic interventions (Table 30.4). Video-assisted thoracoscopic surgery (VATS) most often requires general anesthesia, and frequently requires lung isolation during ventilation. A video camera and surgical instruments are inserted via a trocar (Thoracoport) into the chest, allowing for visualization and manipulation of thoracic structures (Fig. 30.20). The surgeon has less ability to manually compress the lung during this procedure as compared to during an open thoracotomy, and therefore, a DLT may be preferable to a bronchial blocker to facilitate lung isolation. An arterial line is frequently inserted for VATS procedures, except with minimally invasive procedures in selected healthy patients. However, the anesthetic plan should account for the potential need for rapidly obtaining arterial blood gas samples or for possible hemorrhage, which may be difficulty to control during the endoscopic procedure. In addition to the potential for rapid massive hemorrhage, other complications associated with VATS include hypoxemia, cardiac/pulmonary artery/esophageal disruption, cardiac tamponade, left recurrent laryngeal nerve damage, and pulmonary edema. In cases of severe pulmonary compromise, VATS can be performed using epidural anesthesia in the spontaneously breathing patient who is sedated with a variety of techniques.175 The degree of postoperative pain is less with VATS as compared to an open procedure. For this reason, an epidural used solely for postoperative pain management is rarely indicated. Patients who undergo VATS have an improved quality of life for the first year postoperatively as compared to those who undergo open thoracotomy, suggesting that VATS should be the preferred surgical approach for lobectomy in stage I non-small cell lung cancer.176 VATS is associated with greater anti-angiogenic responsiveness in patients with non-small cell lung cancer as compared to open thoracotomy. The differences in anti-angiogenic responsiveness may have an important effect on cancer biology and recurrence after surgery.177 - Thoracoscopic sympathectomy for hyperhidrosis is an outpatient procedure. A DLT is preferred over a bronchial blocker because the procedure is bilateral, and the DLT can more easily switch from ventilation of one lung to the other. The procedure is performed in the supine position, and no chest tubes are inserted.

True or False Pressure-control ventilation has been shown to improve oxygenation versus volume-controlled ventilation for most patients

False Volume-Controlled Versus Pressure-Controlled Ventilation - Traditionally, volume-controlled ventilation has been used in the operating room for all types of surgery. The recent availability of anesthesia ventilators with pressure-control modes has made it possible to study and use this form of ventilation during thoracic surgery. Pressure-control ventilation has not been shown to improve oxygenation versus volume-controlled ventilation for most patients, although the peak airway pressures are lower.138 The decrease in peak pressure with pressure control ventilation may be largely in the anesthetic circuit and not at the distal airway.139 Pressure-control ventilation will avoid sudden increases in peak airway pressures that may result from surgical manipulation in the chest. This will be of benefit in patients at increased risk for lung injury from high volumes or pressures such as after lung transplantation or during a pneumonectomy.140 Because of the rapid changes of lung compliance that occur during pulmonary surgery, when pressure-control ventilation is used, the delivered tidal volume needs to be closely monitored because it may change suddenly.

True or False Fiberoptic Bronchoscopy to Verify Placement of a Double Lumen Tube: Visualize the bronchial (blue) cuff 4-5 mm beyond the carina. Ensure that the cuff is not too proximal or overinflated such as to herniate across the carina and obstruct the contralateral bronchus.

Fiberoptic Bronchoscopy to Verify Placement of a Double Lumen Tube 1. Insert the scope through the tracheal lumen. Visualize the carina distally. (Confirm that the tracheal orifice is within the trachea and not a bronchus, or that the tube is not displaced proximally such that the bronchial cuff fills the trachea.) 2. Visualize the bronchial (blue) cuff 1-2 mm beyond the carina. Ensure that the cuff is not too proximal or overinflated such as to herniate across the carina and obstruct the contralateral bronchus. 3. Insert the scope through the bronchial lumen. Visually confirm that the tip of the bronchial lumen is unobstructed. For left-sided tubes, visualize the bronchial carina distal to the tube tip. For right-sided tubes with a right upper lobe (RUL) ventilation port, visualize that the RUL bronchus is aligned with the ventilation port.

A Maximal oxygen consumption (V̇O2) max less than 10 mL/kg per min (or 40% of the predicted value) is associated with ____________ mortality A. Increased B. Decreased

Increased Cardiopulmonary Reserve - Maximal oxygen consumption (V̇O2 max) testing during exercise is also a strong predictor of patient outcome.31,40,65 A V̇O2 max less than 10 mL/kg per min (or 40% of the predicted value) is associated with increased mortality, whereas a V̇O2 max greater than 20 mL/kg per min is a favorable finding.66 These values may be roughly estimated by evaluating the patient's physical ability, in which the ability to climb five flights of stairs suggests a V̇O2 max greater than 20 mL/kg per min, and the inability to climb one flight of stairs suggests a V̇O2 max of less than 10 mL/kg per min.67 Cardiopulmonary exercise testing may be applied to evaluate patients prior to pulmonary surgery. For example, the inability to walk 500 meters in a 6-minute walk test was correlated with a significant increase in the duration of hospital stay, atrial fibrillation, and blood transfusion in patients undergoing pulmonary lobectomies.68

The most useful test of the gas exchange capacity of the lung is A. diffusing capacity for carbon monoxide (DLco) B. postresection (ppo) value for carbon monoxide (ppoDL) C. FEV1 D. PACO2

Lung Parenchymal Function - As important to the process of respiration as the mechanical delivery of air to the distal airways is the subsequent ability of the lung to exchange oxygen and carbon dioxide between the pulmonary vascular bed and the alveoli. Traditionally, arterial blood gas data such as partial pressure of oxygen in the blood (PaO2) less than 60 mm Hg or partial pressure of carbon dioxide in the blood (PaCO2) greater than 45 mm Hg have been used as cutoff values for pulmonary resection. Cancer resections have now been successfully performed singly or even combined with volume reduction in patients who do not meet these criteria, although they remain useful as warning indicators of increased risk. The most useful test of the gas exchange capacity of the lung is the diffusing capacity for carbon monoxide (DLco). The DLco correlates with the total functioning surface area of alveolar-capillary interface. The corrected DLco can be used to calculate a postresection (ppo) value using the same calculation as for the FEV1 (see Fig. 66-1). A ppoDLco less than 40% predicted correlates with both increased respiratory and cardiac complications and is usually independent of the FEV1.5 The DLco, but not the FEV1, is negatively affected by preoperative chemotherapy and may be the most important predictor of complications in this subgroup of patients. Some authors feel a higher cutoff risk-threshold for ppoDLco of less than 50% may be more appropriate.6 The National Emphysema Treatment Trial has shown that patients with a preoperative FEV1 or DLco less than 20% had an unacceptably frequent perioperative mortality rate.7 These can be considered as the absolute minimal values compatible with a successful outcome.

Patients having ________-sided thoracotomies tend to have a larger shunt and lower PaO2 during OLV A. Left B. Right

Prediction of Hypoxemia During One-Lung Ventilation - The problem of hypoxemia during OLV has prompted much research in thoracic anesthesia. Hypoxemia during OLV is predictable (see Box 66-4), preventable, and treatable in the vast majority of cases.141 Preoperative Ventilation-Perfusion Scan - The shunt and PaO2 during intraoperative OLV are highly correlated with the fractional perfusion of the ventilated lung as determined by a preoperative ventilation perfusion scan.142 Patients with long-standing unilateral disease on the operative side develop a unilateral decrease of ventilation and perfusion and tolerate OLV very well. Similarly, patients who intraoperatively have a higher proportion of gas exchange in the dependent lung during OLV tend to have better oxygenation during OLV. Side of Operation - Patients having right-sided thoracotomies tend to have a larger shunt and lower PaO2 during OLV because the right lung is larger and normally 10% better perfused than the left. The overall mean PaO2 difference between left and right thoracotomies during stable OLV is approximately 100 mm Hg.143 Two-Lung Oxygenation - Patients who have better PaO2 levels during TLV in the lateral position tend to have better oxygenation during OLV. These patients may have better abilities to match ventilation and perfusion (individual variability of HPV response) and/or they may have less atelectasis in the 1973dependent lung. This is a particularly relevant consideration in trauma patients who may require a thoracotomy but have a contusion of the dependent lung. Preoperative Spirometry - Studies consistently show that when the previous factors are controlled, patients with better spirometric lung function preoperatively are more likely to desaturate and have lower PaO2 values during OLV. Clinically this is evident because emphysematous lung volume reduction patients generally tolerate OLV very well. The explanation is not clear but may be related to maintenance of a more favorable FRC in patients with obstructive airways disease during OLV with an open hemithorax because of the development of auto-PEEP.54

This esophagectomy is commonly a two-phase procedure. The first phase involves a laparotomy performed with the patient in the supine position and the creation of a neoesophagus tube using the stomach. The second phase involves a right-sided thoracotomy in the left lateral position and esophageal reconstruction through the thoracic route. A. Transthoracic B. Transhiatal C. Minimally Invasive Approach

Transthoracic Approach - Transthoracic esophagectomy is commonly a two-phase procedure. The first phase involves a laparotomy performed with the patient in the supine position and the creation of a neoesophagus tube using the stomach. The second phase involves a right-sided thoracotomy in the left lateral position and esophageal reconstruction through the thoracic route. Some surgeons may perform this procedure through an extended left thoracoabdominal incision. - The anesthetic management for these patients includes the use of standard monitors, an invasive arterial line, and a CVP catheter to accommodate the large fluid shifts. Access to the right internal jugular is not a problem, but there is always the possibility of a surgical esophageal anastomosis in the left neck contraindicating access to the left internal jugular. A thoracic epidural catheter is usually placed to provide postoperative analgesia. Because of the wide number of dermatomes that must be covered for both incisions by the epidural infusion, it is best to use hydrophilic opioids (such as hydromorphone) in combination with local anesthetics in preference to lipophilic opioids. Most patients with an esophageal carcinoma have gastric reflux; for this reason, precautions (including a rapid-sequence induction with cricoid pressure) should be taken to protect the airway against aspiration. - During the second phase (right thoracotomy), a left-sided DLT or a right-sided bronchial blocker is required to facilitate lung collapse. Because esophagectomy requires a prolonged period of OLV, this procedure is marked by an important inflammatory response. Michelet and colleagues202 have shown that the use of protective ventilatory strategies during OLV decreases the proinflammatory systemic response. This decreased response can be achieved by delivering 5 mL/kg of tidal volume and a PEEP of 5 cm H2O to the dependent lung, instead of the 9 mL/kg of tidal volume that is conventionally used during esophagectomy. - Manipulation of the esophagus during thoracotomy may compromise venous return, which can cause hypotensive episodes. Early extubation in the operating room is encouraged if the patient meets the standard criteria for extubation. If extubation is not possible, the DLT should be replaced with an SLT and mechanical ventilation used in the postoperative period.

True or False Most of the body's physiologic functions, including HPV, are inhibited during hypothermia.

True Anesthetic Management - The development of thoracic anesthesia and thoracic surgery was delayed more than 50 years after the introduction of ether because anesthesiologists could not manage patients during mask anesthesia with spontaneous ventilation and an open chest. These patients developed what was originally called the "pneumothorax syndrome."103 The respiratory systems of mammals do not function adequately with an open hemithorax because of two physiologic problems. The first is paradoxical ventilation (also called pendelluft), in which gas moves into the open-chest lung from the intact lung during expiration and then reverses flow during inspiration. This leads to hypercapnia and hypoxemia. The second is caused by the swinging motion of the mediastinum between the hemithoraces during the respiratory cycle, which interferes with cardiac preload and causes hemodynamic instability. In the early 1900s, several pioneers, such as the New Orleans surgeon Matas, advocated positive-pressure ventilation and a primitive form of endotracheal ventilation, which had been demonstrated to be safe in animal experiments, for thoracic anesthesia. Modern methods incorporating OLV have evolved from this. Essentially, any anesthetic technique that provides safe and stable general anesthesia for major surgery can and has been used for lung resection. Fluid Management - Because of hydrostatic effects, excessive administration of intravenous fluids can cause increased shunting and subsequently lead to pulmonary edema of the dependent lung, particularly during prolonged surgery.104 Because the dependent lung is the lung that must carry on gas exchange during OLV, it is best to be as judicious as possible with fluid administration. Intravenous fluids are administered to replace volume deficits and for maintenance only during lung resection anesthesia. No volume is given for "third space" losses (Box 66-8). It is possible that there is no "third space." Temperature (See Chapter 54) - Maintenance of body temperature can be a problem during thoracic surgery because of heat loss from the open hemithorax. This is particularly a problem at the extremes of the age spectrum. Most of the body's physiologic functions, including HPV, are inhibited during hypothermia. Increasing the ambient room temperature, fluid warmers and the use of lower and/or upper body forced-air patient warmers are the best methods to prevent inadvertent intraoperative hypothermia.

True or False A flexible fiberoptic bronchoscopic examination is necessary to assess a distorted area of the airway before selection of a specific tube or blocker to achieve OLV

True Difficult Airways and One-Lung Ventilation - Some patients requiring OLV are identified during preoperative evaluation to have a potentially difficult airway. Others present with unexpected difficulty to intubate after induction of anesthesia. Between 5% and 8% of patients with primary lung carcinoma also have a carcinoma of the pharynx, usually in the epiglottic area.89 1963Many of these patients have had previous radiation therapy on the neck or previous airway surgery, such as hemimandibulectomy or hemiglossectomy, making intubation and achievement of OLV difficult because of distorted upper airway anatomy. In addition, a patient who requires OLV may have distorted anatomy at or beyond the tracheal carina, such as a descending thoracic aortic aneurysm compressing the entrance of the left mainstem bronchus or an intraluminal or extraluminal tumor near the tracheobronchial bifurcation that makes the insertion of a left-sided DLT relatively difficult or impossible. Such anomalies can be detected by reviewing the chest radiographs and CT scans of the chest. A flexible fiberoptic bronchoscopic examination is necessary to assess a distorted area of the airway before selection of a specific tube or blocker to achieve OLV.- - In patients who require OLV and present with a difficult airway, the primary goal is to establish an airway with an SLT placed orally with the aid of a flexible fiberoptic bronchoscope, after appropriate airway anesthesia is achieved. In selected patients who seem easy to ventilate, this may be performed after induction of anesthesia with a bronchoscope or with a videolaryngoscope.90 Once the SLT is in place, an independent bronchial blocker can be passed. If the patient requires OLV and cannot be intubated orally, an awake nasotracheal intubation can be performed with an SLT and, once the airway is established, then a bronchial blocker can be used. - An alternative to achieve OLV in a patient with a difficult airway is to intubate the patient's trachea with an SLT; then a DLT-SLT tube-exchange catheter can be used to replace the existing SLT with a DLT after general anesthesia is induced. For a DLT, the exchange catheter should be at least 83 cm. A 14-Fr exchange catheter can be used for 41-Fr and 39-Fr DLTs; for 37-Fr or 35-Fr DLTs, an 11-Fr exchange catheter is used. Specially designed exchange catheters for DLTs are available with a softer distal tip to try to decrease the risk of distal airway trauma (e.g., Cook Exchange Catheter, Cook Critical Care). - The exchange catheter, SLT, and the DLT combination should be tested in vitro before the exchange. A sniffing position facilitates tube exchange. After the exchange catheter is lubricated, it is advanced through the SLT. The catheter should not be inserted deeper than 24 cm at the lips to avoid accidental rupture or laceration of the trachea or bronchi. After cuff deflation, the SLT is withdrawn. Then the endobronchial lumen of the DLT is advanced over the exchange catheter. It is optimal to use a videolaryngoscope during the tube exchange to guide the DLT through the glottis under direct vision (Fig. 66-19). If a videolaryngoscope is not available, having an assistant perform standard laryngoscopy during tube exchange partially straightens out the alignment of the oropharynx and glottis and facilitates the exchange. Proper final position of the DLT is then achieved with auscultation and bronchoscopy.

True or False Auto-PEEP has been found to develop in most COPD patients during one-lung anesthesia.

True Flow Limitation - Patients with severe COPD are often "flow-limited" even during tidal volume expiration at rest.38 Flow limitation is present in normal patients only during a forced expiratory maneuver. Flow limitation occurs when an equal pressure point develops in the intrathoracic airways during expiration. During quiet expiration in the normal patient, the pressure in the lumen of the airways always exceeds the intrapleural pressure because of the upstream elastic recoil pressure that is transmitted from the alveoli. The effect of this elastic recoil pressure diminishes as air flows downstream in the airway. With a forced expiration, the intrapleural pressure may equal the intraluminal pressure at a certain point, the equal pressure point, which then limits the expiratory flow. Then, any increase in expiratory effort will not produce an increase in flow at that given lung volume.39 - Flow limitation occurs particularly in emphysematous patients who primarily have a problem with loss of lung elastic recoil and have marked dyspnea on exertion. Flow limitation causes dyspnea because of stimulation of mechanoreceptors in the muscles of respiration, the thoracic cage, and the airway distal to the equal pressure point. Any increase in the work of respiration leads to increased dyspnea. This variable mechanical compression of airways by overinflated alveoli is the primary cause of the airflow obstruction in emphysema. - Severely flow-limited patients are at risk for hemodynamic collapse with the application of positive-pressure ventilation because of dynamic hyperinflation of the lungs. Even the modest positive airway pressures associated with manual ventilation with a bag/mask at induction can lead to hypotension because these patients have no increased resistance to inspiration but a marked obstruction of expiration. In some of these patients, this has contributed to the "Lazarus" syndrome, in which patients have recovered from a cardiac arrest only after resuscitation and positive-pressure ventilation were discontinued.40 Auto-PEEP - Patients with severe COPD often breathe in a pattern that interrupts expiration before the alveolar pressure has fallen to atmospheric pressure. This incomplete expiration is caused by a combination of factors that include flow limitation, increased work of respiration, and increased airway resistance. This interruption leads to an elevation of the end-expiratory lung volume above 1949the FRC. This positive end-expiratory pressure (PEEP) in the alveoli at rest has been termed auto-PEEP or intrinsic PEEP. During spontaneous respiration, the intrapleural pressure has to be decreased to a level that counteracts auto-PEEP before inspiratory flow can begin. Thus COPD patients can have an increased inspiratory load added to their already increased expiratory load. - Auto-PEEP becomes even more important during mechanical ventilation. It is directly proportional to tidal volume and inversely proportional to expiratory time. The presence of auto-PEEP is not detected by the manometer of standard anesthesia ventilators. It can be measured by end-expiratory flow interruption, a feature available on most intensive care ventilators. Auto-PEEP has been found to develop in most COPD patients during one-lung anesthesia.41

True or False The anesthesiologist's aim, to optimize pulmonary blood flow redistribution during OLV, is to maintain the ventilated lung as close as possible to its FRC while facilitating collapse of the nonventilated lung to increase its PVR.

True Hypoxemia - A major concern that influences anesthetic management for thoracic surgery is the occurrence of hypoxemia during OLV. There is no universally acceptable figure for the safest lower limit of oxygen saturation during OLV. A saturation greater than or equal to 90% (PaO2 >60 mm Hg) is commonly accepted, and for brief periods a saturation in the high 80s may be acceptable in patients without significant comorbidity. However, the lowest acceptable saturation will be higher in patients with organs at risk of hypoxia because of limited regional blood flow (e.g., coronary or cerebrovascular disease) and in patients with limited oxygen transport (e.g., anemia or decreased cardiopulmonary reserve). It has been shown that during OLV, patients with COPD desaturate more quickly during isovolemic hemodilution than normal patients - Previously, hypoxemia occurred frequently during OLV. Reports for the period between 1950 and 1980 describe an incidence of hypoxemia (arterial saturation <90%) of 20% to 25%.112 Current reports describe an incidence of less than 5%.113 This improvement is most likely a result of several factors: improved lung isolation techniques such as routine fiberoscopy to prevent lobar obstruction from DLTs, improved anesthetic agents that cause less inhibition of HPV, and better understanding of the pathophysiology of OLV. The pathophysiology of OLV involves the body's ability to redistribute pulmonary blood flow to the ventilated lung. Several factors aid and impede this redistribution and these are under the control of the anesthesiologist to a variable degree. These factors are illustrated in Figure 66-25. The anesthesiologist's goal during OLV is to maximize pulmonary vascular resistance (PVR) in the nonventilated lung while minimizing PVR in the ventilated lung. The key to understanding this physiology is the appreciation that PVR is correlated with lung volume in a hyperbolic fashion (Fig. 66-26). PVR is lowest at FRC and increases as lung volume rises or falls above or below FRC.114 The anesthesiologist's aim, to optimize pulmonary blood flow redistribution during OLV, is to maintain the ventilated lung as close as possible to its FRC while facilitating collapse of the nonventilated lung to increase its PVR. Intraoperative Position (See Chapter 41) - Most thoracic surgery is performed with patients in the lateral position. Patients having OLV in the lateral position have significantly better PaO2 levels than patients during OLV in the supine position.115 This applies both to patients with normal lung function and to those with COPD (Fig. 66-27).116

True or False In bronchial thermoplasty, the patient is not "cured" upon completion of the procedure, and postoperative complications may include bronchospasm, hypoxemia, and respiratory distress.

True Thoracic Surgical Concerns Asthma Treatment - Asthma that is refractory to medical therapy may be treated with thermal ablation of the bronchial smooth muscle to reduce its responsiveness. Bronchial thermoplasty is performed through a fiberoptic bronchoscope. The procedure may be performed under general anesthesia or monitored anesthesia care (MAC). Either approach has positive and negative attributes. Monitored anesthesia care provides the advantage of not needing to instrument the airway; however, suppressing the cough and irritant reflexes while preserving spontaneous ventilation can pose a challenge. Agents which provide sedation while minimizing airway compromise and bronchial irritation (such as dexmedetomidine) are beneficial.164 General anesthesia provides control of the airway and breathing and allows a deeper state of reflex attenuation; however, weaning and extubation may pose challenges. The procedure is completed in three stages, with a few weeks between procedures. Therefore, the patient is not "cured" upon completion of the procedure, and postoperative complications may include bronchospasm, hypoxemia, and respiratory distress.

True or False Although 87% of patients with lung cancer will die of their disease, the 13% cure rate represents approximately 26,000 survivors per year in North America. Surgical resection is responsible for essentially all of these cures.

True Thoracic anesthesia encompasses a wide variety of diagnostic and therapeutic procedures involving the lungs, airways, and other intrathoracic structures. As the patient population presenting for noncardiac thoracic surgery has evolved, so have the anesthetic techniques to manage these patients. At the beginning of the last century, thoracic surgery was primarily done to treat infectious indications (e.g., lung abscess, bronchiectasis, empyema). Although these cases still present for surgery in the postantibiotic era, now the most common indications are related to malignancies (i.e., pulmonary, esophageal, and mediastinal). In addition, the past 2 decades have seen the beginning of surgical therapy for end-stage lung diseases with procedures such as lung transplantation and lung volume reduction. Fundamental to anesthetic management for the majority of thoracic procedures are two techniques: (1) lung isolation to facilitate surgical access within the thorax and (2) management of one-lung anesthesia. In this chapter, we initially discuss preanesthetic assessment for thoracic surgery, outline intraoperative management principles common to most thoracic surgical procedures, discuss specific anesthetic considerations in common and less common surgical operations, and we finally finish with a description of postoperative management issues in thoracic surgical patients. Preoperative Evaluation of the Thoracic Surgery Patient (See Chapter 38) - Preoperative anesthetic assessment before chest surgery is a continually evolving science and art. Recent advances in anesthetic management, surgical techniques, and perioperative care have expanded the realm of patients now considered to be "operable."1 This discussion focuses primarily on preanesthetic assessment for pulmonary resection surgery in cancer patients. However, the basic principles described apply to all other types of nonmalignant pulmonary resections and to other chest surgeries. Although 87% of patients with lung cancer will die of their disease, the 13% cure rate represents approximately 26,000 survivors per year in North America. Surgical resection is responsible for essentially all of these cures. A patient with "resectable" lung cancer has a disease that is still local or locoregional in scope and can be encompassed in a plausible surgical procedure. An operable patient is someone who can tolerate the proposed resection with acceptable risk. - The patient is commonly assessed initially as an outpatient, often not by anesthesia personnel. Yet it is necessary to organize and standardize the approach to preoperative evaluation for these patients into two temporally disjointed phases: the initial (clinic) assessment and the final (day of admission) assessment. Elements vital to each assessment are described. Increasingly, thoracic surgeons are performing lung-sparing resections such as sleeve-lobectomies or segmentectomies, and resections with minimally invasive techniques such as video-assisted thoracoscopic surgery (VATS) or robotic surgery. The postoperative preservation of respiratory function is proportional to the amount of functioning lung parenchyma preserved. To assess patients with limited pulmonary function, these newer surgical options need to be understood in addition to the conventional open lobectomy or pneumonectomy. - Preoperative assessment should identify patients at increased risk and that risk assessment should stratify the perioperative management and focus resources on the high-risk patients to improve their outcome. This is the primary function of the preanesthetic assessment. There are occasions when the anesthesiologist should contribute his or her opinion about whether a specific high-risk patient will tolerate a specific surgical procedure. This may occur preoperatively but also occurs intraoperatively when the surgical findings suggest that a planned procedure, such as a lobectomy, may require a larger resection, such as a pneumonectomy. For these reasons, it is necessary that the anesthesiologist have a complete preoperative knowledge of the patient's medical status and the pathophysiology of lung resection surgery, which are needed for a proper anesthetic to be given. Prethoracotomy assessment naturally involves all of the factors of a complete anesthetic assessment: past history, allergies, medications, upper airway, and so on. This section concentrates on the additional information, beyond a standard anesthetic assessment, that the anesthesiologist needs to manage a patient undergoing a pulmonary resection.


Conjuntos de estudio relacionados

Financial Regulatory System Quiz 1

View Set

LC4 Equilibrium: How Supply Demand Determine Prices

View Set

OFFICIAL Arson/Explosives (right answers)

View Set

Air Equipment Preparation Introductory Course (EPIC)

View Set

NCLEX Pediatrics Neuro and Musculoskeletal

View Set